Download NEET PG 2018 Question Paper with Answers

Download NEET PG (The National Eligibility cum Entrance Test Postgraduate) 2018 Question Paper with Answers

1.NervesofBranchialarchderivedfrom:
a)Mesoderm
b)Endoderm
c)Neuralcrest
d)Neuroectoderm
CorrectAnswer-C
Answer-C.Neuralcrest
Branchialorpharyngealarchesaremassesofmesodermcovered
byectodermandlinedbyendoderm.Withinthesemasses,muscular
andskeletalcomponentsdevelop,aswellasaorticarchesand
nervenetworks.Thearchesareseparatedbygrooves,visibleon
thesurfaceoftheembryoaspharyngealcleftsandintheinterioras
thepharyngealpouches
Inthehumanembryo,thearchesarefirstseenduringthe4thweek
ofdevelopment.
Theyappearasaseriesofoutpouchingsofmesodermonboth
sidesofdevelopingpharynx.
Theneuralcrestarebilaterallypairedstripsofcellsarisinginthe
ectodermatthemarginsoftheneuraltube.Thesecellsmigrateto
manydifferentlocationsanddifferentiateintomanycelltypeswithin
theembryo.
NeuralCrestDerivatives
Akeyfeatureofneuralcrestisthemigrationintootherembryonic
tissuestoformspecificneuralandnon-neuralpopulationsand
structures.
Cranialneuralcrest
migration-dorsolaterallyandintopharyngealarches
craniofacialmesenchyme-cartilage,bone,cranialneurons,glia,

andconnectivetissuesoftheface
pharyngealarchesandpouches-thymiccells,toothodontoblasts,
middleearbones(ossicles),striavasculariscells,andjaw
(mandible)
Inthebodyregion,neuralcrestcellsalsocontributetheperipheral
nervoussystem(bothneuronsandglia)consistingofsensory
ganglia(dorsalrootganglia),sympatheticandparasympathetic
gangliaandneuralplexuseswithinspecifictissues/organs.
Intheheadregion,neuralcrestcellsmigrateintothepharyngeal
archesformingectomesenchymecontributingtissueswhichinthe
bodyregionaretypicallyderivedfrommesoderm(cartilage,bone,
andconnectivetissue).
NeuralCrestOrigin
System
CellType
PeripheralNervous Neurons-sensoryganglia,sympatheticand
System
parasympatheticganglia,entericnervous
(PNS)
system,andplexuses
Neuroglialcells,olfactoryensheathingcells
Schwanncells
Endocrine
Adrenalmedulla
Calcitonin-secretingcells
CarotidbodytypeIcells
Integumentary
Epidermalpigmentcells
Facialcartilage
Facialandanteriorventralskullcartilageand
andbone
bones
Sensory
Innerear,cornealendotheliumandstroma
Connectivetissue Toothpapillae
smoothmuscle,andadiposetissueofskinof
headandneck
Connectivetissueofmeninges,salivary,
lacrimal,thymus,thyroid,andpituitaryglands
Connectivetissueandsmoothmuscleinarteries
ofaorticarchorigin

2.Hardpalatecontains:
a)Keratinised,submucosa,minorsalivarygland
b)Keratinised,absentsubmucosallayer,minorsalivarygland
c)Nonkeratinised,submucosallayer,minorsalivarygland
d)Nonkeratinised,absentsubmucosa,minorsalivarygland
CorrectAnswer-A
Answer-A.Keratinised,submucosa,minorsalivarygland
Thehardpalateislocatedontheroofoftheoralcavity,posterior
andmedialtothealveolarprocessofthemaxilla.
Thebonystructureisformedbythepalatineprocessesofthe
maxillaandthehorizontalplatesofthepalatinebones.
Theperiosteumiscoveredbyafirmlyattachedmucosacentrally,
althoughasubmucosaisapparentlaterallycontainingvessels.The
hardpalateiscontinuouswiththesoftpalateposteriorly.
MacroscopicFeatures
Thehardpalateistypicallyapalepinkcolourandmayhavean
orangepeelappearancefromthepalatinesalivaryglands(more
commonposteriorly).
MicroscopicFeatures
Thehardpalateislinedwithakeratinisingstratifiedsquamous
epithelium
,tightlyboundtotheunderlyingperiosteumofthe
palatinebone/maxilla.Thereisminimalsubmucosa,which
becomesmoreprominentposteriorly.




3.Whatisthetensorofvocalcords:
a)Cricothyroid
b)LateralCricoarytenoid
c)Thyroarytenoids
d)Posteriorcricoarytenoids
CorrectAnswer-A
Answer-A.Cricothyroid
Cricothyroid:
Tensorofvocalcords.
Lateralcricoarytenoid: Abductorofvocalcords.
Thyroarytenoid:
Relaxorofvocalcords.
Posterior
Abductorofvocalcords
cricoarytenoid:
MusclesactingontheLarynx
Movement

Muscles
ElevationofLarynx Thyrohyoid,mylohyoid
Depressionoflarynx Sternothyroid,sternohyoid
Openinginletof
Thyroepiglotticus
larynx
Closinginletof
Aryepiglotticus
larynx
Abductorofvocal Posteriorcricoarytenoidonly
cords
Adductor

Lateralcricoarytenoidtransverse&oblique
ofvocalcords
arytenoids
Tensorofvocal

cords
Cricothyroid
Relaxerofvocal
Thyroarytenoid
cords

4.Uretericbuddevelopsfrom:
a)Mesonephros
b)Metanephros
c)Pronephros
d)Genitalsinus
CorrectAnswer-A
Answer-A.Mesonephros
Theuretericbud,alsoknownasthemetanephrogenic
diverticulum
,isaprotrusionfromthemesonephricductduringthe
developmentoftheurinaryandreproductiveorgans.
Itlaterdevelopsintoaconduit(channel)forurinedrainagefromthe
kidneys,which,incontrast,originatefromthemetanephric
blastema.
Themetanephrogenicblastemaormetanephricblastema(or
metanephricmesenchyme,ormetanephricmesoderm)isoneofthe
twoembryologicalstructuresthatgiverisetothekidney,theother
beingtheuretericbud.

5.AboutWeber'ssyndromewhichis
incorrect:
a)Contralateralhemiplegia
b)IpsilateralOculomotornervepalsy
c)ContralateralParkinsonism
d)Anteriorcerebralpeduncle
CorrectAnswer-D
Answer-D.Anteriorcerebralpeduncle
Weber'ssyndrome(superioralternatinghemiplegia)isaform
ofstrokecharacterizedbythepresenceofan
ipsilateraloculomotornervepalsyand
contralateralhemiparesisorhemiplegia.
Itiscausedbymidbraininfarctionasaresultofocclusionof
theparamedianbranchesoftheposteriorcerebralarteryor
basilarbifurcationperforatingarteries.
Thislesionisusuallyunilateralandaffectsseveralstructuresin
themidbrain:

Contralateralparkinsonismbecauseitsdopaminergicprojections
tothebasalgangliainnervatetheipsilateralhemispheremotorfield,
leadingtoamovementdisorderofthecontralateralbody.
Contralateralhemiparesisandtypicaluppermotorneuronfindings.
Itiscontralateralbecauseitoccursbeforethedecussationinthe
medulla.
Difficultywithcontralaterallowerfacialmusclesandhypoglossal
nervefunctions.
IpsilateralOculomotornervepalsywithadroopingeyelidand
fixedwidepupilpointeddownandout.Thisleadstodiplopia.


6.AboutSibson'sfasciawhichisincorrect:
a)Attachedtotheinnerborderof2ndrib
b)Coversapicalpartoflung
c)Partofscalenusanteriormuscle
d)Vesselpassabovethefascia
CorrectAnswer-A
Answer-A.Attachedtotheinnerborderof2ndrib
ThesuprapleuralmembraneisknownasSibson'sfascia.
Itreferstoathickeningofconnectivetissuethatcoverstheapexof
eachhumanlung.
Itattachestotheinternalborderofthefirstribandthetransverse
processesofvertebraC7.
Suprapleuralmembrane/Sibson'sfascia


7.Thrombosisofposteriorinferiorcerebellar
arterycauses:
a)Lateralmedullarysyndrome
b)Webersyndrome
c)Medialmedullarysyndrome
d)none
CorrectAnswer-A
Answer-A.Lateralmedullarysyndrome
Theposteriorinferiorcerebellarartery(PICA),thelargestbranch
ofthevertebralartery,isoneofthethreemainarterialblood
suppliesforthecerebellum,partofthebrain.
Occlusionoftheposteriorinferiorcerebellararteryoroneofits
branches,orofthevertebralarteryleadstolateralmedullary
syndrome
alsocalledWallenbergsyndrome

8.Broca'sareasituatedin:
a)Inferiorfrontalgyrus
b)Superiortemporalgyrus
c)Angulargyrus
d)Noneoftheabove
CorrectAnswer-A
Answer-A.Inferiorfrontalgyrus
Broca'sarea
ortheBrocaarea(44)isaregioninthefrontallobeof
thedominanthemisphere(usuallytheleft)ofthehominidbrainwith
functionslinkedtospeechproduction.
Inabilitytospeakafterinjurytotheposteriorinferiorfrontalgyrusof
thebrain.
PierrePaulBrocaheidentifiedthisregion,knownasBroca'sarea.
DifficultyinlanguageproductionasBroca'saphasia,also
calledexpressiveaphasia.
Broca'sareaisnowtypicallydefinedintermsofthepars
opercularis
andparstriangularisoftheinferiorfrontalgyrus.


BROCA'SAREA(showninred).Coloredregionisparsopercularis
andparstriangularisoftheinferiorfrontalgyrus.Broca'sareaisnow
typicallydefinedintermsoftheparsopercularisandpars
triangularisoftheinferiorfrontalgyrus.

9.Acutetonsillitiseffectswhichnerve:
a)GlossopharyngealNerve
b)FacialNerve
c)Trigeminalnerve
d)VagusNerve
CorrectAnswer-A
Answer-A.GlossopharyngealNerve
Thenervessupplyingthepalatinetonsilscomefromthemaxillary
divisionofthetrigeminalnerveviathelesserpalatinenerves,and
thetonsillarbranchesoftheglossopharyngealnerve.The
glossopharyngealnervecontinuespastthepalatinetonsiland
innervatestheposterior1/3ofthetonguetoprovidegeneraland
tastesensation.Thisnerveismostlikelytobedamagedduring
atonsillectomy,whichleadstoreducedorlostgeneralsensation
andtastesensationtotheposteriorthirdofthetongue.
Relationsoftonsillarbed



10.StructuresnotpassingthroughAortic
opening:
a)Azygosvein
b)Aorta
c)Thoracicduct
d)Vagus
CorrectAnswer-D
Answer-D.Vagus
Aorticopening(Aortichiatus)isoneofthethreemajorapertures
throughthediaphragm&liesatthelevelofT12.
Severalstructurespassthroughtheaortichiatus:aorta,azygos
vein,thoracicduct.
Vagus
passesthroughtheoesophagealhiatus.

11.WhatshouldbethevalueofBMItobe
consideredas"Lethal"inmen?
a)12
b)18
c)13
d)14
CorrectAnswer-C
Answer:C?13
BMIValueof13isconsideredas"Lethal"inmen.
Bodymassindex(BMI)isanestimateoftotalbodyfatmass
Simplyanindexofweightforheight.
Formula:
Weightinkilogramsdividedbythesquareofheightinmeters.
Bodymassindex=Weight(kg)/(Height)2(m)
Uses:
Usedtoclassifyanddefineunderweight,overweight&obesityin
adults.
Classification&Metrics:
WorldHealthorganizationcategorizedBMIvaluesinto:
Classification
BMIvalue
Normalweight
18.5to24.9kg/m2
Underweight
<18.5
Pre-obesity(Pre-OB)
25to29.9kg/m2
Obesity(OB)
>30kg/m2
ClassIobesity
30.0-34.9kg/m2
ClassIIobesity
ClassIIIobesity(Morbidform)

ClassIIIobesity(Morbidform)
35.0-39.9kg/m2
Morethan40kg/m2
BMIvalueconsideredlethalforMenis13
FemaleswithstandandsurviveevenatlowerBMIrateupto11.

12.Whichreceptorsareblockedin
MyastheniaGravis?
a)Achreceptors
b)Ca++receptors
c)Na2+receptors
d)Opioidreceptors
CorrectAnswer-A
Answer:A-Achreceptors
Anautoimmunedisease
Antibodymediatedautoimmuneattackofacetylcholine
receptors.
Inabilityofneuromuscularjunctionstotransmitsignalsfromnerveto
muscle
Resultinginmuscleweaknessandfatigability.
Unresponsiverespiratorymusclesmaycauserespiratoryfailurein
severecases.
Analysisofneuromusculartransmissionreveal
Decreaseinacetylcholinereceptors(AChRs)
Lossofpost-junctionalfolds.
Circulatingantibodiestoacetylcholinereceptorspositive.
Diagnosis:
Clinicaltest:
Strengthimprovementinresponsetoadministrationof
anticholinesteraseagents.
Treatment:
Anti-cholinesterasedrugs:
Allowsaccumulationoflargeramounts
ofacetylcholineinsynapticjunctions.
Neostigmine.

13.WhatisthecharacteristicpatternseeninBrownsequard
syndrome-
a)C/Llossofjointsenseandposition
b)C/Llossofpain
c)I/Llossofcompletesensoryfunctions
d)C/Lmotorfunctions
CorrectAnswer-B
Answer:B-Contralaterallossofpainsensations
Ifthespinalcordiscompletelytransected?Allthesensationsand
motorfunctionsdistaltosegmentoftransectionareblocked.
Transectionofspinalcordonsinglesideresultsin"Brown
SequardSyndrome"
Functionsaffected:

Function
Position
Description
affected
Motor

Ipsilateralside
Completelossofmotor
functionsbelowthelevelof
transection
Sensory
Combinedeffects Somesensoryfunctionsare
observed
lostontransectedsideand

othersonoppositeside.
Onthe
Resultofdisturbancein
Contralateral
Spinothalamicpathway.
side:
Lossobserved2to6
Sensationofpain, segmentsbelowthelevelof
cold,&heat
transection.
DiscreteCrude
Poorlylocalized(Still
touch
persists).


Partialtransmissionoccursin
oppositeSpinothalamictract

14.WhenVa/Qisinfinity?
a)PartialpressureofO2becomeszero
b)NoexchangeofO2&CO2
c)PartialpressureofCO2alonebecomeszero
d)PartialpressurebothCO2andO2remainnormal
CorrectAnswer-B
Answer:B-NoexchangeofO2&CO2
Varepresentstheventilationinalveoli.
Qrepresentsthebloodflowthroughthealveolus.
TheratioofVaandQexplainstherespiratoryexchangewhenthere
isanimbalancebetweenalveolarventilationandalveolarbloodflow.
Theventilation-perfusionratioisconsiderednormalthereisanequal
amountofbloodflow&ventilationthroughalveolus.
Va/qratioisZero-
Thereisinadequateornil,butperfusionpersists
Va/qratioisinfinity-
Thereisadequateventilationbutnoperfusion
.
BothintheZeroandinfinityVa/Qratio,thereisnoexchangeof
gasesthroughtherespiratorymembranesofcorrespondingalveoli.
WhenVa/Qratioisinfinity,
Thealveolarairequilibratesthehumidifiedinspiredair
Noexchangeofoxygenandcarbon-di-oxideoccurs.
PartialpressuresoftheO2andCO2are149mmHg&0mmHg
respectively

15.Cwaveisseenin-
a)Iso-volumetriccontraction
b)Slowfillingatendofdiastole
c)Endofsystole
d)Startofdiastole
CorrectAnswer-A
Ans.A.Iso-volumetriccontraction
Cwave:
ProducedbybulgingoftricuspidvalveintorightatriumduringIso-
volumetriccontractionofrightventricle.

16.Alphawavesareseenduring?
a)Sleep
b)REMmovements
c)Relaxedstate
d)Activestate
CorrectAnswer-C
Answer:C-Relaxedstate
Regardedasnormalwavefront
Occurwhenatrestandeyesclosedwithactive/wanderingmind
ie.,associatedwithadecreasedlevelofattention(either
duringrelaxed/subconsciousthinking
)
RegularRhythm(Frequency8-13HzAmplitude-50-100V)

17.Inhypovolemicshockthereis-
a)Afferentarteriolarconstriction
b)Efferentarteriolarconstriction
c)Increasedbloodflowtokidney
d)Normalcardiacoutput
CorrectAnswer-A
Answer:A.Afferentarterioleconstriction
Inadequatecirculationvolume.
Poorvenousreturntoheartwilldecreasethestrokevolume&
cardiacoutput.
Compensationbytachycardia&increasedsystemicvascular
resistance(SVR).
Becomecoldperipherally(shutdown).
Mostcommoncauses-Fluidlossofanyetiology
Hemorrhage
Salt&waterloss
Sepsis
Burns

18.Componentsresponsibleforcounter
currentmechanisminkidneyareall
except:

a)Sodiumoutflowinthickascendinglimb
b)Wateroutflowinthindescendinglimb
c)Sodiumoutflowinthinascendinglimb
d)FlowoftubularfluidfromPCTtoDCT
CorrectAnswer-C
Answer:C.Sodiumoutflowinthinascendinglimb
Counter-currentsystemoccursinkidney
Asysteminwhichinflowrunsparallelto,counterto,andinclose
proximitytotheoutflowforsomedistance.
2countercurrentmechanismsavailable:
CountercurrentmultiplieratLoopofHenle
Generatehighmedullaryosmoticgradientpressure
Countercurrentexchangeratvasarectaofmedullarycapillaries
Helpsinmaintainthemedullaryosmoticpressuregradient
Substancesinvolvedincountercurrentmechanisminclude:
Sodiumactivelyabsorbedwithco-transportofpotassium&chlorine
inthickascendinglimbofloopofHenle.
WaterreabsorbedinthindescendinglimbofloopofHenle.
Ureadiffuseoutfromthemedullarycollectingductsintomedullary
interstitium.

19.Glucoseisabsorbedinintestineby?
a)Secondaryactivetransport
b)Facilitateddiffusion
c)Simplediffusion
d)Primaryactivetransport
CorrectAnswer-B
Answer:B.Facilitateddiffusion
Facilitateddiffusionisadiffusionoflargewatersolublemoleculeby
acarrierprotein.
Glucoseandaminoacidsaretransportedacrossthemembraneby
thismethod.

20.Insulinlikegrowthfactorissecretedby:
a)Liver
b)Pituitarygland
c)Pancreas
d)Adrenalglands
CorrectAnswer-A
Answer:A.Liver
Growthhormoneactsonthelivertoformsmallproteinscalled
"Somatomedins"
Somatomedinsincreasesbonegrowthinallaspect
Effectsaresimilartoinsulin,hencereferredtoas"Insulin-like
GrowthFactor"(IGF)
.
Fourtypesavailable-
MostimportantisSomatomedinC-Specificallyreferredtoas
"Insulin-GrowthFactor-1/IGF-1"
Bindstoacarrierproteininblood
Hence,alongerdurationofactionthangrowthhormone.
Half-life-about20Hrs(comparedtogrowthhormone-20mins)
BloodconcentrationofIGF-1followsthelevelsofgrowthhormone.
GrowtheffectsofGHaremostlyattributedtosomatomedin(rather
thanitsdirecteffectonbones&peripheraltissues)

21.WhatisProsopagnosia?
a)Impairmentofconsciousness
b)Beingunawareofone'sproblems
c)Difficultyinidentifyingknownfaces
d)Failuretoidentifyobjects
CorrectAnswer-C
Ans.C.Difficultyinidentifyingknownfaces.
Afeatureofanxietydisorder
Prosopagnosiaisdifficultyinidentifyingknownfaces
Otherfeaturesofanxietydisorderinclude,
Memoryimpairmentwithoutimpairmentofconsciousness
Beingunawareofone'sproblem(Agnosognosia)

22.Tyrosinosisiscausedduetodeficiencyof
whichenzyme?
a)Fumarylacetoacetatehydrolase.
b)p-hydroxyphenylpyruvatedehydrogenase.
c)Tyrosinetransaminase.
d)Tyrosineligase.
CorrectAnswer-A
Answer:A.Fumarylacetoacetatehydrolase
Severalmetabolicdisordersareassociatedwiththetyrosine
catabolicpathway.TheprobablemetabolicdefectintypeI
tyrosinemia(tyrosinosis)isatfumarylacetoacetatehydrolase.
Itischaracterizedbybuildupoftoomuchoftheaminoacidtyrosine
inthebloodandtissuesduetoaninabilitytometabolizeit
Thetherapyemploysadietlowintyrosineandphenylalanine.
Untreatedacuteandchronictyrosinosisleadstodeathfromliver
failure.

23.Lesch?Nyhansyndromeiscausedby
deficiencyofwhichenzyme?
a)OrotatePhosphoribosyltransferase
b)Uracilphosphoribosyltransferase
c)QuinolinatePhosphoribosyltransferase
d)Hypoxanthine-guaninephosphoribosyltransferase(HGPRT)
CorrectAnswer-D
Answer:D.Hypoxanthine-guaninephosphoribosyltransferase
(HGPRT)
Theconditioniscalledksch-Nyhanslmdrome,inwhichthereis
completedeficiencyofHGPRT.
HGPRTdeficiencycausesdecreasedutilization
ofPRPPinsalvagepathway.
ThisresultsinincreasedproductionofpurinenucleotidefromPRPP
viade-novopathway.
Thediseaseischaracterizedbyhyperuricemia,
goutyarthritis,urinarystones,neurologicalsymptoms.

24.Fishodorsyndromeiscausedby
deficiencyofwhichenzyme?
a)Fumarylacetoacetatehydrolase
b)Methanemonooxygenase
c)Monooxygenase3(FMO3)
d)D-aminoacidoxidase
CorrectAnswer-C
Answer:C.Monooxygenase3(FMO3)
Trimethylaminuria,orfishodorsyndrome(FOS),isacondition
characterizedbythepresenceoftrimethylamine(TMA)--atertiary
aminewhoseodorisdescribedasresemblingthatofrottingfish--in
theurine,sweat,andexpiredair.Thecauseofthesyndromeis
rootedinthedysfunctionalmetabolismofTMA,whichisnormally
oxidizedbyflavinmonooxygenase3(FMO3)intonon-odorous
trimethylamine-N-oxide(TMAO).
MostpatientswithFOSareeventuallydiagnosedwithprimary
trimethylaminuria,whichiscausedbyadeficiencyinFMO3thatis
inheritedinanautosomalrecessivefashion.
Thediagnosisismadeonthebasisoftheclinicalpresentationand
urinalysis.UrinecanbeanalyzedfortheconcentrationofbothTMA
andTMAO,andtheresultsmaybegivenasanoxidizingratiobased
ontheformula-TMAO/(TMAO+TMA)x100%.
Shortcoursesoforalneomycin,metronidazole,andamoxicillinhave
beenreportedtobeusefulinsomecases.

25.Galactosemiaisduetodeficiencyof
whichenzymes
a)Galactose-1-phosphateuridyltransferase
b)HGPRT
c)Galactokinase
d)Epimerase
CorrectAnswer-A
Answer:A.Galactose-1-phosphateuridyltransferase
Galactosaemia(Britishgalactosaemia)isararegeneticmetabolic
disorderthataffectsanindividual'sabilitytometabolizethesugar
galactoseproperly.Galactosemiafollowsanautosomalrecessive
modeofinheritancethatconfersadeficiencyinanenzyme
responsibleforadequategalactosedegradation.

26.Whichofthefollowingismostabundant
endproductoffattyacidsynthesis-
a)Oleicacid
b)Palmiticacid
c)Arachidonicacid
d)Glucose
CorrectAnswer-B
Answer:B.Palmiticacid
Fattyacidaresynthesizedbyextramitochondrialsystem.
Thissystemispresentinmanytissuesincludingliverkidneybrain
lungmammaryglandandadiposetissues.
AcetylCoAisimmediatesubstrate.
Theendproductsofthissynthesisareusuallythesaturatedfatty
acidspalmitateandstearatewiththelatterpredominating.

27.AboutDNApolymeraseIwhichoneis
correct?
a)Notrequiredinbacteria
b)RepairanydamagewithDNA
c)Involvedinokazakifragment
d)ParticipateinDNAreplication
CorrectAnswer-A
Answer:A.Notrequiredinbacteria
DNApolymeraseI
participatesintheDNAreplicationof
prokaryotes.FunctionofPolIismainlytorepairanydamagewith
DNA,butitalsoservestoconnectokazakifragmentsdeletingRNA
primersandreplacingthestrandwithDNA.

28.Whatdoeschaperonesassistin?
a)ProteinCleavage
b)ProteinFolding
c)ProteinDegradation
d)ProteinModification
CorrectAnswer-B
Answer:B.ProteinFolding
FoldingofProteinsinVivoIsPromotedbyChaperones

29.Fishyodouroccursduetodeficiencyof
thisvitaminfromdiet-
a)Biotin
b)Thiamine
c)Riboflavin
d)Vit.A
CorrectAnswer-C
Answer:C.Riboflavin
Vitamin
B2orriboflavindeficiencycanbringaboutafishyodorin
thebody.
*Fish-odorsyndrome,alsocalledtrimethylaminuria,isarare
metabolicdisordercausedbytheabsenceofenzynen-oxidase(A
Flavincontainingmonooxygenase,3(FMO3).Genefortrimethylamineoxidase
hasbeenmappedonchromosomeI(Iq23-q25).
*Trimethylamineisnormallyproducedintheintestinefromthe
breakdownofdietarycholineandtrimethylamineoxidebybacteria.
*Eggyolkandliverarethemainsourcesofcholine,andfishisthe
majorsourceoftrimethylamineoxide.
*Trimethylamineisabsorbedand
oxidizedintheliverbytrimethylamineoxidase(flavin-
containingmonooxygenases)totrimethylamineoxide,
whichisodorlessandexcretedintheurine.Deficiencyofthe
enzymeresultsinmassiveexcretionoftrimethylamineinurine.
*Consequently,tothesepatientssuchfoodsmaytaste
likerottenfishandimportafishyodortotheir
salvia,sweatandurine.
Treatment
*Restrictionoffish,eggs,liverandothersourcesofcholine

(suchasnutsandgrains)inthedietsignificantlyreducetheodor.
-Treatmentwithshortcourseoforal
metronidamle,neomycinorlactulosecausestemporaryreductionin
thebodyodor.
*RiboflavinsupplementcanbegiventoenhanceresidualFMO3
activity.

30.VMAisexcretedinurineinwhich
condition-
a)Alkaptonuria
b)Phenylketonuria
c)Pheochromocytoma
d)Diabeticketoacidosis
CorrectAnswer-C
Answer:C.Pheochromocytoma
VMAistheendproductofcatabolismofcatecholamines.
Inpheochromocytomaandneuroblastomathereisexcessive
synthesisofcatecholamineswhichcausesenhancedsynthesisof
VMAanditsexcretionintheurine.
VMAistheurinaryproductofbothepinephrineandnorepinephrine.
Itisagoodscreeningtestforpheochromocytoma,andisalsoused
todiagnoseandfollowupneuroblastomaandganglioneuroma.

31.InCystinuriaallofthefollowingamino-
acidsreabsorptiondefectispresent,
except

a)Lysine
b)Citrulline
c)Arginine
d)Ornithine
CorrectAnswer-B
Ans.is'B'i.e.,Citrulline
Typesofcystinuria
*Type-I:Itishomozygouswithafullyrecessiveform.Thepatient
excretesalargequantityofcystine,ornithine,lysine,and
arginine.GeneinvolvedisrBATonchromosome-2.
*Type-II&III:Theseareheterozygous
variantsofincompletelyrecessiveforms.Theyexcrete
cystine,ornithine,lysine,andargininemorethannormal
butlessthanthehomozygousstate(Type-I).Geneinvolvedis
SLC7A9onchromosome79.
Cystinuria
*BiochemicalDefect
:Anautosomalrecessivedisorderthatresults
intheformationofadefectiveaminoacidtransporterintherenal
tubuleandintestinalepithelialcells.
*Pathophysiology:Theaminoacidtransporterisresponsiblefor
transportingcystine,ornithine,lysine,andarginine.Defectivetubular
reabsorptionoftheseaminoacidsinthekidneysresultsinincreased
cystineintheurine,whichcanprecipitateandcausekidneystones.
*ClinicalManifestations:Cystinekidneystonespresentingwith

severe,intermittentflankpainandhematuria.
*Labfindings:Increasedurinaryexcretionofcystine,ornithine,
arginine,andlysineonurineaminoacidchromatography;hematuria
andcystinecrystals(hexagonal)onthecoolingofacidifiedurine
sediment.
*Imaging:RadiopaquekidneystonesonCTscan.Themost
specifictestisthecyanide?nitroprussidetest
*Treatment:Low-methioninediet;increasedfluidintake;
acetazolamidetoalkalinizetheurine.Ifthisfailsthenpatientsare
usuallystartedonchelatingtherapywithpenicillamine.

32.FibrinopeptideAandfibrinopeptideBare
acidicduetothepresenceofwhichamino
acidsinitsstructure-

a)Serineandthreonine
b)Glutamateandaspartate
c)Histidineandlysine
d)Glutamineandvaline
CorrectAnswer-B
Answer:B.Glutamateandaspartate
TheN-terminalAandBportionsoftheAandBchainsaretermed
fibrinopeptideA(FPA)andfibrinopeptideB(FPB),respectively.
Thesedomainsarehighlynegativelychargedasaresultofan
abundanceofaspartateandglutamateresidues.
Thenegativechargescontributetothesolubilityoffibrinogenin
plasmaandimportantlyalsoservetopreventaggregationby
causingelectrostaticrepulsionbetweenfibrinogenmolecules.

33.HIAAinurinepresentin?
a)Alkaptonuria
b)Albinism
c)Carcinoid
d)Phenylketonuria
CorrectAnswer-C
Answer:C.Carcinoid
Carcinoidsyndromedevelopsinsomepeoplewithcarcinoidtumors
andischaracterizedbycutaneousflushing,abdominalcramps,and
diarrhea.
Carcinoidtumouroccurthroughoutthegastrointestinaltract,most
commonlyintheappendix,ileumandrectumindecreasingorderof
frequency.
Right-sidedvalvularheartdiseasemaydevelopafterseveralyears.
Thesyndromeresultsfromvasoactivesubstances(including
serotonin,bradykinin,histamine,prostaglandins,polypeptide
hormones)secretedbythetumor,whichistypicallyametastatic
intestinalcarcinoid.
Diagnosisisclinicalandbydemonstratingincreasedurinary5-
hydroxyindoleaceticacid(HIAA).
Tumorlocalizationmayrequirearadionuclidescanorlaparotomy.
Treatmentofsymptomsiswithsomatostatinoroctreotide,but
surgicalremovalisperformedwherepossible;chemotherapymay
beusedformalignanttumors.

34.Warthinfinkeldeycellsareseenin
a)Measles
b)Rubella
c)Rabies
d)Typhoid
CorrectAnswer-A
Answer:A.Measles
MultinucleatedcellslikeWarthinFinkeldeyareseeninMeasles
Measlesvirusinfectsbyinvasionofrespiratoryepithelium.
Localmultiplicationleadstoviremia(day2-3),thenspreadtoRE
system.
TwotypesofMultinucleatedgiantcellsinbothepidermis&oral
epitheliumby7-11days.
WarthinFinkeldeycellsofreticuloendothelialsystem
Epithelialgiantcellsofrespiratory&otherepithelia.
Warthin?Finkeldeycell:
Typeofgiantmultinucleatecellfoundinhyperplasticlymphnodes
earlyinthecourseofmeasles
Underthelightmicroscope,thesecellsconsistofalarge,grape-like
clusterofnuclei.
AlsowithHIV-infectedindividualsandKimuradisease.
Rarelyinneoplastic(e.g.lymphoma)&non-neoplasticlymphnode
disorders.
Unknownorigin;Reportsofstainingwithmarkerssimilartofollicular
dendriticcells,includingCD21.

35.CD59markerofwhichdisease
a)PNH
b)PTEN
c)BRR
d)Cowdensyndrome
CorrectAnswer-A
Answer:A.Paroxysmalnocturnalhemoglobinuria(PNH)
Paroxysmalnocturnalhemoglobinuria(PNH)
isadisease,dueto
acquiredmutationsin"Phosphatidylinositol
GlycanComplementationGroupA"gene(PIGA)
..
Associatedwithdeficiencyofglycosylphosphatidylinositol
(GPI)
anchorproteinsalongwithabsenceofexternalsurface
membraneproteinsattachingtoit.
CD55(DAF)andCD59(MIRL)aretwosuchcomplementdefence
proteins
CD59deficiency:
CommonfindinginRBCs&WBCsofpatientswithchronic
hemolysissufferingfromPNH
Diagnosis:
Thedefinitediagnosisbasedondemonstrationofasubstantial
proportionofpatient'sRBChavingincreasedsusceptibilityto
complement(C),duetothedeficiencyontheirsurfaceofproteins
(particularlyCD59&CD55)

36.Opsoninis
a)C3a
b)C3b
c)C5a
d)C6
CorrectAnswer-B
Answer:B.C3b
Theprocessofcoatingaforeignparticletargeting&preparingitfor
phagocytosisprocessis"Opsonization".Substancesinvolvedare
opsonins.
MainopsoninsfromcomplementsystemisC3
Examplesofopsoninsinclude:
Antibodies:
IgGandIgA
Componentsofthecomplementsystem:
C3b,C4b,andiC3b
MannoseBindingLectin(MBL):
InitiatestheformationofC3b
MembraneAttackComplex(MAC)
IncludesC5b,C6,C7,C8&polymericC9
Opsonization&complementproteins:
MainlyC3b,iC3b&C4b
C3:
Mostabundantproteinofallcomplementaryproteins,
CleavesintoC3aandC3b
C3a-
Bindsandactivatesmastcells&basophils,releasehistamine.
C3b-

Mostcriticalcomponentinbothclassical&alternativepathway
C3battachestobacterialsurfacesforopsonizationbyphagocytes

37.Bernard?Souliersyndromedueto
deficiencyof
a)Gp2b/3a
b)Gp1b
c)vWf
d)TNF
CorrectAnswer-B
Answer:B.Gp1b
Bernard
?Souliersyndrome(BSS)/HemorrhagicParous
ThrombocyticDystrophy
Rareautosomalrecessivecoagulopathy
Causesadeficiencyofglycoprotein1b(Gp1b),receptorforvon
Willebrandfactor.


38.Cowdensyndrome
a)P53
b)PTEN
c)Rb
d)Ras
CorrectAnswer-B
Answer:B.PTEN
"PhosphataseandTensin"homolog(PTEN)
-proteininhumans
encodedbythePTENgene.Genemutationspromotesdevelopment
ofcancers.
Cowden'sdisease/MultipleHamartomaSyndrome-
PartofPTENhamartomatumorsyndrome
Anautosomaldominantsyndrome
Trichilemmomas-Numeroustumorsofhairfolliclesinface
MultiplehamartomatouspolypsinGItract.
Veryhighriskofbreast&thyroidcarcinoma
Treatment:
B/Lmastectomiesrecommended
Contraindicatedaremammography&otherradiationexposureof
breasttissue

39.Chromosomeinvolvedinmyotonic
dystrophyis
a)Chromosome19
b)Chromosome20
c)Chromosome21
d)Chromosome22
CorrectAnswer-A
Answer:A-Chromosome19
Myotonicdystrophyistransmittedbymutationinan'unstable
trinucleotiderepeatsequence'ingene19q133.
Features:
Anautosomaldominantdisorder
Mostcommonadultmusculardystrophy
Characteristicsfeature:
Myopathyisdistal
(incontrasttoothermyopathies-mostly
proximal).
MuscleatrophyselectivelyinvolvestypeIfibresonly
Appearsby5years,causesaslowrelaxationofhand
grip
followingaforcedvoluntaryclosure.

40.TRALIoccurswithinhowmanyhoursof
transfusion?
a)48Hrs
b)72Hrs
c)6Hrs
d)12Hrs
CorrectAnswer-C
Answer:C-6Hrs
Transfusion-RelatedAcuteLungInjury(TRALI)
-Syndrome
characterizedbyacuterespiratorydistressfollowing
transfusion.
Symptoms:
Typicallydevelopduring,orwithin6hoursoftransfusion.
Rapidonsetofdyspnea&tachypnea
.
Associatedfever,cyanosis,&hypotension.
Clinicalexamination:
Revealsrespiratorydistress.
Pulmonarycrackles
maybepresentwithnosignsofCHFor
volumeoverload.
CXR-EvidenceofB/LpulmonaryedemaunrelatedtoCHF(non-
cardiogenicpulmonaryedema),
Bilateralpatchyinfiltratesrapidlyprogressingtocomplete"white
out"
indistinguishablefromAcuteRespiratoryDistressSyndrome
(ARDS).

41.Kidneyrespondstoshockby
a)Decreasesrenalbloodflow
b)Increasesafferentarterioleresistance
c)GFRremainsunaltered
d)Perfusionofkidneyincreases
CorrectAnswer-B
Answer:B-Increasesafferentarterioleresistance
Kidneyutilizesthefollowingmechanismsasaresponseto
shock:
Releaseofaldosterone
fromhypoxickidney
ReleaseofADHduetodecreasedeffectivecirculatingblood
volume.
ReducedGFRduetoarterioleconstriction
Tissuefluidshiftintoplasmaduetoloweredhydrostaticpressure
(Hypotension)

42.Whichofthefollowingisepithelialtumor
ofstomach?
a)Carcinoid
b)Lymphoma
c)GIST
d)Gastricadenocarcinoma
CorrectAnswer-D
Answer:D-Gastricadenocarcinoma
Malignantepithelialtumororiginatingfromglandularepithelium
ofgastricmucosa.
Aggressivelyinvadethegastricwall.
Laurenclassification:
Twotypesofgastricadenocarcinomaarepresent.
Intestinaltype
Diffusetype
Intestinaltype-Irregulartubularstructures
Diffusetype-Mucinous&colloidal"Leather-bottlestomach"

43.IdentifyanXlinkeddisorder?
a)Colorblindness
b)Thalassemia
c)Azoospermia
d)RetinitisPigmentosa
CorrectAnswer-A
Answer:A-Colorblindness
Hereditarycolorblindness/Achromatopsia
Theabilitytoappreciateoneormoreprimarycolorisdefective
(anomalous)orabsent(anopia)
DuetomutationsinXchromosome
Red&greenpigmentconescodedbyXchromosome;Bluecoded
onchromosome7
Morecommoninmalesthanfemales
Acquired-(Opticnerve/maculardamage)
Ishiharachart-
Testred/greencolorblindness-Farnsworth100huetest
Others:
Azoospermia&RetinitisPigmentosa-Y-chromosomelinkeddisorder
Thalassemia-Inherited(Autosomalrecessivepattern)blood
disorderscharacterizedbyabnormalhemoglobinproduction.Genes
inChromosome11and16involved.

44.HAndLvarietyseenin
a)Mixedcellularityhodgkin
b)Lymphocytedepleted
c)Lymphocytepredominance
d)Nodularsclerosis
CorrectAnswer-C
Answer:C-Lymphocytepredominance
Hodgkinlymphoma(HL)-common"MalignantLymphomas"
2entities:
ClassicalHL(cHL)
NodularLymphocyte?predominantHL(NLPHL).
Cells:
ClassicalHL-HodgkinandReed/Sternberg(HRS)cells
NodularLymphocytepredominantHL-Lymphocytic&Histiocytic
(L&H)cells

45.Stellategranulomaseenin
a)Sarcoidosis
b)Catscratchdisease
c)Cryptococcosis
d)Histoplasmosis
CorrectAnswer-B
Answer:B-Catscratchdisease
BacterialinfectioncausesbyBartonellahenselae
Acquiredinfectedcat/kittenscratch
Histology:
Characterizedbygranulomatousinflammationoflymphnodes.
Skinlesiondemonstratesacircumscribedfocusofnecrosis
Regionallymphnodesdemonstratefollicularhyperplasiawith
centralstellatenecrosis
withneutrophils,surroundedbypalisading
histiocytes(suppurativegranulomas)&sinusespackedwith
monocytoidBcells,usuallywithoutperifollicularandintrafollicular
epithelioidcells.

46.Whichbestexplains"Flippingeffect"?
a)LDH1>LDH2
b)LDH2>LDH1
c)LDH2>LDH3
d)LDH3>LDH2
CorrectAnswer-A
Answer:A-LDH1>LDH2
Lactatedehydrogenase,
tetramericenzymewith4subunits,
4Subunitswith2isoforms-Hisoform(Heart)&Misoform
(Muscle)
Heart&RBCs-LDH-1(4H);
Reticuloendothelialsystem-LDH-2(3H1M)
Lungs-LDH-3(2H2M)
Kidneys,placenta,&pancreas-LDH-4(1H3M)
Liver&striatedmuscle-LDH-5(4M)
Uses:
LDHlevelsaremoreinRBC
HelpfulinassessmentofHemolysis/Tissuebreakdown
Flippingeffect:
UsuallyLDH2inpredominantinserum&LDH1ispredominantin
heart
HigherlevelsofLDH1thanLDH2(Flippedpattern)issuggestiveof
myocardialinfarction
DamagedcardiactissuesreleaseLDH1intobloodstream.

47.Nudemiceisnotresistanttoxenograft
duetoabsenceof
a)Bcell
b)Tcell
c)Bothbandtcell
d)None
CorrectAnswer-B
Answer:B-Tcell
Nudemicelacks"Thymus"&cannotgeneratematureT
lymphocytes.
AbsenceofT-lymphocytesmakesitunabletomountadaptive
immuneresponses
requiringCD4,helperTcells,CD8and
cytotoxicTcells.
Adaptiveimmuneresponsesthatremainunresponsivetonude
miceinclude:
Antibodyformation(CD4+helperTcells)
Cell-mediatedimmuneresponses(CD4+and/orCD8+Tcells)
Delayed-typehypersensitivityresponses(CD4+Tcells)
Killingofvirus-infectedormalignantcells(CD8+cytotoxicTcells)
Graftrejection(bothCD4+&CD8+Tcells)
Uses:
Laboratorystudyanimal-Insightsintoimmunesystem,leukemia,
solidtumors,AIDS&otherimmunedeficiencydiseases.
AbsenceoffunctioningTcellspreventsthemrejectingtheallografts
&Xenografts.

48.Anaplasiais
a)Changingonetypeofepitheliumtoanother
b)Nuclearchromatin
c)Lackofdifferentiation
d)Morphologicalchanges
CorrectAnswer-C
Answer:C-Lackofdifferentiation
Referstoalackofdifferentiationinneoplasticcells.
Well-differentiatedtumorsresembletheirtissueoforigin
Poorly-differentiatedorundifferentiated(anaplastic)tumorcells
appearprimitiveandlackspecializationalonganyparticularcellline.

49.Whichlevelofprolactindefinitelysuggest
prolactinoma?
a)300ng/ml
b)150ng/ml
c)200ng/ml
d)100ng/ml
CorrectAnswer-C
Answer:C-200ng/ml
Prolactinomasarethemostcommontypeofhyperfunctioning
pituitaryadenoma.
Benigntumorsofpituitaryglandproducingprolactin.
Hyperprolactinemiacausesamenorrhea,galactorrhea,lossoflibido,
andinfertility.
Becausemanymanifestationsofhyperprolactinemia(e.g.,
amenorrhea)aremoreobviousinpremenopausalwomenthanin
menorpostmenopausalwomen,prolactinomasusuallyare
diagnosedatanearlierstageinwomenofreproductiveagethanin
otherpersonssoaffected.
Higherbloodprolactinconcentrationsareseen.
mildelevationsofserumprolactin(lessthan200g/L)inapatient
withapituitaryadenomadonotnecessarilyindicateaprolactin-
secretingneoplasm.

50.Laxativeabusecauseswhichofthe
followingrenalstones?
a)Uricacid
b)Ammoniumurate
c)Struvite
d)Caoxalate
CorrectAnswer-B
Answer:B-Ammoniumurate
Basedonthechemicalnaturetwotypesofkidneystones
:
Calciumoxalate(majority).
OthersincludeUricacid,Struvite(Infectedstones),andCystine
stones(rarehereditarymetabolicdisorder
Characteristicstoneformationinlaxativeabuse:
Laxativeabuseactsafactorinkidneystoneformation.
Laxativeabusecausespotassiumloss
Asacompensationmechanismkidneyproduceslargeamountof
ammonium.
Resultinginformationofuncommonstonetype-ammoniumacid
urate.

51.Whichofthefollowingcanresultin
dactylitis
a)Hemophilia
b)Vonwillebranddisease1
c)Measles
d)SickleCellAnemia
CorrectAnswer-D
Answer:D-SickleCellAnemia
Dactylitis(Hand-FootSyndrome)isseeninsicklecellanemia
Severepainaffectingthebonesofhands,feet,orboth.
Often1stsymptomofsicklecellanemiainbabies.

52.Whichchromosomeisresponsibleforthe
productionofMIF?
a)Chromosome16
b)Chromosome22
c)XChromosome
d)Ychromosome
CorrectAnswer-D
Answer:D-YChromosome
Anti-MullerianHormone(AMH)/MullerianInhibitingFactor(MIF);
Mullerian-inhibitingHormone(MIH)/Mullerian-inhibitingSubstance
(MIS).
AMH-DownstreamgenesregulatedbySRYpathway
SRY-Genein"Sexdeterminingregion"-shortarmofY
chromosome-Testisdeterminingfactor.
SecretedbySertolicellsofthetestes.
TheproductionofAMHiscontrolledbytwoautosomalgeneloci.
Hormonecode
Receptorcode.
Glycoproteinhormone
Relatedtoinhibin&activin
Memberofthetransforminggrowthfactor-(TGF-)
Keyrolesareingrowthdifferentiationandfolliculogenesis.

53.SiteofactionofamphotericinBis:
a)Ribosomes
b)Cellwall
c)Plasmamembrane
d)Protein
CorrectAnswer-B
Answer:-B-Cellwall
PolyenedrugcompoundslikeAmphotericinBactsoncell
membrane-
AmphotericinB,antifungalagent.
Chemicallyapolyenecompound
ObtainedfromStreptomycesnodosus.
Polyeneshaveahighaffinityfor"ergosterol"presentinthefungal
cellmembrane.
Bindsandgetsinsertedintothecellmembraneforming"Micropore".
Markedincreaseinpermeabilityofcellmembrane.
PolyenedrugcompoundslikeAmphotericinBactsoncell
membrane
AmphotericinB,antifungalagent.
Chemicallyapolyenecompound
ObtainedfromStreptomycesnodosus.
Polyeneshaveahighaffinityfor"ergosterol"presentinthefungal
cellmembrane.
Bindsandgetsinsertedintothecellmembraneforming"Micropore".
Markedincreaseinpermeabilityofcellmembrane.

54.Whichantiretroviraldrugalsohasanti
hepatitisactivity?
a)Abacavir
b)Tenofovir
c)Nevirapine
d)Emtricitabine
CorrectAnswer-D
Answer:D-Emtricitabine
Emtricitabine,NRTIdrugwithbothantiretroviral&anti-hepatitis
properties
Nucleosidereversetranscriptaseinhibitorforthepreventionand
treatmentofHIVinfectioninadultsandchildren.
Alsousedincombinationwithtenofovir

55.Drugofchoiceforresistantrheumatic
chorea?
a)Valproate
b)Haloperidol
c)Diazepam
d)Probenecid
CorrectAnswer-A
Answer:-A-Valproate
Valproate,
Sulpiride,&diazepamareusedforsymptomatic
treatment.
AcuteRheumaticFever:
AbnormalimmuneresponsetogroupAstreptococcalinfection
Commonlyaffectingthejoints,heart,brain,andskin.
Symptoms:Arthritis,relatedtocarditis&chorea.
Sydenham'schorea/Choreaminor
Characterizedbyrapid,uncoordinatedjerkingmovementsprimarily
affectingtheface,handsandfeet.
Signs&symptomsofchoreausuallydonotrespondwellto
treatmentwithantirheumaticagents
Symptomatictreatmentincludeanticonvulsants(eg,valproate,
carbamazepine
)andneuroleptics(eg,pimozide,haloperidol,
risperidone,olanzapine

56.AtpKa=pH-
a)Conc.ofdrugis50%ionicand50%non-ionic
b)Absorptionofdrugis50%ionicand50%ionic
c)Concofdrugis75%ionicand25%non-ionic
d)Concofdrugis25%ionicand75%non-ionic
CorrectAnswer-A
Answer:A-Concentrationofdrugis50%ionic&50%Non-
ionic
NumericallyequalpKa&pHrepresents50%drugionization
pKaisnegativelogarithmofacidicdissociationofweakelectrolyte.
Onequalconcentrationsofionized&unionizeddrugs,log1is
zero.
Thus,whenpKaisnumericallyequaltopH
(pKa=pH)50%drugisionized.

57.Physiologicaldoseofhydrocortisone
(mg/kg/day)is-
a)5mg/kg/day
b)10mg/kg/day
c)15mg/kg/day
d)20mg/kg/day
CorrectAnswer-B
Answer:-B-10mg/kg/day
Thenormalrateofsecretionoftwoprinciplecorticoids
Hydrocortisone-10mg/kg/day(nearlyhalfinmorninghours)
Aldosterone-0.125mg/daily

58.Whatismechanismofactionofcolchicine
inacutegout?
a)Inhibitionofpurinemetabolism
b)Inhibitionofuricacidconversion
c)Migrationofleukocytes
d)Leukocytes,lymphocytesinhibition&microtubularinhibitor
CorrectAnswer-D
Answer:D-Leukocytes,Lymphocytesmigrationinhibition&
microtubularinhibitor.
Colchicineactsbyinhibitingthegranulocytemigrationintothe
inflamedjoint.
AnalkaloidfromColchiumautumnale
Specificallysuppressesgoutyinflammation.
Doesn'tinhibitthesynthesisorpromotetheexcretionofuricacid.
Mechanismofaction:
Colchicineactsby,inhibitsthereleaseofglycoprotein
Bindstofibrillarproteintubulesinhibitinggranulocyte
migrationintotheinflamedjoint
.
Anacuteattackofgoutstartsbyprecipitationofuratecrystalsin
synovialfluid.
Inflammatoryresponsestartswithgranulocytemigrationintojoint
Phagocytosinguratecrystalsreleasingglycoprotein
Glycoproteinincreaseslacticacidproductionandreleasing
lysosomalenzymescausingmorejointdestruction.

59.Basiliximabisan-
a)IL-1receptorantagonist
b)Anti-CD3antibody
c)IL-2receptorantagonist
d)TNFinhibitor
CorrectAnswer-C
Answer:-C-IL-2receptorantagonist
BasiliximabexhibitshighaffinitytowardsIL-2receptor,
inhibitingit.
Anti-CD-25antibody
HighaffinityforIL-2receptor
Shortplasmahalflife-1week
Usefulinpreventingtransplantrejectionreactions.
Adverseeffects-Anaphylacticreactions&opportunisticinfections.

60.Pirenzapineisused-
a)Gastriculcer
b)Glaucoma
c)Hypertension
d)Congestivecardiacfailure
CorrectAnswer-A
Answer:A-GastricUlcer
Pirenzepine,aselectiveM1anticholinergicdruginhibiting
gastricacidsecretion.
Lowtherapeuticdoserange.
Usedfortreatinggastriculcer.

61.Whichofthefollowingantipsychotichave
increasedprolactinsecretion-
a)Olanzapine
b)Ziprasidone
c)Clozapine
d)Risperidone
CorrectAnswer-D
Answer:-D-Risperidone
Significantriseinprolactinlevelsduringrisperidonetherapyis
observed.
Risperidone-
Antipsychoticdrugwithcombined5-HT2aand
dopamineD2antagonistactivity
Highaffinitytoalpha1,alpha2andH1receptors
MorepotentD2blockerthanclozapine
Amelioratessymptomsofschizophrenia
Prolactinlevelsriseduringrisperidonetherapy,butless
epileptogenicthanclozapine.
Producesextrapyramidalsideeffectsarelessonlyatlowerdoses
(<6mg/day).
Blockadeofthesecontributetoefficacyandsideeffectslikepostural
hypotension.
Frequentlycausesagitation.

62.Whichofthefollowingisglucocorticoid
synthesisinhibitor?
a)Mifepristone
b)Flutamide
c)Finasteride
d)Metyrapone
CorrectAnswer-D
Answer:D-Metyrapone
Inhibits1113-Hydroxylaseinadrenalcortex
Preventsthesynthesisofhydrocortisone


63.Whichofthefollowingstatementsis
incorrectw.r.tPrasugrel?
a)Notaprodrug
b)P2Ypurinergicreceptorblocker
c)Hasastrongantiplateletactivity
d)CausesintracranialhemorrhageinTIApatients.
CorrectAnswer-A
Answer:A-Notaprodrug
Prasugrelisaprodrug,similartoClopidogrel
Thienopyridinedrugclass
IrreversibleantagonistofP2Y12ADPreceptors
Rapidlyabsorbed,completelyactivated&exertsmore
consistentplateletinhibition.
Stronganti-plateletactivity
Bleedingcomplicationsaremoreseriousandfrequent.
Contraindicatedinpatientswithhistoryofischemicstrokes
andTIA's


64.Q-Telongationisseeninwhichdrug?
a)Quinidine
b)Amiodarone
c)MagnesiumSulfate
d)Lignocaine
CorrectAnswer-A
Answer:A-Quinidine
SpecificpatternofQ-Tprolongationisreferredtoas"Torsades
depointes"

DrugscausingTorsadesdePointes
Quinidine(mostcommon)
Sotalol
Procainamide
Disopyramide
Phenothiazines
Tricyclicantidepressants

65.Sacubitrilis,
a)ACEinhibitor
b)Neutralendopeptidaseinhibitor
c)Calciumchannelinhibitor
d)Betaadrenergicblocker
CorrectAnswer-B
Answer:B-Neuro-endopeptidaseinhibitor
Sacubitril,aprodruginhibitingneuro-endopeptidaseenzyme
ActivatedtoSacubitril,
Inhibitingenzymeneprilysin(Neutralendopeptidases)
Combinationdrugusedinheartfailurepatients
UsuallycombinedwithACEinhibitorslikevalsartaninratioof1:1

66.Niacintherapyiscontraindicatedin
diabetesbecause-
a)Increasesthebloodsugarlevels
b)Causesscleroderma
c)Difficulttogiveinjection
d)Increasesthemetabolismoforalhypoglycemicdrugs
CorrectAnswer-A
Answer:A-increasesthebloodsugarlevels
Niacintherapyhaspotentialeffectsonbloodsugarlevels.
Increasesthebloodglucoselevelsindiabetespatients

67.Endothelinactsthroughwhichreceptors?
a)cAMP
b)cGMP
c)Na+receptors
d)Calciumreceptos
CorrectAnswer-A
Answer:A-cGMP
Endothelin-1(ET-1)isapotentendogenousvasoconstrictor,mainly
secretedbyendothelialcells.

68.Whichisthecentrallyactingalpha2
agonistmusclerelaxant-
a)Diazepam
b)Bromocriptine
c)Tizanidine
d)Methocarbamol
CorrectAnswer-C
Answer:-C-Tizanidine
Centralalpha2adrenergicagonist
Mechanismofaction:
Inhibitsthereleaseofexcitatoryaminoacidsinspinalinterneurons
Facilitatestheinhibitorytransmitterglycine
Inhibitspostsynapticreflexes
Reducingmuscletone,frequencyofmusclespasms.without
reducingthestrengthofmuscle.
Indications:
Spasticityinneurologicaldisorders
Painfulmusclespasmofspinalorigin.
Contraindications:
Patientsonantihypertensivesspeciallyclonidine.

69.Apixabanis-
a)Antithrombininhibitor
b)DirectXainhibitor
c)Plateletactivator
d)ClottingFactorXII
CorrectAnswer-B
Answer:B-DirectXainhibitor
DirectXainhibitor
Anticoagulantfortreatment&prophylaxisofvenousthromboembolic
events
DVT&PE

70.Anaerobesareresistantintrinsically
against-
a)Betalactamantibiotics
b)Aminoglycosides
c)Azithromycin
d)Metronidazole
CorrectAnswer-B
Answer:B-Aminoglycosides
Anaerobicbacteriaparticularlyareresistantto
aminoglycosidesduetolackofoxidativemechanismtodrive
druguptakingprocess.
Intrinsicresistance/Insensitivity
:
Innateabilityofbacteriatoresistactivityofparticularantimicrobial
agent
Inherentstructuralorfunctionalcharacteristicsallowsfortolerance
ofaparticulardrugorantimicrobialclass.i.e.,Susceptibilitytothat
particulardrugisreduced.

71.Whichisnotbacteriostaticantibiotic-
a)Clindamycin
b)Vancomycin
c)Tetracycline
d)Cephalosporins
CorrectAnswer-B
Answer:B-Vancomycin
Bacteriostaticantibiotics
Limitbacterialgrowthbyinterferingwithbacterialprotein
production,DNAreplication,
orotheraspectsofbacterialcellular
metabolism.
Tetracyclines,sulfonamides,clindamycin,
spectinomycin,trimethoprim,chloramphenicol,macrolidesand
lincosamides.
Bactericidalantibiotics
Inhibitcellwallsynthesis
(Irreversiblekilling)
Aminoglycosides.cephalosporins.fluoroquinolones.metronidazole.
penicillin.vancomycin

72.Whichofthefollowingcausesmelanosis
coli?
a)Senna
b)Sorbitol
c)MagnesiumSulphate
d)Bisacodyl
CorrectAnswer-A
Answer:A-Senna
Laxativeabusewithdrugslikesennacausemelanosiscoli
Anthranoidlaxatives(aloe,cascarasagrada,andsenna
)are
derivedfromnaturallyoccurringplants
Consideredtobestimulantlaxatives.
Safershorttermuse.
Longtermabusecancausemelanosiscoli&possibly
increasesriskofcoloniccancer.


73.Whichamongthefollowingwillthechoice
ofantibioticforabedriddenpatientwith
catheter-relatedUTIandpneumonia.

a)Amoxicillin
b)BetaLactamantibioticswithbetalactamase
c)3rdgencephalosporins
d)2ndgencephalosporins
CorrectAnswer-B
Answer:B-BetaLactamAntibiotics
Containsbetalactamaseenzymeforpotentactionagainst
organismscausingUTI


74.Mycoplasmaisresistantto-
a)Ceftriaxone
b)Cephalosporins
c)Aminoglycosides
d)Fluoroquinolones
CorrectAnswer-A
Answer:A-Ceftriaxone
MycoplasmashowsresistancetowardsCeftriaxone,athird
generationcephalosporin(betalactamantibiotic)
Lackofcellwallinmycoplasmasmakesthemintrinsically
resistantto-lactams
&toallantimicrobialsthattargetcellwall.
Mycoplasmapneumoniae:
Mycoplasmapneumoniaeisapathogenicmycoplasmaresponsible
forrespiratorytractinfectionsinhumans.
First-linetreatment:macrolides&relatedantibiotics,tetracyclines
andfluoroquinolonesispreferred.

75.Tadalafilshouldnotbegivenwith:
a)Vasodilator
b)Antibiotics
c)Vasoconstrictors
d)Valproate
CorrectAnswer-A
Answer:A-Vasodilators
Combinationwithvasodilatorsresultsinsuddenchangesof
bloodpressurevalues
Tadalafilrelaxesmusclesofthebloodvesselsandincreases
bloodflowtoparticularareasofthebody.
Usedtotreaterectiledysfunction(Impotence),andsymptomsof
benignprostatichypertrophy(Enlargedprostate)
Eg:Takingtadalafilwithvasodilatordruglikenitratecancause
sudden&seriousdecreaseinbloodpressure
.

76.Estimatevolumeofringerlactateinfirst8
hrsfor40%burnsin50kgmalewith2?
burns?

a)8lt
b)4lt
c)2lt
d)6lt
CorrectAnswer-B
Answer:B.4lt
ParklandformulamostcommonlyusedIVfluid-LactatedRinger's
Solution
Fluidcalculation
4xweightinkgx%TBSAburn
Give1/2ofthatvolumeinthefirst8hours
Giveother1/2innext16hours
Warning:Despitetheformulasuggestingcuttingthefluidrateinhalf
at8hours,thefluidrateshouldbegraduallyreducedthroughoutthe
resuscitationtomaintainthetargetedurineoutput,i.e.,donotfollow
thesecondpartoftheformulathatsaystoreducetherateat8
hours,adjusttheratebasedontheurineoutput.
Exampleoffluidcalculation
50-kgmanwith40%TBSAburn
Parklandformula:
4x50x40=8,000ml
Give1/2infirst8hours=4,000mlinfirst8hours
Adjustfluidratetomaintainurineoutputof50ml/hr

77.Posthumouschildis:
a)Childdeliveredafterdeathofbiologicalmother
b)Childdeliveredafterdeathofbiologicalfather
c)Bornafterdeathofparents
d)hasbeenabandonedbyparents
CorrectAnswer-B
Answer:B.Childdeliveredafterdeathofbiologicalfather
Achildbornafterdeathofhis/herbiologicalfather

78.

M'naghtenrulecomesunderwhichsectionof
IPC?

a)Crpc84
b)Cpc48
c)Ipc84
d)IPC48
CorrectAnswer-C
Answer:C.Ipc84
M'naghtenrule(legaltestorrightorwrongtest):
Itstatesthatanaccusedpersonisnotlegallyresponsible,ifitis
clearlyprovedthatatthetimeofcommittingthecrime,personwas
sufferingfromsuchadefectofreasonfromabnormalityofmindthat
hedidn'tknowthenatureandqualityofacthewasdoingorthat
whathewasdoingwaswrongi.e.apersonisnotresponsibleifheis
notofsoundmind.
Itisacceptedinindiaaslawofcriminalresponsibilityandis
embodiedinsection841PCas-"nothingisanoffencewhichis
donebyaperson,whoatthetimeofdoingit,byreasonof
unsoundnessofmindisincapableofknowingthenatureofact,or
thatheisdoingwhatiseitherwrongorcontrarytolaw".

79.Bluishdiscolorationofgastricmucosa
seeninwhichpoisoning?
a)Mercury
b)Cadmium
c)Amytalsodium
d)Arsenic
CorrectAnswer-C
Answer:C.Amytalsodium
S.
Poison
Color
No.
1.
Coppersulfate,amytalcapsule
Blue
2.
Ferroussulfate
Green
3.
Sulphurichydrochloric/aceticacid
Black/charred
4.
Nitricacid
Yellow
5.
Carbolicacid
Buff/white
6.
Arsenic
Whiteparticles
7.
Mercury
Slate
8.
Cresols
Brown

80.Musclepain,nephropathycausedby
whichmetalpoisoning
a)Arsenic
b)Cadmium
c)Mercury
d)Lead
CorrectAnswer-A
Answer:A.Arsenic
Nephropathyiscausedbymostofheavymetals.Musclepainis
associatedwitharsenic.

81.Whichisthefirstorgantoputrefy:
a)Brian
b)Heart
c)Prostate
d)Kidney
CorrectAnswer-A
Answer:A.Brian
Theorderofputrefactionis-earliesttolastlarynx,trachea
Stomach,intestineliver,spleenBrain,LungsHeart,Kidney
Bladder,Uterus/ProstrateSkin,muscles,tendonlastly,
bones.)

82.Locardisfamousfor:
a)Theoryofexchange
b)Fingerprintstudy
c)Formulaforestimationofstature
d)Systemofpersonalidentificationusingthebodymeasurement
CorrectAnswer-A
Answer:A.Theoryofexchange
Edmundlocardisfamousfortheoryofexchange.

83.Whendoesbasiocciputfuseswith
basisphenoid?
a)18to22
b)22to25
c)14-16
d)12-14
CorrectAnswer-A
Answer:A.18to22
Thebasioccipitalfuseswiththebasisphenoidatabout18to21
years.

84.Whatisthesmellofmummifiedbody?
a)Odourless
b)Putrid
c)Pungent
d)Offensive
CorrectAnswer-A
Answer:A.Odourless
Amummywillsmellodourless,thisisbecausetheinternalorgans
areremoved(whicharethebiggestfactorsinthedecayprocess)
andreplacedwithnatron(whichdriesthemout,preventing'proper'
decay).Therestofthebodyisalsodriedusingnatron.

85.Patientpresentedwithproximaltubule
proteinuria.Whichmetalislikelytobe
associatedwithit?

a)Cadmium
b)Mercury
c)Gold
d)Lead
CorrectAnswer-A
Answer:A.Cadmium
Earlykidneydamageandproteinuriaseeninpeople,occupationally
orenvironmentallyexposedtocadmium.

86.Whichofthefollowingconstitutional
articleisnotrelatedtochildren:
a)23
b)21-A
c)42
d)24
CorrectAnswer-C
Answer:C.42
ConstitutionalGuaranteesthataremeantspecificallyfor
childreninclude:
Righttofreeandcompulsoryelementaryeducationforallchildrenin
the6-14yearagegroup(Article21A)
Righttobeprotectedfromanyhazardousemploymenttilltheageof
14years(Article24)
Righttobeprotectedfrombeingabusedandforcedbyeconomic
necessitytoenteroccupationsunsuitedtotheirageorstrength
(Article39(e))
Righttoequalopportunitiesandfacilitiestodevelopinahealthy
mannerandinconditionsoffreedomanddignityandguaranteed
protectionofchildhoodandyouthagainstexploitationandagainst
moralandmaterialabandonment(Article39(f))
Righttoearlychildhoodcareandeducationtoallchildrenuntilthey
completetheageofsixyears(Article45)
Besides,ChildrenalsohaverightsasequalcitizensofIndia,
justasanyotheradultmaleorfemale:
Righttoequality(Article14)
Rightagainstdiscrimination(Article15)
Righttopersonallibertyanddueprocessoflaw(Article21)

Righttobeingprotectedfrombeingtraffickedandforcedinto
bondedlabour(Article23)
Rightofminoritiesforprotectionoftheirinterests(Article29)
Rightofweakersectionsofthepeopletobeprotectedfromsocial
injusticeandallformsofexploitation(Article46)
Righttonutritionandstandardoflivingandimprovedpublichealth
(Article47)

87.WhichofthestatementregardingFactory
actiscorrect
a)Childagelessthan14carryingseriousworkearnmoremoney
b)Lessthan14yrnotdoneforfactoryact
c)Morethan72hourworkperweek
d)Morethan82hoursaweek
CorrectAnswer-B
Answer:B-Lessthan14yrnotdoneforfactoryact
TheFactoryActprohibitstheemploymentofchildrenbelow14years
anddeclares15to18yearsasbelongingtotheadolescentgroup.
Adolescentsrequirefitnesscertificatepriortoemploymentinajob.
Actalsoprescribedamaximum48hoursperweek,notexceeding9
Hoursperdaywithatleasthalfhourrestafter5hourcontinuous
work.

88.Burkholderiacepaciaisresistanttowhich
ofthefollowingdrugs:
a)Ceftazidime
b)Trimethoprim-sulfamethoxazole
c)Temocillin
d)Cefotetan
CorrectAnswer-D
Answer:D.Cefotetan
B.cepaciacomplexstrainsareintrinsicallyresistanttoawiderange
ofantimicrobialagents,includingaminoglycosides,polymyxin,first
andsecondgenerationcephalosporins,andcarboxypenicillins
AntimicrobialagentsthatareeffectiveagainstB.cepaciacomplex
includemeropenem,ceftazidime,piperacillin,temocillin,and
trimethoprim-sulfamethoxazole.

89.ShinglesIscausedbywhichofthe
following?
a)Varicella-zoster
b)Herpessimplex
c)CMV
d)None
CorrectAnswer-A
Ans.A.Varicella-zoster
Shingles,alsocalledherpeszoster,isapainfulskinrash.
Shinglesiscausedbyreactivationofthevaricellazostervirus,the
samevirusthatcauseschickenpox.

90.Ureabreathtestisusedfordiagnosisof:
a)H.pylori
b)Campylobacterjejuni
c)E.coli
d)Lactobacillus
CorrectAnswer-A
Answer:A.H.pylori
Theureabreathtestisarapiddiagnosticprocedureusedtoidentify
infectionsbyHelicobacterpylori,aspiralbacteriumimplicatedin
gastritis,gastriculcer,andpepticulcerdisease.Itisbaseduponthe
abilityofH.pyloritoconvertureatoammoniaandcarbondioxide

91.Hyperacutegraftrejectionoccursafter
howmuchtime?
a)24hours
b)2weeksright
c)Inminutes
d)Years
CorrectAnswer-C
Answer:C.Inminutes
HyperacuteTransplantRejectionoccursalmostimmediatelyandis
oftenevidentwhileyouarestillinsurgery.Itiscausedbyaccidental
ABOBloodtypemismatchingofthedonorandrecipientwhich
almostneverhappensanymore..Acuteonsetisinfewweeksto
month.Chroniconsetisfrommonthstoyears.

92.Australianantigenforhepatitisbis?
a)HbSag
b)HbEag
c)HbDag
d)HbVDna
CorrectAnswer-A
Answer:A.HbSag
HBsAg(alsoknownastheAustraliaantigen)isthesurfaceantigen
ofthehepatitisBvirus(HBV).ItindicatescurrenthepatitisB
infection.

93.Whichfungusismostcommonly
associatedwithorbitalcellulitisin
patientswithdiabeticketoacidosis.

a)Candida
b)Mucor
c)Aspergillus
d)Rhizopus
CorrectAnswer-C
Answer:C.Aspergillus
Orbitalcellulitistermisreservedforinfectionsbehindtheorbital
septumwhichmayormaynotspillovertolids.BacterialOCismore
commoninchildrenandfulminantinfection(&ischemicinfarction)
withMucororAspergillustypicallyaffectspatientswithdiabetes(esp
ketoacidosis)andimmunosuppression.PresentationisExtensive
swellingoflidswithchemosisoftenobscureproptosis(i.e.most
commonlylateral&downwards).Proptosiswithimpairedmobility
resultingindiplopiaPainissevere,increasedbymovementofeye
orpressureUnilateral,tender,warm&redperiorbitaledema,painful
ophthalmoplegia.

94.SabinFeldmandyetestisusedfor
diagnosisofwhichofthefollowing
condition:

a)Botulism
b)Toxoplasmosis
c)Sarcoidosis
d)Yellowfever
CorrectAnswer-B
Answer:B.Toxoplasmosis
ASabin?Feldmandyetestisaserologictesttodiagnosefor
toxoplasmosis

95.AcuteHemorrhagicConjunctivitisis
causedbywhichofenterovirustype?
a)69
b)68
c)70
d)71
CorrectAnswer-C
Answer:C.70
Acute
hemorrhagicconjunctivitis(AHC)ischaracterized
byconjunctivalcongestion,vasculardilatation,andonsetofedema
.Serologicstudieshavebeenusefulinshowingthepresenceof
neutralizingantibodiestoCoxsackiegroupA24(CA24)and
enterovirusE70(EV70)strainsasthecausativeagent.

96.Echinococcusgranulosusarecommonly
seeninwhichofthegivenanimals:
a)Dog
b)Cat
c)Fox
d)Pig
CorrectAnswer-A
Answer:A.Dog
Echinococcusgranulosus
,alsocalledthehydatidworm,hyper
tape-wormordogtapeworm.Domesticdogs(Canisfamiliaris)have
beenrecognisedasthedefinitivehostoftheparasite.

97.Ananaerobecausingmultipleabscess
withdischargingsinuses,demonstrating
sulphurgranulesinpusis?

a)Actinomycetes
b)Nocardia
c)Salmonella
d)Tularemia
CorrectAnswer-A
Answer:A.Actinomycetes
Actinomycosisisararesubacutetochronicinfectioncausedbythe
gram-positivefilamentousnon-acidfastanaerobictomicroaerophilic
bacteria,Actinomyces.
Thechronicformhasmultipleabscessesthatformsinustractsand
areassociatedwithsulfurgranules.About70%ofinfectionsaredue
toeitherActinomycesisraeliiorActinomycesgerencseriae.
Thecharacteristicofthediseaseisthesulfurgranuleswhichare
yellow.Theyareformedprimarilybymycelialfragmentswithsome
proteinaceouspolysaccharidecomplexes,whichactasaresistance
mechanismtoavoidandinhibitphagocytosis.
Multipleabscesswithdischargingsinuses,demonstratingsulphur
granulesinpusarecharacteristicsofactinomycetes

98.Wholebloodisusedasasamplefor
whichtest?
a)Bacteria
b)IGRA
c)Genexpert
d)Virus
CorrectAnswer-B
Answer:B.IGRA
Interferon-GammaReleaseAssays(IGRAs)arewhole-bloodtests
thatcanaidindiagnosingMycobacteriumtuberculosisinfection

99.Whichorganismcausingacutebacterial
prostatitis?
a)Enterococcus
b)Streptococcusviridans
c)Peptostreptococcus
d)E.coli
CorrectAnswer-D
Answer:D.E.coli
Aerobicgram-negativebacilliarethepredominantpathogensin
bacterialprostatitis.E.colicause50%?80%ofcases;other
pathogensincludeEnterobacteriaceae(eg,KlebsiellaandProteus,
whichaccountfor10%?30%ofcases),Enterococcusspecies(5%?
10%)

100.Whichofthefollowingorganism
releaseshistamineandcausescombroid
fishpoisoning-

a)Salmonella
b)Staphylococcus
c)P.aeruginosa
d)Weissella
CorrectAnswer-C
Answer:C.P.aeruginosa
Scombroidpoisoning
isoneofthemostcommoncausesof
morbidityassociatedwithfishintakewhichhavenotbeen
refrigeratedproperlyfromthetimetheywerecaughtuntilthetime
theywereserved.Bacteriaactoncompoundsinthefish,releasing
histamine.Processisinducedbyenzymesproducedbyprimarily
entericgram-negativebacteria(e.g.,Morganellamorganii,
Escherichiacoli,KlebsiellaspeciesandPseudomonasaeruginosa)
foundinthefish'scutisandintestines.,.

101.Whoisthefatherofmicrobiology?
a)A.V.L.hook
b)Robertbrown
c)J.CBose
d)Pasteur
CorrectAnswer-A
Answer:A.A.V.L.hook
"AntonivanLeeuwenhoek
"iscommonlyknownas"theFatherof
Microbiology.
AntoniePhilipsvanLeeuwenhoek(24thOct,1632?26thAug,
1723)isknownas'TheFatherofMicrobiology'.Hewasknownso
becauseofhiscontributionstowardstheestablishmentof
microbiology.HewasscientistfromDelft,Netherlandsandis
consideredtobe'TheFirstMicrobiologistintheWorld'.
leeuwenhoekiscalledasfatherofmicrobiology.Hewasthefirst
persontoseebacteriathroughhisselfmadesinglelensed
microscope.Andhenamedthem"animalcules".LouisPasteuris
knowas"fatherofmordenmicrobiology".

102.Cutaneouslarvamigranscausedby
whichorganism?
a)Strongyloides
b)Toxocaracanis
c)Ancylostomabraziliense
d)Necatoramericanus
CorrectAnswer-C
Answer:C.Ancylostomabraziliense
Cutaneouslarvamigrans(CLM)
/Itisaskindiseaseinhumans,causedbythelarvaeofvarious
nematodeparasitesofthehookwormfamily(Ancylostomatidae).
ThemostcommonspeciescausingthisdiseaseintheAmericasis
Ancylostomabraziliense.
Theseparasitesliveintheintestinesofdogs,catsandwildanimals
andshouldnotbeconfusedwithothermembersofthehookworm
familyforwhichhumansaredefinitivehosts,namelyAncylostoma
duodenaleandNecatoramericanus.
Colloquiallycalledcreepingeruptionduetoitspresentation,the
diseaseisalsosomewhatambiguouslyknownas"grounditch"or(in
somepartsoftheSouthernUSA)"sandworms",asthelarvaeliketo
liveinsandysoil.
Anothervernacularnameisplumber'sitch.
ThemedicaltermCLMliterallymeans"wanderinglarvaeintheskin"

103.Afterkidneytransplantationwhich
organismsinfectionismorelikelyto
happens-

a)CMV
b)Klebsiella
c)Streptococcus
d)Staphylococcus
CorrectAnswer-A
Answer:A.CMV
CMVisthemostcommonviralinfectionafterKidney
Transplantation.MostcommonCMVsyndromeinkidneytransplant
patientisfever(mostcommon),leukopenia,hepatosplenomegaly,
myalgiaandarthralgia.

104.Incidenceofadiseaseis4per1000of
populationwithdurationof2years.
Calculatetheprevalence?

a)8/1000
b)4/1000
c)2/1000
d)6/1000
CorrectAnswer-A
Ans.A.8/1000
Prevalence=(IncidenceRate)x(AverageDurationofDisease)

105.Cytotoxicandexpireddrugdisposalis
donebywhichmethod?
a)Dumping
b)Autoclave
c)Landfill
d)Burning
CorrectAnswer-C
Ans.C.Landfill
Colour
Typeof
Treatment
WasteCategory
coding
Container
options
Humanandanimal
wastes,Microbialand Incineration/
Yellow
PlasticBags
Biologicalwastesand DeepBurial
soiledwastes
(Cat1,2,3and6)
Microbiologicaland
Autoclave/
Disinfected
Biologicalwastes,
Microwave/
Red
container/
Soiledwastes,Solid
Chemical
Plasticbags
wastes
Treatment)
(Cat3,6,7)
Autoclave/
Microwave/
Plasticbag,
Wastesharpsand
Blue/White/
Chemical
Punctureproof solidwaste
Transparent
Treatment
container
(Cat4&7)
Destruction
andShredding
Discarded

medicines,
Disposalin
Black
Plasticbag
Cytotoxicdrugs,
securedland
Incinerationashand fills
chemicalwaste
(Cat5,9&10)
Generalwastesuch
Disposedin
Plastic
Green
asofficewaste,food
secured
Container
waste&gardenwaste landfills

106.ForNRRtobe1coupleprotectionrate
shouldbe?
a)50%
b)60%
c)55%
d)75%
CorrectAnswer-B
Ans.B.60%
CoupleProtectionRate(CPR)
Itisanindicatoroftheprevalenceofcontraceptivepracticeinthe
community
Definition:thepercentageofeligiblecoupleseffectivelyprotected
againstchildbirthbyoneortheotherapprovedmethodsoffamily
planning
Sterilization
IUD
Condom
OCP's
NRR=1canbeachievedonlyiftheCPR>60%

107.NewRNTCPsoftwareonlinetomonitor
TBcontrolprogrammeis-
a)NIKSHAY
b)NICHAY
c)E-DOTS
d)NIRBHAI
CorrectAnswer-A
Ans.A.NIKSHAY
TokeepatrackoftheTBpatientsacrossthecountry,the
GovernmentofIndiahasintroducedasystemcalledNIKSHAY.
ThewordiscombinationoftwoHindiwordsNIandKSHAYmeaning
eradicationoftuberculosis.
NIKSHAY(www.nikshay.gov.in)isawebenabledapplication,which
facilitatesmonitoringofuniversalaccesstoTBpatientsdatabyall
concerned.
ThesystemhasbeendevelopedjointlybytheCentralTBDivisionof
theMinistryofHealthandFamilyWelfareandNationalInformatics
Centre(NIC)anditwaslaunchedbytheGovernmentofIndiain
June2012withissueofrequiredadministrativedirectionsfrom
CentralTBDivisionforuseofNIKSHAY

108.Studyunitofecologicalstudyis
a)Population
b)Patient
c)Community
d)Case
CorrectAnswer-A
Ans.A.Population
Inecologicalstudiestheunitofobservationisthepopulationor
community.
Diseaseratesandexposuresaremeasuredineachofaseriesof
populationsandtheirrelationisexamined.
Oftentheinformationaboutdiseaseandexposureisabstractedfrom
publishedstatistics.


109.InascreeningtestforDMoutof1000
population,90werepositive.Thenthe
goldstandardtestwasdoneinwhich
100werepositive.Calculatethe
sensitivity?

a)90/100
b)100/110
c)80/100
d)100/100
CorrectAnswer-A
Ans.A.90/100
So,Truepositive(a)=90
Falsenegative(c)=10
Sensitivity=a/(a+c)=90/100

Sensitivity=a/(a+c)=90/100

110.Whatisthemasschemoprophylaxisfor
meningococcalmeningitis?
a)Rifampicin
b)Chloramphenicol
c)Tetracycline
d)Penicillin
CorrectAnswer-A
Ans.A.Rifampicin
RecommendedChemoprophylaxisforHigh-RiskClose
Contacts:

Age
Dose
Duration
Cautions
RIFAMPICIN:
<1
5mg/kg
Oralevery12hrs
month
2days
>1
10mg/kg Oralevery12hrs Notrecommendedforusein
month
2day
pregnancy
CEFTRIAXONE:
<15
125mg
IMsingledose
years
>15
250mg
IMsingledose
years
CIPROFLOXACIN
>18
500mg
Oralsingledose
Notrecommendedforusein
years
pregnancy

111.Whichamongthefollowingisanactive
formofchlorination?
a)Hypochloriteion
b)Hydrogenchloride
c)Hypochlorousacid
d)Chlorideion
CorrectAnswer-C
Ans.C.Hypochlorousacid
Thedisinfectingactionofchlorineispredominantlydueto
hypochlorousacid.
Hypochlorousacidisthemosteffectiveformofchlorineanditis
almost70-80timesmoreeffectivethanhypochloriteions.


112.Kala-Azarisfoundinallendemicareas
except.
a)WestBengal
b)UP
c)Bihar
d)Assam
CorrectAnswer-D
Ans.D.Assam
UP,WestBengalBiharAndJharkhandaretheendemicstates
ofkalaAzar


113.Riskamongexposedtoriskamongnon
exposedisdefinedtobe?
a)Relativerisk
b)Oddsratio
c)Attributablerisk
d)Noneoftheabove
CorrectAnswer-A
Ans.A.Relativerisk
Peventwhenexposed
RR= Peventwhennotexposed
Instatisticandepidemiology,relativeriskorriskratio(RR)isthe
ratiooftheprobabilityofaneventoccurring(forexample,developing
adisease,beinginjured)inanexposedgrouptotheprobabilityof
theeventoccurringinacomparison,non-exposedgroup.
RR=1meansthatexposuredoesnotaffecttheoutcome
RR<1meansthattheriskoftheoutcomeisdecreasedbythe
exposure
RR>1meansthattheriskoftheoutcomeisincreasedbythe
exposure

114.Pasteurizationisdoneat-
a)73oCFor20min
b)63oCFor30min
c)72oCFor30seconds
d)63oCFor30seconds
CorrectAnswer-B
Ans.B.63oCFor30min
Pasteurizationofmilk,widelypracticedinseveralcountries,notably
theUnitedStates,requirestemperaturesofabout63?C(145?F)
maintainedfor30minutesor,alternatively,heatingtoahigher
temperature,72?C(162?F),andholdingfor15seconds(andyet
highertemperaturesforshorterperiodsoftime).

115.Idealtimegapbetween2livevaccination
-
a)2weeks
b)4weeks
c)8weeks
d)12weeks
CorrectAnswer-B
Ans.B.4weeks
Ifliveparenteral(injected)vaccines(MMR,MMRV,varicella,zoster,
andyellowfever)andliveintranasalinfluenzavaccine(LAIV)arenot
administeredatthesamevisit,theyshouldbeseparatedbyatleast
4weeks.

116.Susceptiblepersondevelopeddisease
withinrangeofIPaftercomingincontact
withprimarycase-

a)Secondaryattackrate
b)Casefatalityrate
c)Primaryattackrate
d)Tertiaryattackrate
CorrectAnswer-A
Ans.A.Secondaryattackrate
SecondaryAttackRate(SAR)Numberofexposedpersons
developingthediseasewithintherangeoftheincubationperiod,
followingexposuretoprimarycase.

117.Outof100womenwhowereofferedocp
forcontraception10womengot
pregnantwhenfollowedfor24months.
WhatisPearl'sindex?

a)10
b)5
c)4
d)2
CorrectAnswer-B
Ans.B.5
NumberofPregnancies*12
Pearl-Index=
*100
NumberofWomen*NumberofMonths
PearlIndex=10x12x100/100x24=5

118.Whichofthefollowingdonotcause
hardnessofwater?
a)Calciumcarbonate
b)Calciumsulphate
c)Calciumbicarbonate
d)Magnesiumbicarbonate
CorrectAnswer-A
Ans.A.Calciumcarbonate
Temporaryhardnessisatypeofwaterhardnesscausedbythe
presenceofdissolvedbicarbonateminerals(calciumbicarbonate
andmagnesiumbicarbonate).
Permanenthardnessiscausedbydissolvedcalciumsulfate(which
isnotremovedbyboiling).

119.Whichofthefollowingisnotanexample
ofdirecttransmissionincommunicable
diseases?

a)Transplacental(vertical)
b)Soil
c)Respiratory
d)STD
CorrectAnswer-C
Ans.C.Respiratory
Themodesoftransmissionofinfectiousdiseasescanbe
classifiedas:
DirectTransmission.Directcontact;Dropletinfection;Contactwith
soil;Inoculationintoskinormucosa;Transplacental
(vertical)transmission.
IndirectTransmission.Vehicle-borne;Water;food/milk;Vector-
borne.Mechanical;Biological.Airborne.


120.Water'sviewisusedtoobtaindiagnostic
informationof:
a)Maxillarysinus
b)Ethmoidalsinuses
c)Frontalsinus
d)Sphenoidsinus
CorrectAnswer-A
ANSWER:A.Maxillarysinus
Waters'view
(alsoknownastheOccipitomentalview)isa
radiographicview,whereanX-raybeamisangledat45?tothe
orbitomeatalline.
Therayspassfrombehindtheheadandareperpendiculartothe
radiographicplate.
Itiscommonlyusedtogetabetterviewofthemaxillarysinuses.

121.Tracheostomyindicationis:
a)Vocalcordreplacement
b)Pharynxreplacement
c)Tracheomalacia
d)Foreignbodyobstructingairway
CorrectAnswer-D
Answer-D.Foreignbodyobstructingairway
Indications
oftracheostomy:
Upperrespiratorytractobstruction;Laryngeal,supralaryngeal,and
trachealcauses.(Causesofstridor)
Lowerrespiratorytractobstruction:(Secretoryobstruction,Wetlung
syndrome).


122.Caldwell'sviewisusedfor:
a)Maxillarysinus
b)Frontalsinus
c)Ethmoidalsinus
d)Alloftheabove
CorrectAnswer-B
Answer-B.Frontalsinus
Caldwell'sview
(orOccipitofrontalview)isaradiographicviewof
skull,whereX-rayplateisangledat20?toorbitomeatalline.
Therayspassfrombehindtheheadandareperpendicularto
radiographicplate.
Itiscommonlyusedtogetbetterviewoffrontalsinuses.

123.Astigmatisminemmetropiceyeof
elderlypersoncontributeto:
a)+1d
b)+2D
c)+3d
d)+4d
CorrectAnswer-C
AnswerC.+3d
Laservisionenhancements
Whenplanningpresbyopia-correctingIOL(Intraocularlens)surgery
inapatientwithahighlevelofpre-existingastigmatism(ie,more
than3D),
abiopticsapproach(ie,IOLfollowedbylaservision
enhancement)maybeneeded.
LRIsaloneareunlikelytocorrecttheastigmatismcompletely.Limbal
RelaxingIncisions(LRI)arearefractivesurgicalprocedureto
correctminorastigmatismintheeye.
Thereareseveraldifferentstrategiesfortheseplannedlaservision
enhancements.Thefirstistoperformthepresbyopia-correctingIOL
surgeryfollowedbyLASIKorPRK.

124.100dayGlaucomaseeninwhichofthe
followingcondition:
a)Centralretinalveinocclusion(CRVO)
b)Neovascularglaucoma
c)Centralretinalarteryocclusion(CRAO)
d)SteroidinducedGlaucoma
CorrectAnswer-A
Answer-A,Centralretinalveinocclusion(CRVO)
100daysglaucomaisaneovascularglaucomaoccurringin
CRVO
.
CentralRetinalveinocclusion(CRVO)
1.Predisposingfactors:-
Increasingage?seenin6th-7th-decadesoflife.
Systemichypertensionisthemostcommoncause.
Blooddyscrasias?hyperviscosityduetochronicleukemias
andpolycythemia
RaisedIOP(POAG)
Periphlebitis?sarcoidosis,Beh?et'sdisease
2.ClassificationofCRVO:-
Non?Ischemic
Ischemic
3.ClinicalFeatures:-
Tortuosityanddilationofretinalveins
Flameshapedhemorrhage?developinthenervefiberlayerofthe
retina,especiallyaroundtheopticdisc,asaresultofthehigh
intravascularpressurethatdilatestheveinsandcollateralvessels.
Cotton?woolspotsandappearanceofcollateralsattheopticdisc
areitsprominentdiagnosticsigns.

Opticdiscedemaandhyperemiaareseen.
4.Complications:-Rubeosisiridisandneovascularglaucoma
(NVG)occurinmorethan50percentcaseswithin3months(so
alsocalledas90daysglaucoma),afewcasesdevelopvitreous
hemorrhageandproliferativeretinopathy.
5.Treatment:-Panretinalphotocoagulation(PRP)orcryo-
application,ifthemediaishazy,mayberequiredtoprevent
neovascularglaucomainpatientswithwidespreadcapillary
occlusion.
Photocoagulationshouldbecarriedoutwhenmostoftheintraretinal
bloodisabsorbed,whichusuallytakesabout3-4months.
100-dayglaucomaorNVGresultsfromconditionswhichleadto
neovascularizationintheeyeeg.PDR,CRVO,Retinalmalignancies
andrarelyinCRAO.
Hemorrhagicglaucomaisalsoknownas100day
glaucoma
becauseitstarts3monthsaftertheepisodeofcentral
retinalveinocclusion.

125.QRothspotsisseenin:
a)Uvealmelanoma
b)Acuteleukaemia
c)Botha&b
d)Noneoftheabove
CorrectAnswer-B
Answer-B.Acuteleukaemia
Roth'sspots
areretinalhemorrhageswithwhiteorpalecenters.
Composedofcoagulatedfibrinincludingplatelets,focalischemia,
inflammatoryinfiltrate,infectiousorganisms,orneoplasticcells.
Roth'sspotsmaybeobservedinleukemia,diabetes,subacute
bacterialendocarditis,perniciousanemia,ischemicevents,
hypertensiveretinopathyandrarelyinHIVretinopathy.
Roth'sspotsarenamedafterMoritzRoth.


126.Yokemuscleofrightlateralrectus:
a)Ltmedialrectus
b)Ltsuperiorrectus
c)Ltlateralrectus
d)Ltinferioroblique
CorrectAnswer-A
Answer-A.Ltmedialrectus
Contralaterallypairedextraocularmusclesthatwork
synergisticallytodirectthegazeinagivendirection.
Forexample,indirectingthegazetotheright,therightlateral
rectusandleftmedialrectusoperatetogetherasyokemuscles.
YokeMuscles:2Muscles(1ineacheye)thataretheprimemovers
oftheirrespectiveeyesinagivenpossitionoofgaze
For-Example-whentheeyesmoveintorightgaze(dextroversin),
therightlateralrectus&theleftmedialrectusmuscleareyoke
muscles.

127.SevereConjunctivitiscausedby:
a)Neisseria
b)Staphylococcus
c)Streptococcus
d)Haemophilus
CorrectAnswer-A
ANSWER-A.Neisseria
Themostcommoncausesofacutebacterialconjunctivitisare
Staphylococcusaureus,Streptococcuspneumoniae,and
Haemophilusinfluenzae.
HyperacutecasesareusuallycausedbyNeisseriagonorrhoeaeor
N.meningitidis.
Chroniccasesofbacterialconjunctivitisarethoselastinglongerthan
3weeks,andaretypicallycausedbyStaphylococcusaureus,
Moraxellalacunata,orgram-negativeentericflora.
Neisseriagonorrhoeaecausesgonococcalconjunctivitis,which
usuallyresultsfromsexualcontactwithapersonwhohasagenital
infection.
Theincidenceratesofgonococcalconjunctivitisincreaseduring
springandsummer.
Thisisapotentiallydevastatingocularinfection,becauseN.
gonorrhoeaecancausesevereulcerativekeratitis,whichmay
rapidlyprogresstocornealperforation.

128.WhichisexampleoftheSimpleMyopic
Astigmatismamongtheprescriptions
givenbelow:

a)Rx(+)sphere
b)Rxwillbeplano(-)
c)Rxwillbe(-)sphere
d)(-)(+)(+)(-)onboth90and180degreeaxis
CorrectAnswer-B
Answer-B.Rxwillbeplano(-)
Wheneyewearprescriptionsarewritten,theycanbeclassifiedinto
differentareasdependingonthepowerorrefractiveerror.
Therearesevencategoriestowhichprescriptionscanfall:
1. SimpleHyperopia,theRxwillbe(+)sphere
2. SimpleMyopia,theRxwillbe(-)sphere
3. SimpleMyopicAstigmatism,theRxwillbeplano(-)
4. SimpleHyperopicAstigmatism,theRxwillbe(+)
5. CompoundHyperopicAstigmatism,majormeridianpowerwillbe
(+)(+)onboth90and180degreeaxis
6. CompoundMyopicAstigmatism,majormeridianpowerwillbe(-)
(-)onboth90and180degreeaxis
7. MixedAstigmatism,majormeridianpowerswillbeopposites(-)
(+)(+)(-)onboth90and180degreeaxis

129.Blowoutfractureoforbitinvolves:
a)Floor
b)Medialwall
c)Lateralwall
d)Roof
CorrectAnswer-A
Answer-A.Floor
Orbitalfloorfracture
,alsoknownas"blowout"fractureoftheorbit.
Blowoutfractureoforbitinvolves:
1. Fracturesoftheorbitalfloorarecommon:itisestimatedthatabout
10%ofallfacialfracturesareisolatedorbitalwallfractures(the
majorityofthesebeingtheorbitalfloor),andthat30-40%ofallfacial
fracturesinvolvetheorbit.
2. Theanatomyoftheorbitalfloorpredisposesittofracture.
3. Theinferiororbitalneurovascularbundle(comprisingtheinfraorbital
nerveandartery)courseswithinthebonyflooroftheorbit;theroof
ofthisinfraorbitalcanalisonly0.23mmthick,andtheboneofthe
posteriormedialorbitalflooraverages0.37mmthick.
4. Bycontrast,theboneofthelateralportionoftheorbitalfloor
averages1.25mmthick,over5timesthethicknessoftheboneover
theneurovascularbundle.Asonemightsuspect,itisthisverythin
areaoftheorbitalflooroverlyingtheneurovascularbundlewhere
isolatedorbitalfloorfracturesinvariablyoccur.



130.Whichofthefollowingisendogenous
pyrogen
a)PGE2
b)PGD2
c)PGF2alpha
d)PGI2
CorrectAnswer-A
Answer:-A.PGE2
Pyrogen
Pyrogensaresubstancesthatcausefever.
Pyrogensmaybeexogenousorendogenous
ExogenousBacterialtoxins
EndogenousIL-1,TNF-a,IL-6,Interferons,Ciliaryneurotrophic
factor
ThesepyrogensincreasethelevelofPGE,inthehypothalamusthat
elevatesthethermoregulatorysetpointandcausesfever.

131.InBarttersyndromedefectisseenin:
a)DefectinPCT
b)DefectinDCT
c)Defectinthickascendinglimbofloopofhenle
d)None
CorrectAnswer-C
Answer:C-Defectinthickascendinglimbofloopofhenle
Autosomalrecessivedisorder.
Geneticdefectinthethickascendinglimboftheloopofhenle
DefectsinNa-K-2C/coLtransporter,KorCl
channelsresultinlackofconcentratingability

132.Notseeninallergicpulmonary
aspergillosisis
a)HighIgElevel
b)Recurrentpneumonia
c)Occurrenceinpatientswitholdcavitarylesions
d)Pleuraleffusion
CorrectAnswer-B
Answer:B-Recurrentpneumonia
Allergicbronchopulmonaryaspergillosisisapulmonary
hypersensitivitydisordercausedbyallergytofungalantigensthat
colonizethetracheobronchialtree.
Itmostcommonlyoccursinatopicasthmaticindividualsinresponse
toantigenofaspergillusspecies.
Maindiagnosticcriteria
ClinicalhistoryofAsthmaQ
Pulmonaryinfiltrates(transient/fleetingorfixed)Q
Peripheraleosinophilia(>1000/,uL)Q
ImmediateskinreactivitytoAspergillusantigen(whealandflare
response)
SerumprecipitinstoA.fumigatus
ElevatedserumIgElevels(>100Ong/ml)
Central/proximalbronchiectasis
Secondarydiagnosticcriteria
Historyofbrownishplugsinsputum
Identification/cultureofA.,fumigatusfromsputum
Lateskinreactivitytoaspergillusantigen-CMDT
ElevatedIgE(andIgG)classantibodiesspecificforA.fumigatus-
Harrisons

Note
ElevatedIgE(andIgG)classantibodiesspecificforA.fumigatushas
beenmentionedasasecondarydiagnosticcriteriainHarrison's
textbookwhileFishman'stextbookincludesthisasamain/major
diagnosticcriteria.

133.PseudoPPulmonale
a)Hypokalemia
b)Hyponatremia
c)Hypocalcemia
d)Hypercalcemia
CorrectAnswer-A
Answer:A.Hypokalemia
Insomecasestherecanbeanotched(orbifid)p-waveknownas"p
mitrale",indicativeofleftatrialhypertrophywhichmaybecausedby
mitralstenosis.Theremaybetallpeakedp-waves.Thisiscalled"p-
pulmonale"andisindicativeofrightatrialhypertrophyoften
secondarytotricuspidstenosisorpulmonaryhypertension.
Asimilarpicturecanbeseeninhypokalemia(knownas"pseudop-
pulmonale").

134.AutomaticImplantableCardioverter
Defibrillator,(AICD)implantationisdone
forwhichoffollowingconditions:

a)Brugadasyndrome
b)Ventricularfibrillation
c)AcutecoronarysyndromewithlowEF
d)Alloftheabove
CorrectAnswer-D
Answer:D.Alloftheabove
AnAutomaticImplantableCardioverterDefibrillator,(AICD),isa
smallelectronicdevicethatisimplantedintoyourchesttomonitor
andcorrectanabnormalheartrhythm,orarrhythmia.Thesedevices
areusedtotreatseriousandlife-threateningarrhythmiasandare
themosteffectivewayofdoingso.
Brugadasyndromeisaconditionthatcausesadisruptionofthe
heart'snormalrhythm.
Ventricularfibrillationisaheartrhythmproblemthatoccurswhen
theheartbeatswithrapid,erraticelectricalimpulse.
Acutecoronarysyndromeisatermusedtodescribearangeof
conditionsassociatedwithsudden,reducedbloodflowtotheheart.

135.Whatisthelineoftreatmentfor
intractableSydenhamchorea?
a)Haloperidol
b)Valproate
c)Warfarin
d)Risperidone
CorrectAnswer-B
Answer:B.Valproate
Sydenham'schorea(SC)/Choreaminor/Rheumaticchorea
(RC)/St.Vitus'sDance
Majordiagnosticcriteriaforrheumaticdisease
Mostcommonacquiredchoreaduringchildhood
Characterizedbyrapid,uncoordinatedjerkingmovementsprimarily
affectingtheface,handsandfeet
BenzodiazepinesfacilitatetheactionofGABAandvalproate
enhancestheactiononGABA,hencetheseagentsareusedtotreat
chorea

136.Neurofibromatosis1criteriaexcept
a)Braintumor
b)Acousticneuromas
c)Pseudoarthrosis
d)Cafe-au-laitspots
CorrectAnswer-B
Answer:B-Acousticneuromas
Clinicaldiagnosisrequirespresenceofatleast2of7criteriato
confirmpresenceofneurofibromatosis,type1.
Despitesuspicion,mostsignsdonotappearuntillatechildhood/
adolescent
The7clinicalcriteriausedtodiagnoseNF1areasfollows:
Sixormorecaf?-au-laitspots/hyperpigmentedmaculesgreater
thanorequalto5mmindiameterinprepubertalchildren&15mm
postpubertal
Axillaryoringuinalfreckles(>2)
Twoormoretypicalneurofibromasoroneplexiformneurofibroma
Opticnerveglioma
Twoormoreirishamartomas(Lischnodules)(oftenidentifiedonly
throughslit-lampexamination)
Sphenoiddysplasiaortypicallong-boneabnormalitiessuchas
pseudarthrosis
Strongfamilyhistory(1stdegreerelativewithNF1)

137.Mostcommonsiteofchronicgastric
ulcer
a)Pyloricantrum
b)Upperpartoflessercurvature
c)Lowerpartoflessercurvature
d)Segmentoflargeintestine
CorrectAnswer-A
Answer:A-Pyloricantrum
Amajorcausativefactor(60%ofgastric&upto50?75%of
duodenalulcers)ischronicinflammationduetoHelicobacter
pylori
thatcolonizestheantralmucosa.

138.ApproximatetimeintervalbetweenHIV
infection&manifestationofAIDSis?
a)7.5yr
b)10yr
c)12yr
d)11yr
CorrectAnswer-B
Answer:B-10years
Theapproximatetimeframerequiredforincubationisusually10
years.
mediantimefrominfectiontothedevelopmentofAIDSinadultsis
about9years.AsmallproportionofuntreatedHIV-infectedpeople
arelong-termnon-progressors,withCD4countsinthereference
rangefor10yearsormore.Somelong-termnon-progressorshave
undetectableviralloadsandareknownas`elitecontrollers'.

139.Heller'smyotomyisdonefor
a)Zenker'sdiverticulum
b)Achalasiacardia
c)Bunions
d)Kneearthroscopy
CorrectAnswer-B
Answer:B-Achalasiacardia
Achalasia,adisorderofesophaguscharacterizedbyprogressive
inabilitytoswallowsolids&liquids.
Causesincludeweakenedesophagealmuscles&issueswithlower
esophagealsphincterrelaxation
HellerMyotomy,surgicalprocedureofferslongtermsymptomatic
relieftothesepatients.
Itinvolvesweakeningofmusclesatgastroesophagealjunction,
allowingthevalvebetweenoesophagus&stomachtoremainopen.

140.Myocardialstunningpatternnot
matchingtheECG.Whatisthe
diagnosis.

a)Takotsubocardiomyopathy
b)Restrictivecardiomyopathy
c)Brigade'scardiomyopathy
d)Pericardialsomething
CorrectAnswer-A
Answer:A-Takotsubocardiomyopathy
"MyocardialStunning"
isastatewherecertainsegmentsof
myocardium
(correspondingtoareaofmajorcoronary
occlusion)showsformsofcontractileabnormality.
Asegmentaldysfunctionpersistingforavariableperiodoftime,
abouttwoweeks,evenafterischemiahasbeenrelieved(byfor
instanceangioplastyorcoronaryarterybypasssurgery).
Takotsubocardiomyopathy/Takotsubosyndrome,atemporary
conditionwhereyourheartmusclebecomessuddenlyweakenedor
'stunned'.

141.Alternativedrugforcardiacarrestin
placeofepinephrineis?
a)Amiodaroneinfusion
b)Atropine
c)Highdosevasopressin
d)Adenosine
CorrectAnswer-C
Answer:C-Highdosevasopressin
Vasopressinisanalternativevasopressorathighdoses,causes
vasoconstrictionbydirectlystimulatingsmoothmuscleV1
receptors.
AmericanHeartAssociation(AHA)guidelinesstates"Vasopressinis
areasonablefirst-linevasopressorinpatientswithventricular
fibrillationorpulselessventriculartachycardia.Additionally,the
guidelinescommentthatonedoseofvasopressin40Umayreplace
thefirstorseconddoseofepinephrineinallpulselesssudden
cardiacarrestscenarios,includingasystoleandpulselesselectrical
activity.

142.Patientpresentingwithcutaneous
vasculitis,glomerulonephritis,peripheral
neuropathy,Whichinvestigationistobe
performednextthatwillhelpyou
diagnosethecondition?

a)ANCA
b)RAfactor
c)Hbsag
d)MIF
CorrectAnswer-A
Answer:A-ANCA
Anti-neutrophilcytoplasmicantibodies
(ANCAs):
Groupofautoantibodies
IgGtypemainly,
Producedagainstantigensincytoplasmofneutrophilgranulocytes&
monocytes.
Particularlyassociatedwithsystemicvasculitis,socalled"ANCA-
associatedvasculitis".

143.Cryoglobulinemia
a)Hepatitisc
b)Ovariancancer
c)Diabetes
d)Leukemia
CorrectAnswer-A
Answer:A.HepatitisC
Cryoglobulinemia/Cryoglobulinemicdiseasewithlargeamountsof
cryoglobulinsinblood
Cryoglobulinsareproteins(mostlyimmunoglobulinsthemselves)
Becomeinsolubleatreducedtemperatures.
MixturesofmonoclonalorpolyclonalIgM,IgG,and/orIgA&blood
complementproteinslikeC4areassociatedwithcasesofinfectious
diseases,particularlyHepatitisCinfection,

144.Causesofhypokalemicmetabolic
alkalosiswithhypertension
a)Liddlesyndrome
b)Barttersyndrome
c)Gitelmansyndrome
d)Renaltubularacidosis
CorrectAnswer-A
Answer:A.Liddlesyndrome
Liddlesyndrome-Rarehereditarydisorder
Increasedactivityoftheepithelialsodiumchannel(E-NaCh)
Activatingkidneystoexcretepotassiumretainingexcessivesodium
&water,leadingtohypertension.

145.GoldcriteriaforverysevereCOPD
a)Fev1/Fvc<70andFev1<30
b)Fev1/Fvc<70andFev1<70
c)Fev1/Fvc<70andFev1<50
d)BothAandC
CorrectAnswer-A
Answer:A.Fev1/Fvc<70andFev1<30
COPDshouldbeconsideredinanypatientwhohasdyspnea,
chroniccoughorsputumproduction,and/orahistoryofexposureto
riskfactorsforthedisease.
Spirometryisrequiredtomakethediagnosis.
Presenceofpost-bronchodilatorFEV1/FVC<0.70confirmsthe
presenceofpersistentairflowlimitation.
StageIV/VerySevereCOPD
Severeairflowlimitation(FEV1/FVC<70%;FEV1<.30%predicted)
orFEV1<50%predictedpluschronicrespiratoryfailure.
PatientsmayhaveVerySevere(StageIV)COPDevenifFEV1is>
30%predicted.

146.ABPIincreasesartificiallyin
a)Arteriosclerosiscalcifiedarteries
b)Ischemiculcers
c)Intermittentclaudication
d)DVT
CorrectAnswer-A
Answer:A.Arteriosclerosiscalcifiedarteries
Theankle-brachialpressureindex(ABPI)/Ankle-Brachialindex
(ABI):
Ratioofbloodpressureatankletobloodpressureinupperarm
(brachium).
Comparedtoarm,lowerbloodpressureinlegsuggestsblocked
arteriesduetoperipheralarterydisease(PAD).
Anklebrachialpressureindex(ABPI)isamethodforthe
quantificationofperipheralvasculardiseasethatresultsfrom
advancedatherosclerosis.

147.Minimaldysfunctionsyndromeseenin
a)Dyslexia
b)ADHD
c)Mentalsubnormality
d)Down'ssyndrome
CorrectAnswer-A
Answer:A.Dyslexia
ClinicalfeatureofMinimaldysfunctionsyndromeincludesdyslexia.
Minimalbraindysfunction:
Neurodevelopmentaldisorder.
Characterizedbyevidencesofimmaturityinvolvingcontrolof
activity,emotions,&behavior
Specificlearningdisabilitiesinvolvingthecommunicatingskills
neededinreading,writing,andmathematics.
Inabilitytomaintainattention&concentration
Inabilitytoskillfullyblendtheauditory&visualfunctionsessentialin
languageperformance

148.VanNuysprognosticindexisnotbased
on:
a)Age
b)Microcalcification
c)Size
d)ERstatus
CorrectAnswer-D
Answer:D.ERstatus
DCISmaybeclassifiedusingtheVanNuyssystem,whichcombines
the
1. Patient'sage,
2. TypeofDCIS
3. Presenceofmicrocalcification,
4. Theextentofresectionmargin
5. Thesizeofthedisease.
VanNuysPrognosticIndex
Parameter
Score1
Score2
Score3
VanNuys
Group1
Group2
Group3
Classification
Non-high
Non-High
Highnuclear
nucleargrade
nucleargrade gradewithor
withoutnecrosis withnecrosis
withoutnecrosis
Margins
10mm
1?9mm
Size
16?40mm
>40mm
Age
>60
40?60
<40


149.Commoncauseofchronicpancreatitis
a)Chronicalcohol
b)Chronicpancreaticcalculi
c)pancreasdivisum
d)Gallbladderstones
CorrectAnswer-A
Answer:A.Chronicalcohol
"Worldwide,alcoholconsumptionandabuseisassociatedwith
chronicpancreatitisinupto70%ofcases"

150.ThefollowingstatementaboutKeloidis
true
a)Itcontaingrowthfactor
b)Extendedexcisionisthetreatmentofchoice
c)Itdonotextendbeyondthewound
d)Noneoftheabove
CorrectAnswer-A
Answer:A.Itcontaingrowthfactor
Vaccinations,injections,insectbites,earpiercing,ormayarise
spontaneously.
Keloidstendtooccur3monthstoyearsaftertheinitialinsult,and
evenminorinjuriescanresultinlargelesions.
Theyvaryinsizefromafewmillimeterstolarge,pedunculated
lesionswithasofttorubberyorhardconsistency.
Althoughtheyprojectabovesurroundingskin,theyrarelyextendinto
underlyingsubcutaneoustissues.
Certainbodysiteshaveahigherincidenceofkeloidformation,
includingtheskinoftheearlobeaswellasthedeltoid,presternal,
andupperbackregions.
Theyrarelyoccuroneyelids,genitalia,palms,soles,oracross
joints.
Keloidsrarelyinvolutespontaneously,whereassurgicalintervention
canleadtorecurrence,oftenwithaworseresult.

151.Whichofthefollowinglayersarecut
duringfasciotomy?
a)Skin
b)Skin+subcutaneousfascia
c)Skin+subcutaneoustissue+Superficialfascia
d)Skin+subcutaneoustissue+Superficialfascia+deepfascia
CorrectAnswer-D
Answer:D.Skin+subcutaneoustissue+Superficialfascia+deep
fascia
Fasciotomyorfasciectomyisasurgicalprocedurewherecomplete
openingofallfascialenvelopeisdonetorelievetensionorpressure
commonlytotreattheresultinglossofcirculationtoanareaoftissue
ormuscle.Fasciotomyisalimb-savingprocedurewhenusedto
treatacutecompartmentsyndrome.

152.Whichstatementisnottrueregarding
crohn'sdisease:
a)Rectumisnotinvolved
b)Continuouslesionvisualizedinendoscopy
c)Noncaseatinggranulomas
d)Cobblestoneappearance
CorrectAnswer-B
Answer:B.Continuouslesionvisualizedinendoscopy
Crohn'sdiseaseisfrequentlyassociatedwith"skip
lesions,"discontinuousareasofactivediseaseinthecolonand
smallintestinewithinterveningsegmentsthatappearnormal.

153.Whichisthebestinvestigationfor
carcinomaheadofpancreas:
a)Guidedbiopsy
b)ERCP
c)Transduodenal/transperitonealsampling
d)EUS
CorrectAnswer-D
Answer:D
Medicalimagingtechniques,suchascomputedtomography(CT
scan)andendoscopicultrasound(EUS)areusedbothtoconfirmthe
diagnosisandtohelpdecidewhetherthetumorcanbesurgically
removed

154.Anabdominalmassisbest
demonstratedincongenitalhypertrophic
pyloricstenosisby:

a)Inpalpationovertheepigastrium
b)Inlefthypochondriac
c)Rightiliacfossa
d)Duringfeeding
CorrectAnswer-D
Answer:D.Duringfeeding
Congenitalhypertrophicpyloricstenosis
Presentation
Vomitingisthepresentingsymptom(childvomitsmilkandnobileis
present)
Immediatelyaftervomitingthechildishungryi.e.lossofappetite
doesnotoccur.
Weightlossisstrikingandrapidlytheinfantbecomesemaciatedand
dehydrated.
However,greaterawarenessofpyloricstenosishasledtoearlier
identificationofpatientsandhencewithfewerincidencesofchronic
malnutritionandseveredehydration-Nelson10th/1130
Thediagnosisisusuallymadewithatestfeed:Inthis,thebaby
isfedwiththebottlebyanurseormotherandsurgeon:
IHPS/CHPScanbediagnosedclinically.Duringatestfeed,thereis
visiblegastricperistalsispassingfromlefttorightacrosstheupper
abdomenandinarelaxedbaby,thepyloric'tumor'ispalpableasan
'olive'intherightupperquadrant.
Thediagnosiscanbeconfirmedbyanultrasound,whichshows

thethickenedpyloricmuscle.


155.CalculateGCSof25yearoldheadinjury
patientwithfollowingparameters
confused,openingeyesinresponseto
pain,localizingpainwillbe-

a)6
b)11
c)12
d)7
CorrectAnswer-B
Answer:B.11
Glasgowcomascale:


156.RETproto-oncogeneisassociatedwith
thedevelopmentof
a)Medullarycarcinomathyroid
b)Astrocytoma
c)Paraganglioma
d)Hurthlecelltumorthyroid
CorrectAnswer-A
Answer:A.Medullarycarcinomathyroid
RETproto-oncogeneisagrowthfactorreceptor(receptortyrosine
kinase)
TheRETproteinisareceptorfortheglialcellline-derived
neurotrophicfactorandstructurallyrelatedproteinsthatpromotecell
survivalduringneuraldevelopment.
RETisnormallyexpressedinthefollowingcells
ParafollicularCcellsofthethyroid
Adrenalmedulla
Medullarycarcinomamayoccurincombinationwithadrenal
phaeochromocytomaandhyperparathyroidism(HPT)(usuallydueto
hyperplasia)inthesyndromeknownasmultipleendocrineneoplasia
type2A(MEN-2A).
Thesearetumorsoftheparafollicular(Ccells)derivedfromthe
neuralcrestratherthanthecellsofthethyroidfollicleasareother
primarythyroidcarcinomas
Whenthefamilialformisassociatedwithprominentmucosal
neuromasinvolvingthelips,tongueandinneraspectoftheeyelids,
withaMarfanoidhabitus,thesyndromeisreferredtoasMENtype
2B.
RETassociations:MEN2a,MEN2b,Familialmedullarycarcinoma

thyroid.

157.Cutoffforsurgeryinanabdominalaortic
aneurysminasymptomaticpatients-
a)5.5cm
b)6.5cm
c)7.5cm
d)8.5cm
CorrectAnswer-A
Answer:A.5.5cm
Operativerepairoftheaneurysmwiththeinsertionofa
prostheticgraftorendovascularplacementofaorticstentgraft
isindicatedfor:
Abdominalaorticaneurysmsofanysizethatareexpandingrapidly
orareassociatedwithsymptoms.
Forasymptomaticaneurysms,abdominalaorticaneurysmrepairis
indicatedifthediameteris>5.5cm.

158.Whichofthefontanelleisthelastto
close?
a)Anterolateral
b)Anterior
c)Lateral
d)Occipital
CorrectAnswer-B
Ans.B.Anterior
Theposteriorfontanellenormallycloses2to3monthsafterbirth
Thesphenoidalfontanelleisthenexttoclosearound6monthsafter
birth
Themastoidfontanelleclosesnextfrom6to18monthsafterbirth;
Theanteriorfontanelleisgenerallythelasttoclosebetween18?24
months.

159.WhichenzymedeficiencycausesLesch?
Nyhansyndrome?
a)Hypoxanthine-guaninephosphoribosyltransferase(HGPRT)
b)Xanthineoxidase
c)Adeninephosphoribosyltransferase(APRT)
d)AMPdeaminase
CorrectAnswer-A
Ans.A.Hypoxanthine-guaninephosphoribosyltransferase
(HGPRT)
Lesch?Nyhansyndrome(LNS),alsoknownasjuvenilegout,isa
rareinheriteddisordercausedbyadeficiencyoftheenzyme
hypoxanthine-guaninephosphoribosyltransferase(HGPRT),
producedbymutationsintheHPRTgenelocatedontheX
chromosome


160.Whichvaccineisnotincludein
indradhanushmission?
a)Tuberculosis
b)Measles
c)JapaneseEncephalitis
d)Diphtheria
CorrectAnswer-C
Ans.C.JapaneseEncephalitis
TheMissionIndradhanush,depictingsevencoloursofthe
rainbow,targetstoimmunizeallchildrenagainstsevenvaccine
preventablediseasesnamely:
Diphtheria
Pertussis
Tetanus
ChildhoodTuberculosis
Polio
HepatitisB
Measles.

161.WhichamongthefollowingistheMost
commoncauseforneonatalblindness?
a)Neisseriagonorrhoeae
b)Chlamydiatrachomatis
c)Klebsiella
d)Enterobacter
CorrectAnswer-A
Ans.A.Neisseriagonorrhoeae
Ophthalmianeonatorum(ON)isdefinedasapurulentconjunctivitis
occurringduringthefirstfourweeksoflife.
ThetwomostcommoncausativeagentsareNeisseriagonorrhoeae
andChlamydiatrachomatis,theformerbeingofmoreconcernhere
becauseofitspropensitytocauseblindness.

162.MusclesaffectedinDequervain
tenosynovitis-
a)Abductorpollicislongusandextensorpollicisbrevis
b)Adductorpollicislongusandextensorpollicisbrevis
c)AbductorpollicislongusandFlexorpollicisbrevis
d)AdductorpollicislongusandFlexorpollicisbrevis
CorrectAnswer-A
Ans.A.Abductorpollicislongusandextensorpollicisbrevis
DeQuervainsyndrome,isatenosynovitisofthesheathortunnel
thatsurroundstwotendonsthatcontrolmovementofthethumb.
DeQuervainsyndromeinvolvesnon-inflammatorythickeningofthe
tendonsandthesynovialsheathsthatthetendonsrunthrough.
Thetwotendonsconcernedarethoseoftheextensorpollicisbrevis
andabductorpollicislongusmuscles.

163.WhatismeantbyPerilunatedislocations
?
a)Lowerradius,scaphoidandlunateandcapitateallinsame
plane
b)Lowerradius,scaphoidandcapitateinalignment,lunatealone
outofplane
c)Lowerradius,scaphoidandLunateinalignment,capitatealone
isoutofplane
d)Bothlunateandcapitateareoutofplane
CorrectAnswer-B
Ans.B.Lowerradius,scaphoidandcapitateinalignment,lunate
aloneoutofplane
Perilunatedislocationandperilunatefracturedislocationareinjuries
thatinvolvetraumaticruptureoftheradioscaphocapitate(RSC)
ligament,thescapholunateinterosseousligament,andthe
lunotriquetralinterosseousligament.
Lateralradiographswillreveallossofcollinearitybetweenthe
capitate,lunate,andradius
Typicallythecapitateislocateddorsaltothelunateandisaligned
withtheradius


164.Fallenfragmentsign-
a)Simplebonecyst
b)Osteosarcoma
c)Adamantinoma
d)Aneurysmalbonecyst
CorrectAnswer-A
Ans.A.Simplebonecyst
Thefallenfragmentsignreferstothepresenceofabonefracture
fragmentrestingdependentlyinacysticbonelesion.Thisfindingis
saidtobepathognomonicforasimple(unicameral)bonecyst
followingapathologicalfracture.
Althoughithasoccasionallybeenreportedwithothercysticlesions,
e.g.eosinophilicgranuloma

165.Youarepostedasaninternin
causality.Whichamongthefollowing
patientswithfracturewillbeyour1st
prioritytocallorthoPGandinform?

a)Patient'sfingerisblackening
b)Patientcan'textendhisarm
c)A10cmabrasion
d)IntraarticularfractureofElbowJoint
CorrectAnswer-A
Ans.A.Patient'sfingerisblackening
BlackeningoffingerafterFractureisanindicationofcutofofblood
supplythatmayleadtoseverecomplicationlikegangrene.Sothis
shouldbethefirstPrioritytotreatamongalltheaboveoptions

166.InRheumatoidarthritis,whichtypeof
cellsareprominentlypresent?
a)Bcells
b)Tcells
c)Macrophages
d)Dendriticcells
CorrectAnswer-C
Ans.C.Macrophages
Synovialliningorintimallayer:Normally,thislayerisonly1-3cells
thick.InRA,thisliningisgreatlyhypertrophied(8-10cellsthick).
Primarycellpopulationsinthislayerarefibroblastsand
macrophages.

167.8thand9thribcostalcartilageforms
whichtypeofjoint?
a)Costochondraljoint
b)Interchondraljoint
c)Synovialjoint
d)Costovertebraljoint
CorrectAnswer-C
Ans.C.Synovialjoint
Thesixth,seventh,eighth,ninthandtenthcostalcartilagesare
jointedwitheachotheralongthebordersbysynovialjoints.
Costochondraljointmeansthejointbetweentheribanditscostal
cartilage.
Thefirstcostalcartilageofbothsidesattachtothemanubrium
sterni.Atthisjoint,nomovementispossible.
Thesecondcostalcartilagearticulateswiththebodyofsternumand
themanubriumsternibyasynovialjointwheremovementis
possible.
Thethirdtoseventhcostalcartilagesarticulatewithlateralborderof
thebodyofsternumatmobilesynovialjoints.

168.Tomsmithsepticarthritisis-
a)AcuteGonococcalarthritis
b)Smallpoxarthritis
c)Septicarthritisofinfancy
d)Chronicpyogenicarthritis
CorrectAnswer-C
Ans.C.Septicarthritisofinfancy
Septicarthritisofinfancy(Tomsmithsepticarthritis)
Itisasepticarthritisofhipseenininfants
Theonsetisacutewithrapidabscessformation,whichmayburst
outorbeincisedandhealsrapidly.
Telescopetestispositive
Clinicallythisconditionresemblesacongenitaldislocationofhip

169.Whichofthefollowingiscorrect
regardingplacenta?
a)Placentalarteryprovidesnutrientsthroughumbilicalcordto
baby
b)PlacentahasWharton'sjelly
c)Placentahas2veinsand1artery
d)Estrogenissecretedbyplacenta
CorrectAnswer-B
Ans.B.PlacentahasWharton'sjelly
Theumbilicalcordisastructurethatprovidesvascularflowbetween
thefetusandtheplacenta.
Itcontainstwoarteriesandonevein,whicharesurroundedand
supportedbygelatinoustissueknownasWharton'sjelly.

170.Acutefattylivercommonlyseenin
pregnancyat-
a)3rdtrimester
b)1sttrimester
c)Immediatepostpartum
d)Intrapartum
CorrectAnswer-A
Ans.A.3rdtrimester
Acutefattyliverofpregnancy(AFLP)isarare,potentiallyfatal
complicationthatoccursinthethirdtrimesterorearlypostpartum
period.
Acutefattyliverofpregnancyismorecommonlyassociatedwith:
Malefetus
Firstpregnancy
Maternalobesity
Itisduetodefectinthelong-chain-3-hydroxyacyl-CoA-
dehydrogenasepathway.
Itisuncommoninsubsequentpregnancies.

171.Establishmentoffetoplacental
circulationseenat-
a)11to13days
b)20to22days
c)7days
d)25to26days
CorrectAnswer-B
Ans.B.20to22days
ImportantEventsFollowingFertilization

0'hour
Fertilization(day-15fromLMP)
30hours
2cellstage(blastomeres)
40?50hours
4cellstage
72hours
12cellstage
96hours
16cellstage.Morulaenterstheuterinecavity
5thday
Blastocyst
4?5thday
Zonapellucidadisappears
5?6thday
Blastocystattachmenttoendometrialsurface
6?7thday
Differentiationofcytoandsyncytiotrophoblastlayers
10thday
SynthesisofhCGbysyncytiotrophoblast
9?10thday
Lacunarnetworkforms
10?11thday
Trophoblastsinvadeendometrialsinusoids
establishinguteroplacentalcirculation
Interstitialimplantationcompletedwithentire
decidualcoverage
13thday
Primaryvilli
16thday
Secondaryvilli

16thday
Secondaryvilli
21stday
Tertiaryvilli
21st?22nd
Fetalheart.Fetoplacentalcirculation
day

172.Fimbriectomyprocedureisknownas-
a)Uchidamethod
b)Irvingmethod
c)Madlenertechnique
d)Kroenermethod
CorrectAnswer-D
Ans.D.Kroenermethod
Uchidatechnique--Asalinesolutionisinjectedsubserosalinthe
midportionofthetubetocreateableb.
Irvingmethod--Thetubeisligatedoneithersideandmidportionof
thetube(betweentheties)isexcised.
Madlenertechnique-Itistheeasiestmethod.Theloopofthetubeis
crushedwithanarteryforceps.
Kroenermethodoffimbriectomyisnotacommonprocedure

173.RDAofiodineinlactationinmicrogram-
a)150
b)220
c)100
d)250
CorrectAnswer-D
Ans.D.250
Toaccommodateincreasediodineneedsduringpregnancyand
lactation,theiodineRDAis220mcg/dayforpregnantwomenand
250mcg/dayforlactatingwomen

174.WhichOneofthefollowingisnota
causeofsecondaryPostpartum
Haemorrhage?

a)Placentaprevia
b)Retainedbitsofplacenta
c)Endometritis
d)Polyp
CorrectAnswer-A
Ans.A.Placentaprevia
CausesofsecondaryPostpartumHaemorrhageare:
Retainedbitsofplacenta
Postpartuminfection
InfectionofCervicalandVaginalTears
PuerperalInversionofUterus
UterinePolyporFibroid:
Undiagnosedcarcinomaofcervix
Chorion-epithelioma

175.Besttimetodoquadrupletest
a)8-12weeks
b)11-15weeks
c)15-20weeks
d)18-22weeks
CorrectAnswer-C
Ans.C.15-20weeks
Thequadscreenisdoneinthesecondtrimester,usuallybetween
15and20weeksofpregnancy.
Ideally,thetestshouldbeperformedinconjunctionwithfirst-
trimesterscreeningtests.

176.Drugthatisusedforfetallungmaturity
is:
a)Dexamethasone
b)Folicacid
c)Beclomethasone
d)None
CorrectAnswer-A
Ans.A.Dexamethasone
Betamethasoneanddexamethasonearecorticosteroids,alsocalled
glucocorticoids,thataregivenbeforebirth(antenatally)tospeedup
apretermfetuseslungdevelopment.
Eitherisusedwhenamotherisinpretermlaborandbirthmayoccur
in24to48hours.

177.InUterineprolapsehowtoknowifringis
inplace?
a)Ifnotexpelledafterincreasedabdominalpressure
b)IfBleedingdoesnotoccur
c)Ifpatientfeelsdiscomfort
d)None
CorrectAnswer-A
Ans.A.Ifnotexpelledafterincreasedabdominalpressure
Avaginalpessaryisaremovabledeviceplacedintothevagina.
Itisdesignedtosupportareasofpelvicorganprolapse.
Avarietyofpessariesareavailable,includingtheringpessaries
Ifnotexpelledafterincreasedabdominalpressureringpessaryis
supposedtobeplacedinplace

178.HTindicatedinmenopausalwomen
a)Hotflash
b)Cabreast
c)Endometriosis
d)Uterinebleeding
CorrectAnswer-A
Ans.A.Hotflash
HormoneTherapy(HT)isoneofthegovernment-approved
treatmentsforreliefofmenopausalsymptoms.
Thesesymptoms,causedbylowerlevelsofestrogenat
menopause,include:
Hotflashes,
Sleepdisturbances,and
Vaginaldryness.
HTisalsoapprovedforthepreventionofosteoporosis.

179.Dilatation&curettage(D&C)is
contraindicatedin-
a)Pelvicinflammatorydisease(PID)
b)Endometriosis
c)Ectopicpregnancy
d)None
CorrectAnswer-A
Ans.A.Pelvicinflammatorydisease(PID)
PredisposingriskfactorsforPIDare:
Sexualcontact
HistoryofSTI
Proceduresinvolvingtheupperfemalegenitaltractincluding:
Dilatation&curettage(D&C)
Recentintrauterinedevice(IUD)insertion
Therapeuticabortion(T/A)

180.Whichofthesestepsisfollowedfirstfor
themanagementofshoulderdystocia
afterMcRobertsmaneuver?

a)Sharpflexionofhipjointstowardsabdomen
b)Suprapubicpressure
c)90degreerotationofposteriorshoulder
d)Emergencyc-section
CorrectAnswer-B
Ans.B.Suprapubicpressure
Applyingagentlesuprapubicpressureisthefirststepin
managementofshoulderdystociaafterMcRobertsmaneuver.
Mnemonic"HELPERR"asaguidefortreatingshoulder
dystocia:
"H"standsforhelp
"E"standsforevaluateforepisiotomy.
"L"standsforlegs(pullyourlegstowardyourstomachMcRoberts
maneuver)
"P"standsforsuprapubicpressure
"E"standsforentermaneuvers(internalrotationofbaby'sshoulders)
"R"standsforremovetheposteriorarmfromthebirthcanal.
"R"standsforrollthepatient.

181.Truehermaphroditismkaryotype:
a)45X0streakedgonads
b)46XXOvotestis
c)47XY+9
d)47XX
CorrectAnswer-B
Ans.B.46XXOvotestis
Truehermaphroditeorovotesticulardisorderofsexualdifferentiation
(OVO-DSD)isoneoftherarestvarietyofallintersexanomalies.
Inabout90%ofcases,patientshave46XXkaryotype.
Rarely,46XY/46XXmosaicismmayoccur.

182.Peripartumcardiomyopathyoccursat-
a)Within7days
b)Within6weeks
c)Within24months
d)Within5months
CorrectAnswer-D
Ans.D.Within5months
PPCMisastructuralheartmusclediseasethatoccursinwomen
eitherattheendofpregnancyoruptofivemonthsaftergivingbirth.
Pearson'sdiagnosticcriteriaforperipartumcardiomyopathy:
Developmentofcardiacfailureinthelastmonthofpregnancyor
within5monthsafterdelivery
Absenceofanidentifiablecauseforthecardiacfailure
Absenceofrecognizableheartdiseasepriortothelastmonthof
pregnancy,and
Leftventricularsystolicdysfunctiondemonstratedbyclassic
echocardiographiccriteria,suchasdepressedejectionfractionor
fractionalshorteningalongwithadilatedleftventricle.

183.Nervemostlycompressedinpregnancy
puerperium:
a)Radialnerve
b)Mediannerve
c)Femoralnerve
d)Facialnerve
CorrectAnswer-C
AnsC.Femoralnerve
Themostcommonpostpartumnerveinjuryisfootdropduetoinjury
totheperonealnerveandinjurytothelateralfemoralcutaneous
nerve.
Obstetriciansmayconsiderfrequentpositionchangesinlabor,
avoidanceofprolongedhipflexion,andshorteningthepushingtime
byallowingforpassivedescentofthefetusbeforepushingbegins
asmeansofavoidinglateralfemoralcutaneousnerveinjury.

184.Inpregnancywhichofthefollowinglevel
isalteredmostly:
a)TotalT3
b)FreeT3
c)FreeT4
d)TSH
CorrectAnswer-A
AnsA.TotalT3
TotalT3levelsarealteredthroughoutpregnancy.
Hormonesduringpregnancy:
Increased
Decreased
Unchanged
GrowthHormone
Luteinizinghormone ADH
Follicle-stimulating
Free
hormone
T3,FreeT4
Di-Hydro
Prolactin
EpiAndroSterone
TotalT3,TotalT4
Aldosterone
Testosterone,Androstenodione
andcortisol
Insulin(duetoinsulinresistance)
Estrogen
Progesterone
TotalT3andtotalT4increaseduetostimulationofthyroidtissueby
hCGduringtheearlyperiodofgestation.Butthereisnochangein
freeT3andT4duetoincreaseinTBG.
ThisleadstotransientfallinTSHduringanearlyweeksofgestation.

ThelevelofTSHreachesnormallevelduringsecondandthird
trimester.

185.Paget'sisassociatedwithwhichother
cancer:
a)Vulva
b)Vagina
c)Cervix
d)Uterus
CorrectAnswer-A
Ans.A.Vulva
ExtramammaryPaget'sdisease(EMPD),alsoextramammaryPaget
disease,isarare,slow-growing,usuallynoninvasiveintraepithelial
(intheskin)adenocarcinomaoutsidethemammaryglandand
includesPaget'sdiseaseofthevulvaandtheextremelyrarePaget's
diseaseofthepenis.

186.WhatismeantbySuperfecundation?
a)Fertilizationoftwoormoreovainoneintercourse
b)Fertilizationoftwoormoreovaindifferentintercoursesinsame
menstrualcycle
c)Fertilizationofovaandthenit'sdivision
d)Fertilizationofsecondovumfirstbeingimplanted
CorrectAnswer-B
Ans.B.Fertilizationoftwoormoreovaindifferentintercourses
insamemenstrualcycle
Superfecundationisthefertilizationoftwoormoreovafromthe
samecyclebyspermfromseparateactsofsexualintercourse,
whichcanleadtotwinbabiesfromtwoseparatebiologicalfathers.
Thetermsuperfecundationisderivedfromfecund,meaningthe
abilitytoproduceoffspring.

187.Fetalheartstartscontractingat-
a)10-12days
b)10-12weeks
c)3-5weeks
d)3-5month
CorrectAnswer-C
Ans.C3-5weeks
Thefetalheartstartscontractingatapproximately23daysof
gestation.

188.

Anesthesiaofchoiceforcesareansectionin
severepre-eclampsia:

a)Spinal
b)GA
c)Epidural
d)Spinal+epidural
CorrectAnswer-C
Ans.C.Epidural
ContinuousEpiduralAnesthesiaisthefirstchoiceforpatientswith
preeclampsiaduringlabour,Vaginaldeliveryandcesareansection.
Preeclampsiapatientshaveariskofsevereairwayedema,which
makesintubationdifficult
ContinuousEpiduralAnesthesiacanimproveuteroplacental
perfusionandalsodecreasecatecholaminesecretions.

189.Whichofthefollowingisnotahigh-risk
pregnancy?
a)Previoushistoryofmanualremovalofplacenta
b)Anemia
c)Diabetes
d)Obesity
CorrectAnswer-A
Ans.A.Previoushistoryofmanualremovalofplacenta
HighRiskPregnancyisseenin:
Cardiovascularandrenalfactors:
Moderatetoseverepreeclampsia
Chronichypertension
Severeheartfailure(classII-IV,NYHAclassification)
Moderatetosevererenaldisorders
Metabolicdisorders:
Obesity
Insulin-dependentdiabetes
Previousendocrineablation
Obstetrichistory:
FetalexchangetransfusionbecauseofRhincompatibility
Stillbirth
Lateabortion(16?20wk)
Post-termpregnancy(>42wk)
Pretermnewborn(<37wkand<2500g)
Intrauterinegrowthrestriction(weight<10thpercentileforestimated
gestationalage)
Polyhydramnios
Multifetalpregnancy

Previousbrachialplexusinjury
Abnormalfetalposition
Otherdisorders:
Abnormalcervicalcytologicfindings
Sicklecelldisease
Severeanemia
Thrombophilia
Autoimmunedisorders
Anatomicabnormalities:
Uterinemalformation
Insufficient(incompetent)cervix
Exposuretoteratogens:
GroupBstreptococcalinfections
Smoking>10cigarettes/day(associatedwithprematureruptureof
membranes)
Currentpregnancycomplications:
Pretermlaborat<37Wks
Pretermprematureruptureofmembranes
Moderatetoseverepreeclampsia
Polyhydramnios(hydramnios)oroligohydramnios
Post-term(>42wk)
Uterinerupture
Placentaprevia
Abruptioplacentae
Chorioamnionitis
Prolapsedcord
Fetalbradycardiafor>30min
Fetaltachycardiafor>30min
Fetalweight<2.5kg
Fetalweight>4kg
FetalacidosispH7

190.Whichofthefollowingisnotusedin
preeclampsia?
a)Methyldopa
b)Atenolol
c)Labetalol
d)Hydralazine
CorrectAnswer-B
Ans.B.Atenolol
Choiceofanti-hypertensivesinpregnancy
PIH/pre-eclampsia/severepre-eclampsia:Labetalol
AcuteHypertension/Hypertensioncrisis:IVlabetalol>Nifedipine
>Hydralazine
ChronicHypertensioninpregnancy:Labetalol>Methyldopa.

191.Cutismarmorataoccursduetoexposure
to?
a)Coldtemperature
b)Dust
c)Hottemperature
d)Humidity
CorrectAnswer-A
Ans.A.Coldtemperature
Cutismarmorataisaconditionwhereintheskinhasapinkishblue
mottledormarbledappearancewhensubjectedtocold
temperatures.
Itisseenthroughoutinfancyandin50%ofchildren.
Rewarmingrestorestheskintonormal.
Itiscausedbysuperficialsmallbloodvesselsintheskindilatingand
contractingatthesametime.

192.Murphy'seyeisseenin-
a)Macintoshlaryngoscope
b)Endotrachealtube
c)LMA
d)Flexiblelaryngoscope
CorrectAnswer-B
Answer:B-EndotrachealTube
The"Murphyeye"istheeponymousnameforaholeonthesideof
mostendotrachealtubes(ETTs)thatfunctionsasavent,and
preventsthecompleteobstructionofthepatient'sairway,shouldthe
primarydistalopeningofanETTbecomeoccluded

193.ModifiedMallamPatigradingisusedin
assessmentof-
a)Difficultintubation
b)Airwayobstruction
c)Deathduetoaspiration
d)Intubation
CorrectAnswer-A
Answer:A-Difficultyinintubation
ModifiedMallampaticlassification
Class0:Abilitytoseeanypartoftheepiglottisuponmouthopening
andtongueprotrusion
ClassI:Softpalate,fauces,uvula,pillarsvisible
ClassII:Softpalate,fauces,uvulavisible
ClassIII:Softpalate,baseofuvulavisible
ClassIV:Softpalatenotvisibleatall
Test:Theassessmentisperformedwiththepatientsittingup
straight,mouthopenandtonguemaximallyprotruded,without
speakingorsaying"ahh."
Difficultlaryngoscopy:Goodaccuracy(areaunderSummary
ReceiverOperatingCharacteristic[SROC]curve0.89?0.05)
Difficultintubation:Goodaccuracy(areaunderSROCcurve0.83
?0.03)
Difficultmaskventilation:Poorpredictor
Usedalone,theMallampatitestshavelimitedaccuracyfor
predictingthedifficultairwayandthusarenotusefulscreeningtests
Mallampaticlassificationisonlyoneof11nonreassuringfindings
duringairwayexamination

194.Whichnerveistestedforadequacyof
anaesthesia-
a)MedianNerve
b)UlnarNerve
c)Radialnerve
d)Mandibularnerve
CorrectAnswer-A
Answer:A-Mediannerve
Mediannerveblockcanbeevaluatedbytestingthelateralaspectof
theringfinger

195.Mosteffectivecircuitinspontaneous
anaesthesiais-
a)MaplesonA
b)MaplesonB
c)MaplesonC
d)MaplesonD
CorrectAnswer-A
Answer:A-MaplesonA
MAPLESONA-(Magill)CIRCUIT
Usefulinspontaneousventilation
Thepatientinspireswhateverisinthetube,usingthebagasa
volumereservoir.
Oneatedtubeandincomingfreshgas.Whenthebagisfull,exhaled
alveolargasisventedfrxpiration,thebagrefillsfromacombination
ofexpiredgasgoingbackupthecorrugomtheexhalevalve,and
thenduringanyexpiratorypause,FGFpushestheremaining
alveolargasout.
TheoreticallyFGF=0.7xValvshouldpreventsignificantrebreathing
becausedeadspacegas(fresh)isnotwasted,butFGF=VAmore
reliablypreventsrebreathing.
Tubevolumemustexceed(Vt-Vd)oralveolargascouldcontaminate
thebag.
InadequateFGFcausesrebreathing.Difficulttodetectfromthe
CO2waveformalone-allthathappensisthattherapidfallon
inspirationisdelayed.IfVAexceedstubingvolume,CO2entersthe
bagandwillbeseenoninspirationonthecapnogram.
Controlledventilation
Iftheanaesthetistfullyclosedthevalvewhilesqueezingthebagand

didn'topenituntiljustbeforethebagfilled,thiscircuitwouldbeOK.
Morecommonlythevalveispartiallyclosed-enoughtopermit
adequatetidalvolumesdespiteparallellossofgasoutthevalve.
FGFmustbeincreasedtocompensateforgaslostduringinspiration
-typically2.5xminuteventilation.
TheLacksystem
Aco-axialMagill,withtheexpiratoryvalvebroughtcoaxiallybackto
theFreshGasoutlet.Notpopularduetoinefficiencyduring
controlledventilation.

196.Whatismechanismofactionof
Curaniumdrugsasmusclerelaxant?
a)PersistentlydepolarizingatNeuromuscularjunction
b)ActcompetitivelyonAchreceptorsblockingpost-synaptically
c)RepetitivestimulationofAchreceptorsonmuscleendplate
d)Inhibitingthecalciumchannelonpresynapticmembrane
CorrectAnswer-B
Answer:B-ActcompetitivelyonAchreceptorsblockingpost-
synaptically
Candocuroniumiodideisanaminosteroidneuromuscular-blocking
drugorskeletalmusclerelaxantinthecategoryofnon-depolarizing
neuromuscular-blockingdrugs.
ActsonAchreceptorscompetitivelypost-synapticallyblockingthem.
Potentialadjunctiveuseinanaesthesiatofacilitateendotracheal
intubation&provideskeletalmusclerelaxation.
Candocuroniumdemonstratedashortdurationandarapidonsetof
action,withlittleornoganglionblockingactivity,anditwasonly
slightlylesspotentthanpancuronium

197.Dyeusedindiagnosisofesophageal
perforation:
a)Iohexol
b)Bariumsulphate
c)Gadolinium
d)Iodinedye
CorrectAnswer-B
Answer-B.Bariumsulphate
Bariumsulfateinsuspensionisfrequentlyusedmedicallyasa
radiocontrastagentforX-rayimagingandotherdiagnostic
procedures.
ItismostoftenusedinimagingoftheGItractduringwhatis
colloquiallyknownasa"bariummeal".
Fluoroscopy
mostsensitivewithinthefirst24hours.
patientexaminedsemi-supine(~20degrees)onfluoroscopytable
awater-solubleagentshouldbeusedinitiallyasbariumcancause
mediastinitis
esophagealperforationmayberepresentedasmucosalirregularity
orgrossextraluminalcontrastextravasation
someauthorssuggesttheuseofsmallamountsofloworhigh
concentrationsofbariumifnoleakisevidentoninitialscreeningwith
watersolublecontrast
Iohexol
,tradenamesOmnipaqueamongothers,isacontrast
agentusedduringX-rays.Thisincludeswhenvisualizingarteries,
veins,ventriclesofthebrain,theurinarysystem,andjoints,aswell
asduringcomputertomography.Itisgivenbymouth,injectionintoa
vein,orintoabodycavity.



198.Braggpeakeffectpronouncedin:
a)Xray
b)Proton
c)Neutron
d)Electron
CorrectAnswer-B
Answer-B.Proton
Nowadaysprotontherapy(PT)representsanestablishedalternative
tophotonradiotherapyforthetreatmentofspecifictypesofcancer.
Thetherapeuticuseofprotonbeams(andofchargedparticlesin
general)ismotivatedprimarilybytheirinverteddepth-doseprofile
comparedtophotons,beingcharacterizedbytheso-calledBragg
peak.
Severalbeamsofdifferentenergycanthenbecombinedinorderto
achievetheprescribeddoseinaregionaslargeasthetarget
volume,resultingintheproductionofwhatisdefinedastheSpread-
OutBraggPeak(SOBP).


199.SaltandPepperpotappearanceofskull
seenin:
a)Hyperparathyroidism
b)Multiplemyeloma
c)Hyperthyroidism
d)Pseudohyperparathyroidism
CorrectAnswer-A
Answer-A.Hyperparathyroidism
Pepperpotskull
isoccasionallyusedinplaceofsaltandpepper
skulltodescribethetypicalradiographicappearanceofmultiple
smallradiolucentlesionsoftheskullvault.
Inprimaryhyperparathyroidism,extensiveresorptionboneinthe
skullincombinationwithcysticareasofosteopeniaare
termedpepperpotskull.
Classicallyseeninhyperparathyroidism,andisoccasionallyused
(inaccurately)todescribetheraindropskullofmultiplemyeloma.
:SullX-raywithatypical"pepper-pot"appearance

200.ImagingtechniquesusedinUterus
anomaliesEXCEPT:
a)HSG
b)MRIguidedHSG
c)CTguidedHSG
d)USG
CorrectAnswer-C
AnswerC.CTguidedHSG
Imagingstudies,suchasahysterosalpingogram
(HSG)
andultrasound,oranMRIarerequiredtovisualisethe
uterusandconfirmthatacongenitaluterineanomalyispresent.
Ahysterosalpingogramisnotconsideredasusefulduetothe
inabilityofthetechniquetoevaluatetheexteriorcontourofthe
uterusanddistinguishbetweenabicornuateandseptateuterus.
Inaddition,laparoscopyand/orhysteroscopymaybeindicated.

201.Semensqueeze
a)Erectiledysfunction
b)Prematureejaculation
c)Retrogradeejaculation
d)Antegradeejaculation
CorrectAnswer-B
Answer:B-PrematureEjaculation
SqueezeTechnique:
VariationoftheMastersandJohnsonmethod.
Asamanapproachesclimax,eitherheorhispartnersqueezesthe
tipofthepenisjustbelowtheheadofthepenisasheapproaches
thepointofclimax.
Pressureisheldthereuntilthesensationofimpendingorgasm
diminishes.
Thispressurecanevenbehelduntilthereissomereductionin
erection.
Theprocesscanthenbestartedoveragainsothatovertimeaman
prolongsthetimeperioduntilhereachesejaculation.
Usefultotreatcasesofprematureejaculation

202.ApatientwithahistoryofRTAbefore2
monthspresentswithcomplaintsof
dreamsofaccidents.Heisableto
visualizethesamescenewheneverhe
visitstheplace.Henceisafraidtogo
backtotheaccidentsite.Identifythe
typeofdisorderthathemightbe
sufferingfrom?

a)Adjustmentdisorder
b)PTSD
c)Anxietydisorder
d)OCD
CorrectAnswer-B
Answer:B-Post-traumaticStressdisorder
Post-traumaticstressdisorder,basicallyatypeofanxiety
disorder
Thoughitsharesfeatureswithotheranxietydisorders,hasitsown
specificcharacteristicsinpresentation.
Criteria1:
Etiologicallysignificanttraumashouldbepresenttobeclassified
underthiscondition.
Criteria2:
"Intentionality"or"aboutness",animportantfactorinPTSD.
PTSDconcernswithmemoryintrusionofpaststressorsintopresent.
Nightmares,flashbacks,orrelivingexperiencesshouldberelatedto

thepastexperience.
Criteria3:
Avoidastimulusoractivitythatprovokesthememoryofpastevent.

203.Freud'stheoryofdreamincludesall
except:
a)Displacement
b)Condensation
c)Symbolisation
d)Correlation
CorrectAnswer-D
Answer:-D-Correlation
SigmundFreudtheoryofdream:
Dreamworkinvolvestheprocessofcondensation,displacement,
andsecondaryelaboration.
Conceptofunconsciousmind:
PrimaryassumptionofFreudiantheoryisthattheunconsciousmind
governsbehaviortoagreaterdegreethanpeoplesuspect.
Goalofpsychoanalysisistomaketheunconscious,conscious.
Theoryofdream:
Freudtheoryofdreamelaboratesthestateofunconsciousmindwith
respecttodreaminterpretation,
Processinvolvedincludecondensation,displacement,and
secondaryelaboration.
Displacementtakesplacewhenwetransformthepersonorobject
wearereallyconcernedabouttosomeoneelse.
Theprocessofcondensationisthejoiningoftwoormore
ideas/imagesintoone.

204.Expressionandconsequentreleaseof
previouslyrepressedemotioniscalled
as

a)Regression
b)Dissociation
c)Abreaction
d)Alloftheabove
CorrectAnswer-C
Answer:C-Abreaction
Theexpressionandconsequentreleaseofapreviouslyrepressed
emotion,achievedthroughrelivingtheexperiencethatcausedit.
Donetypicallythrough
Hypnosis
Suggestion

205.Allarehabitdisorderexcept
a)Nailbiting
b)Thumbsucking
c)Tempertantrum
d)Tics
CorrectAnswer-C
Answer:C-Tempertantrum
Definition:
Habitdisorderisatermusedtodescribeseveralrelateddisorders
linkedbythepresenceofrepetitiveandrelativelystablebehaviour
thatseemtooccurbeyondtheawarenessofthepersonperforming
thebehaviour.
Thefirstgroupofhabitdisorderincludes"TicDisorder"
Ticsareinvoluntarymovements,sounds,orwordsthataresudden,
rapid,recurrentandnon-rhythmic"
InadditiontoTD's,body-focussedbehaviours,
Recurrenthairpulling-Trichotillomania(TTM)
Skinpicking(SP)
Nailbiting,areincludedwithinhabitdisorders.
Headbanging,rockingofbody,teethgrinding&thumbsucking
arerepetitivedisorders.
Seeninchildrenbetweenagegroupof6monthsto2years.
Benign&self-limited.
Themovementsserveameansoftensiondischarginginchildren.
Aschildrenbecomeolder,theylearntoinhibitsomeoftheirrhythmic
patterns.
Undueattentionbyparentscanleadtoaggravationofthese
problems.


206.Newnameofmentalretardation
accordingtoAmericanAssociationof
MentalRetardation

a)FeebleMindedness
b)Madness
c)Intellectualdisability
d)Mentallyunstable
CorrectAnswer-C
Answer:C-Intellectualdisability
Intellectualdisability(ID),alsoknownasgenerallearningdisability,
Mentalretardation(MR),isageneralizedneurodevelopmental
disordercharacterizedbysignificantlyimpairedintellectualand
adaptivefunctioning.

207.Now-a-daysDownsyndromeIsreferred
toas.
a)Submentaldisorder
b)Oligophrenia
c)Madness
d)Mentallyunstable
CorrectAnswer-A
Answer:A-Submentaldisorder
MajorityofchildrenwithDownsyndromefunctioninmildto
moderaterangeofmentalretardation.
Downsyndrome/Trisomy21-
Geneticdisordercausedbypresenceofallorpartofathirdcopy
ofchromosome21.
Typicallyassociatedwithphysicalgrowthdelays,characteristicfacial
featuresandmildtomoderateintellectualdisability.

208.A55yearsagedchronicalcoholicmale,
presentedwithirrelevanttalks,tremor
andsweating.Hehadhislastdrink3
daysback.Whatwilltheprobable
diagnosis?

a)Deliriumtremens
b)Korsakoffpsychosis
c)Post-Acutewithdrawalsyndrome
d)Discontinuationsyndrome
CorrectAnswer-A
Answer:A-DeliriumTremens
Thedescriptionofsymptomsisrelatedtothecondition"Delirium
Tremens"
DeliriumTremens/AlcoholWithdrawalDelirium(AWD):
Mostsevereformofethanolwithdrawalmanifestedby,
Alteredmentalstatus(Globalconfusion)
Autonomichyperactivity(Sympatheticoverdrive)
Mechanism:
Alcoholabuseaffectsneurotransmittersystemsinbrainmainlyby,
LossofGABAinhibitorymechanism-Reduceschlorideion
influx.
AlcoholactsasNMDAreceptorantagonist-Withdrawalincreases
theexcitatoryneurotransmitter.
Theclinicalmanifestationsofethanolwithdrawalarecombination
effectsofGABA&NMDAreceptoractivity.
Thuscausingtremors,diaphoresis,tachycardia,anxiety&in

severecasesSeizure.


209.Identifythetypeofmuscleshowninthe
imagebelow.
a)Cruciate
b)Multipennate
c)Parallel
d)Unipennate
CorrectAnswer-B
Answer-B.Multipennate
Themuscledepictedaboveisthepectoralismajor.
PectoralisMajorhasmultiplerowsofdiagonalfibreswithasingle
tendon,Henceisamultipennatemuscle.


210.Functionofthemuscleshowninpicture:
a)Flexion
b)Extension
c)Adduction
d)Abduction
CorrectAnswer-A
Answer-A.Flexion
Lumbricals
arefourshorthandmuscleslocatedinthemetacarpus
deeptothepalmarfascia.
Thelumbricalsareintrinsicmusclesofthehandthatflexthe
metacarpophalangealjointsandextendtheinterphalangealjoints.
Thelumbricalsareusedduringanupstrokeinwriting.


211.Followingimageisalsoknownas?
a)5-hydroxytryptamine(5-HT)
b)N-methylphenylamine
c)3-Methoxytyramine
d)Phenethylamine
CorrectAnswer-A
Ans:A.5-hydroxytryptamine(5-HT)
Serotonin/5-hydroxytryptamine(5-HT)isamonoamine
neurotransmitter.
Followinghydroxylationoftryptophanto5-hydroxytryptophanby
livertryptophanhydroxylase,subsequentdecarboxylationforms
serotonin(5-hydroxytryptamine),apotentvasoconstrictorand
stimulatorofsmoothmusclecontraction.


212.Choosethebestmethodofdiagnosisfor
theclinicalsignrepresentedinthe
image.

a)Serumcopper
b)Serumceruloplasmin
c)Karyotyping
d)PCR
CorrectAnswer-B
Answer:B-SerumCeruloplasmin
Imagerepresents"Kayser-Fleischer"ring
KFringisgoldenbrownringduetodepositionofcopperin
Descement'smembraneofcornea.
Clinicalfeatureof"Wilson'sdisease"
Wilson'sDisease:
Rareautosomalrecessivedisease.
Characterizedbyabnormalcoppermetabolism
Ophthalmicpresentation:
Depositionofcopperinposteriorcapsuleoflensresultsinsunflower
cataract.
Diagnosis:
Sternlieb'scriteria:


PresenceofKFrings
Decreasedserumceruloplasmin(coppercontainingenzyme/
ferroxidase)levels
Measuringhepaticcopperlevelsinliverbiopsy
24-Hrurinecopperexcretion
KayserFleischerring:
Excesscirculatingcopperdepositsindescemet'smembrane.
Usuallygoldenbrownlocatedinperipheralcornea,beginningat
schwalbe'slineupto5mmintocornea.
DetectedviaGonioscopyinearlierstage&seenbynakedeyesin
advancedstage.


213.Identifytheconditionrepresentedinthe
image.
a)Seminoma
b)Germcelldifferentiatetumor
c)Non-seminoma
d)Teratoma
CorrectAnswer-A
Answer:A-Seminoma
Seminoma-Germcelltumoroftesticle/rarelymediastinumorother
extra-gonadallocations.
Malignantneoplasm;
Oneofmosttreatable&curablecancers;
Survivalrate>95%ifdiscoveredinearlystages.
Usuallyunaffectedfertility&othersexualfunctionsremainsintact.
Originatesingerminalepitheliumofseminiferoustubules
Histology:
Typicallyprominentlymphocyticinfiltrateinthefibrousstroma
separatingtheclustersoftumorcells.
Treatment:
Requiresremovalofonetesticle.


214.Identifytheconditionshowninthe
image.
a)LobarPneumonia
b)Bronchopneumonia
c)Acuteglomerulonephritis
d)Congestedkidney
CorrectAnswer-A
Answer:A-LobarPneumonia
Imagedepictstheredhepatizationoflungs
Redhepatizationisseeninconditionslikelobarpneumonia.
LobarPneumonia:.
Pneumococcalpneumonia-Mostcommoncauseoflobar
pneumonia
Progressesfromaredhepatizationphasetoagrayhepatization
phase
Redhepatization:
Characterizedbyconsolidationofairspacesoflungs.
C/Srevealslungsappearbrown-red,firm,&airless.resemblingliver.

Histological:
Congestedalveolarcapillaries&alveolarspacesarefilledwith
erythrocytes,neutrophils,andfibrin.
Redcellsdisintegrate,withpersistenceoftheneutrophilsandfibrin.


215.Identifytheconditionrepresentedin
belowimage
a)Miliarytuberculosis
b)Bronchiectasis
c)COPD
d)Lungcancer
CorrectAnswer-A
Answer:A-Miliarytb
Miliarytuberculosis(TB)iswidespreaddisseminationof
Mycobacteriumtuberculosis
Resultofhematogenousspread.
Distinctivepatternseenonachestradiograph
Tinysizedlesions(1?5mm),seenastinyspotsdistributed
throughoutlungfieldsappearingsimilartomilletseeds
Hence,theterm"miliary"tuberculosis.
MiliaryTBmayalsoinfectliver&spleen.


216.Identifytheorganismrelatedtoblood
smearimage-
a)P.falciparum
b)S.Typhi
c)Treponemapallidum
d)Toxoplasmagondii
CorrectAnswer-A
Answer:A.-->P.falciparum
Malariaparasitecanbeseeninthebloodsmearofpatientsaffected
bythediseaseatspecificstagesinthedisease.
Plasmodiumfalciparum--ring-shapedtrophozoite.Theredbloodcell
containingfourring-shapedtrophozoites.
Notetheveryhighpercentageofredcellscontainingringforms.
Thishigh-levelparasitemiaismoreoftenseeninPlasmodium
falciparuminfectionthanininfectionbytheotherplasmodia.
Thethicksmearsallowdetectionwhereasthethinsmearsallow
identificationofthespecies.Thisthinsmearshowsmultiplemalaria

trophozoitesinsideredbloodcells(Earlyringform).However,
trophozoitescanalsobeseenoutsideredbloodcells.


217.Whichisthickenednerveshownhere:
a)FacialNerve
b)Greaterauricularnerve
c)VagusNerve
d)GlossopharyngealNerve
CorrectAnswer-B
Answer-B.Greaterauricularnerve
Thegreaterauricularnerveisacutaneousbranchofthecervical
plexusthatinnervatestheskinoftheauricleaswellasskinoverthe
parotidglandandmastoidprocess.Thegreaterauricularnervealso
suppliesbranchesthatinnervatethedeeplayeroftheparotidfascia.
Origin
ThegreaterauricularnervearisesfromtheventralramiofC2and
C3spinalnerves,althoughitreceivesconsiderablymorefibresfrom
C2.
Course



Thegreaterauricularnerveemergesalongtheposterioraspectof
thesternocleidomastoidmuscleatthepunctumnervosum(Erb
point)andascendsverticallyacrosstheobliquesternocleidomastoid
muscle.Whenthegreaterauricularnerveapproachestheinferior
poleoftheparotidglanditdividesintoanteriorandposteriorterminal
branches.


218.Whichtypeofretractorisshowninthe
image
a)Morrisretractor
b)Czernyretractor
c)Richardsonretractor
d)Lowerlidretractor
CorrectAnswer-A
Answer:A.Morrisretractor
Morrisretractor-
Thisisabigretractorwhichisusefulforgivingmaximumexposure
inlargeincisionssuchasthoseusedintheabdomen.
ltcanbeusedtoimprovevisibilityononesideofanincision(by
puttingfirmlyinthatdirection);soisvaluableduringtheinitialphase
ofalaparotomy.
SirHenryMorris(1844-1926)wasasurgeonattheMiddlesex
Hospital.London,UK.


219.Whichofthefollowingstatementistrue
aboutsuturematerialintheimage:
a)Madeofrabbitsubmucosa
b)Madeofcatsubmucosa
c)Notdegraded
d)Degradedbyenzymaticdegradation
CorrectAnswer-D
Answer:D.Degradedbyenzymaticdegradation
Catgut-
Types-plain
Rawmaterial-Collagenderivedfromhealthysheeporcattle
Tensilestrengthretentioninvivo-Lostwithin7-10days.Marked
patientvariability.Unpredictableandnotrecommended.
Absorptionrate-Phagocytosisandenzymaticdegradationwithin7-
10days
Tissuereaction-high
Contraindications-Notforuseintissuesthathealslowlyandrequire
prolongedsupport.Syntheticabsorbableissuperior.
Uses-Ligatesuperficialvessels,suturesubcutaneoustissues.
Stomasandothertissuesthathealrapidly.


220.APatienthashistoryofRTA2years
back,atthesamesighthedeveloped
painandswelling.Xrayshowsthe
followingfeatures.Whatwillbethe
diagnosis?

a)Osteogenicsarcoma
b)Ewing'ssarcoma
c)Chronicosteomyelitis
d)Multiplemyeloma
CorrectAnswer-C
Ans.C.Chronicosteomyelitis
Garr?sclerosingosteomyelitis,orchronicnonsuppurativesclerosing
osteomyelitis,isaformofchronicosteomyelitis.
Mildinflammationandinfectionleadtosubperiostealbone
deposition.
Thediseaseisfrequentlyasymptomatic.
Thecharacteristicradiographicappearanceisanareaofperiosteal


proliferationsurroundedbysuccessivelayersofcondensedcortical
bone(arrows),describedasanonionskinappearance.


221.IdentifythebonenumberedintheX-ray
belowthatmostcommonlyfracture
whenapersonfallsonoutstretched
hands?

a)1
b)2
c)3
d)4
CorrectAnswer-A
Ans.A.1
Adistalradiusfracture,alsoknownasawristfracture,isabreakof
thepartoftheradiusbonewhichisclosetothewrist.
Symptomsincludepain,bruising,andswellingofrapidonset
Thewristmaybedeformed.
Inyoungerpeople,thesefracturestypicallyoccurduringsportsora
motorvehiclecollision.Inolderpeople,themostcommoncauseis
fallingonanoutstretchedhand.


fallingonanoutstretchedhand.
SpecifictypesincludeColles,Smith,Barton,andHutchinson
fractures


222.IdentifytheconditionshownintheCT
Scanimagebelow.
a)Dandywalkermalformation
b)Cerebellarvermishypoplasia
c)Megacisternamagna
d)None
CorrectAnswer-A
Ans.A.Dandywalkermalformation





223.IdentifytheXrayHSGShownbelow:
a)Septateuterus
b)Uterusdidelphys
c)Unicornuateuterus
d)Bicornuateuterus
CorrectAnswer-C
Ans.C.Unicornuateuterus


224.InawomancomplainingofAUB
followingimagewasseeninendoscopic
examinationofuterus.Whatwillbethe
diagnosis?

a)Leiomyoma
b)Adenomyosis
c)Ovarianneoplasm
d)Carcinomaofuterus
CorrectAnswer-A
Ans.A.Leiomyoma
Leiomyomaisthemostcommonpelvictumorinwomen
Benign,originatefrommyometrialsmoothmuscles
Symptomsinclude:
AUB
Pelvicpainandpressure
Infertilityoradversepregnancyoutcome


225.Identifythefollowinglesion.
a)Beckernevus
b)Hypopigmentedmacule
c)Spitznevus
d)Epidermalnevus
CorrectAnswer-A
Ans.A.
BeckerNevus:
Usuallystartsinadolescenceasanirregularsmooth
hyperpigmentedmacule.
Usuallyinvolvestheshoulder,anteriorchestandscapularregion.
Slowlygrowsinsizeofapalmwileacquiringthickdarkhair.
Oftenlesionresemblingacnevulgarisindifferentstagesmayappear
onthesurface.
Notreatmentisrequired.


226.Achildhasarashasshowninthe
picture.Hisfamilyhistoryispositivefor
asthma.Whatcouldbethemost
probablediagnosis?

a)Seborrheicdermatitis
b)Atopicdermatitis
c)Allergiccontactdermatitis
d)Erysipelas
CorrectAnswer-C
Ans.C.Allergiccontactdermatitis
ACDisaformofcontactdermatitisthatmanifestsasanallergic
responsecausedbycontactwithasubstance.Itsahypersensitive
reactioncharacterisedbythepresenceofrashoraskinlesioninthe
formofpapules,blistersorvesiclesetc.Thedifferentialfeatures
are
Itisconfinedtotheareawherethetriggertouchedthezone,
Itoccursafteradayortwooftheexposureand
Thesymptomsreappearwhenincontactwiththeallergen.

Thesymptomsreappearwhenincontactwiththeallergen.


227.Spotradiographfromadoublecontrast
esophagram.Imagerepresents:
a)Esophagealatresia
b)Esophagealstenosis
c)Felineoesophagus
d)Tracheoesophagealfistula
CorrectAnswer-C
Answer-C.Felineoesophagus
Inthegivenimage,therearenumerous1-2mmradiolucentfoldsacrosstheesophagus.
Thefoldsareangledconcerningtothecenteroftheesophagusina"herringbone"pattern.
Thefoldsoccurtransiently.
Thefelineesophagusalsoisknownasanoesophagealshiver,referstothetransient

transversebandsseeninthemidandloweresophagusonadouble-contrastbarium
swallow.
Thesefinemucosalfoldsareatransientfindingproducedbycontractionofthemuscularis
mucosa.Asimilarappearancemaybeseenincats.Itisusuallyanormalvariantbutmay
beassociatedwithGERD.
Thesefinefoldsareprobablyofnosignificanceandrelatetothetechniqueastheyare
mostandrelatetothetechniqueastheyaremostoftenseenwhentheesophagusinonly
partiallydistended.
Radiographicfeatures
Thefoldsare1-2mmthickandrunhorizontallyaroundtheentirecircumferenceofthe
oesophageallumen.Thefindingsaretransient,seenfollowingrefluxandnotduring
swallowing.Theappearanceisconfinedtothedistaltwo-thirdsofthethoracicesophagus.


228.MRIofskullrepresents:
a)VeinofGalen
b)DandywalkerSyndrome
c)Pneumocephalus
d)Crouzonsyndrome
CorrectAnswer-A
AnswerA.VeinofGalen
TheveinofGalenislocatedunderthecerebralhemispheresand
drainstheanteriorandcentralregionsofthebrainintothesinuses
oftheposteriorcerebralfossa.
TheveinofGalen,alsoknownasthegreatcerebralveinorgreat
veinofGalen,
isashorttrunkformedbytheunionofthetwo
internalcerebralveinsandbasalveinsofRosenthal.Itliesinthe
quadrigeminalcistern.Itcurvesbackwardandupwardaroundthe
posteriorborderofthespleniumofthecorpuscallosumtodraininto


theconfluenceoftheinferiorsagittalsinusandtheanteriorextremity
ofthestraightsinus.
AneurysmalmalformationsofVeinofGalenMalformations(VGAM)
typicallyresultsinhighoutputcongestiveheartfailureormay
representdevelopmentaldelay,hydrocephalusandseizures.
VGAMresultsfromaneurysmalmalformationwitharteriovenous
shuntingofblood.


229.Whatisadiagnosisbasedonagiven
image:
a)Uterusdidelphys
b)BicornuateUterus
c)UnicornuateUterus
d)Septateuterus
CorrectAnswer-C
Answer-C.UnicornuateUterus
Aboveimageisunicornuateuterusasseenona
hysterosalpingogramrepresents:
ThisresultsfromfailureofnormaldevelopmentofoneM?llerianduct
andisassociatedwithincreasedspontaneousabortionandobstetric
complications,andrenalabnormalities.
OnT2-weightedMRimages,theunicornuateuterusdemonstratesa
curved,elongateduteruswithtaperingofthefundalsegmentoff

midline(the`banana-like'configuration)bestseenontheaxial
oblique(longaxis)images.


230.X-rayofskullshowingwhichlesionsin
thebrain:
a)Paget'sdisease
b)Multiplemyeloma
c)Osteosarcoma
d)Osteomyelitis
CorrectAnswer-A
Answer-A.Paget'sdisease
Pagetdiseaseofthebone
isacommon,chronicbonedisorder
characterizedbyexcessiveabnormalboneremodeling.Itfrequently
affectsthepelvis,spine,skullandproximallongbonesandhas
characteristicradiographicfeatures.
Radiographicfeatures
Theconditionisfoundmostcommonlyinthesacrumandlumbar
spine,followedbyskull,pelvis,andfemur.


Intheskull,thediseasebeginsasadestructiveprocessaffecting
theoutertableandsparingtheinnertable.
Thefullpictureofosteoporosiscircumscriptaisrarelyseen.Inthe
reparativestage,sclerosisoftheinnertableispronounced,andlater
thediploicspacesandtheoutertablebecomethickened.
Aclassical,widespread'cottonwool'effectresults.
Thecranialcavityisnotencroachedupon.


231.CTofThoraxrepresents:
a)AscendingAorticdissection
b)DescendingAorticdissection
c)Aorticaneurysm
d)Cysticfibrosis
CorrectAnswer-B
AnswerB.DescendingAorticdissection
Aorticdissectionisadevastatingdiseasethatoccursasaresultof
thedegenerationofthethoracicaorta.
Middle-agedmenarethemostoftenaffectedwithonly5%of
dissectionsoccurringintheunder40agegroup.
Theclassicalpresentationisthesuddenonsetofarazor-sharppain
betweenshoulderblades.
Therearetwomajortypesofclassification,theDeBakeyandthe


Stanford.
TheStanfordclassificationissimplerwithtwotypesof
dissectiondescribed,TypeAandTypeB.TypeAisequivalenttoa
DeBakeyTypeIorType11dissection,involvingtheascending
arch,whereasTypeBjustinvolvesthedescendingaorta.TypeAis
generallytreatedwithurgentsurgerywithTypeBgenerallybeing
treatedconservatively.
Signonchestradiograph-
Themostcommonabnormalityisthewideningofthemediastinum
whichisoftenassociatedwithanindistinctoutlineoftheaortaoran
irregularwavycontour.
Morecharacteristicbutalessfrequentradiologicalfindingis
localizeddilatationoftheaorticknuckleandupperdescendingaorta,
givingrisetoaprominenthumpsignwhichwillindicateaneurysmal
dilatationorunfolding
ofthearch.
TheCTscanwillshowthetrueandfalselumensassociatedwith
dissection.


232.CTscanofabdomenshowinganarea
thatbranchingintotheliver.Identifythe
structure?

a)SVC
b)IVC
c)Portalvein
d)Splenicvein
CorrectAnswer-C
Ans:C.Portalvein





Theportalveinorhepaticportalveinisabloodvesselthatcarries
bloodfromthegastrointestinaltract,gallbladder,pancreas,and
spleentotheliver.
TheliverisfurthersubdividedintoCouinaudsegmentsbasedonthe
vascularsupply.
ThecaudatelobeorsegmentIhaveautonomousbloodsupplyfrom
bothleftandrightbranchesoftheportalveinandhepaticartery
alongwithindependentvenousdrainagedirectlyintotheIVC.


233.Identifyartery`X'inthegiven
angiographyanatomyimage:
a)Superiormesentericartery
b)Subclavianartery
c)Celiacartery
d)Brachiocephalicartery
CorrectAnswer-A
Answer?A.Superiormesentericartery
Thesuperiormesentericartery(SMA)isamajorarteryofthe
abdomen.Itarisesfromtheabdominalaorta,andsuppliesarterial
bloodtotheorgansofthemidgut?whichspansfromthemajor
duodenalpapilla(oftheduodenum)totheproximal2/3ofthe
transversecolon.

AnatomicalPosition
Thesuperiormesentericarteryisthesecondofthethreemajor
anteriorbranchesoftheabdominalaorta(theothertwoarethe
coeliactrunkandinferiormesentericartery).Itarisesanteriorlyfrom
theabdominalaortaattheleveloftheL1vertebrae,immediately
inferiortotheoriginofthecoeliactrunk.
Afterarisingfromtheabdominalaorta,thesuperiormesenteric
arterydescendsdowntheposterioraspectoftheabdomen.At
thispoint,ithasseveralimportantanatomicalrelations:
AnteriortotheSMA
?pyloricpartofthestomach,splenicveinand
neckofthepancreas.
PosteriortotheSMA?leftrenalvein,uncinateprocessofthe
pancreasandinferiorpartoftheduodenum.
Theuncinateprocessistheonlypartofthepancreasthathooks
aroundthebackoftheSMA.


MajorBranches
Thesuperiormesentericarterythengivesrisetovariousbranches
thatsupplythesmallintestines,cecum,ascendingandpartofthe
transversecolon(fig).


234.Awomanshowssymptomsofmassive
pulmonarythromboembolism.Thegross
appearanceofliverautopsyisshown.
Whichofthefollowingstatementbest
characterizesthepatient'scondition?

a)MetastasisfromPE
b)Angiosarcoma
c)Colonicadenocarcinomawithmetastasis
d)Locallyinvadedhepatocellularcarcinoma
CorrectAnswer-C
Answer:C.Colonicadenocarcinomawithmetastasis
Thefigureshowsappearanceofmetastaticlesionsfromamalignant
neoplasmwithmultipletumormasses
Theliveristhemostcommonsiteofmetastasesfortumorsitesthat
draininitiallyviatheportalcirculation.Metastaticliverdiseaseis
foundin10%to25%ofpatientshavingsurgeryforprimary
colorectalcancer

colorectalcancer
Surgicalresectionisthemosteffectivetherapyformetastatic
colorectalcancerisolatedtotheliver.


235.WhatisthestructureseeninthegivenX-
raybelow
a)Stent
b)Surgicalclips
c)Foleycatheter
d)Intravesicalwire
CorrectAnswer-A
Ans.A.Stent
PlainabdominalX-rayshowingastentintherightandleftureter.
Uretericstents,alsoknownasdoubleJstentsorretrograde
uretericstents
,isaurologicalcatheterthathastwo"J-shaped"
(curled)ends,whereoneisanchoredintherenalpelvisandthe
otherinsidethebladder.
Ureteralstentsareimplantsusedtoprovidedrainageoftheupper
urinarytractwhenobstructionoftheureterispresentoranticipated.
Thisobstructionmaybeduetointernalorexternalissues,suchas
edemaaftermanipulationoftheureter,ureteralstricture,passageof


stonefragmentsorexternalcompressionoftheureter.Stentsare
alsofrequentlyusedinreconstructivesurgery.
Stent-relatedcomplications
DevelopmentofUTIsorformationofencrustations,complicating
subsequentstentremoval.
bilateraluretericstents


236.Whatisthediagnosisbasedonthe
followingX-ray?
a)UterineFibroid
b)BladderCarcinoma
c)Bladderstone
d)RenalTuberculosis
CorrectAnswer-C
Ans.C.Bladderstone
Multiplebladdercalculi.Fourovalradiopaquebladderstonesare
visiblecentrallyinthepelvis.Mostbladdercalculiareroundoroval,
buttheymayalsobeamorphous,laminated,orevenspeculated.
Calculimayformasaresultofinfection(especiallywithproteusin
anabnormallyfunctioningbladder)whenthecalculiaremagnesium
ammoniumphosphateandapatite.
Calculiareoftenasymptomaticbutmaypresentwithhaematuria,
suprapubicpain,disruptionoftheurinestreamorrecurrentinfection.
Theymayappearasradiopaquespiculated(jackstone),laminatedor
amorphouslesions.
Aswithuppertractcalculi,oncontrastinvestigations,theyare
usuallyseenasfillingdefects,astheyaremostlylessradiopaque


thancontrast.
Ultrasoundreliablydemonstratesthemashighlyechogenic
mobilefoci,especiallywhenthebladderisfull.


237.Aredsofttofirmswellingonsternum
thatonbiopsyshowsfollowing
histology.Whatisthediagnosis?

a)Hemangioma
b)Osteochondroma
c)Osteoidosteoma
d)Pagetdisease
CorrectAnswer-A
Ans.A.Hemangioma
Clinicalpresentation
Thesetumoursareslowgrowingandaregenerallyasymptomatic
unlesstheyexertmasseffectonsensitivestructures.Occasionally
theymaypresentasaswellingorapalpablemass,especiallyinthe
skull.Whenlargeandstrategicallylocatedtheymaypresentwitha
pathologicalfracture.
Iftheyarehigh-flowlesions,shunt-relatedsymptomsmayalsobe
present.
Pathology
Primaryintraosseoushaemangiomasareslowgrowingvascular
neoplasms,usuallylocatedinthemedullarycavity.Theyare

classifiedasbenign,butrarelymaybelocallyaggressive.
Histology
Histologically,intraosseoushaemangiomasdemonstrate
hamartomatousvasculartissuewithinendothelium,butmayalso
containfat,smoothmuscle,fibroustissue,andthrombi.


238.Whichofthefollowingiscarryingagent
forthediseasewithgivencharacteristic
onpolarizedmicroscopy?

a)Anopheles
b)Ixodesscapularisticks
c)Louse
d)Ratflea
CorrectAnswer-B
Answer:B.Ixodesscapularisticks
BabesiosisinfectstheRBCsandresidesinsidetheRBCs(
intraerythrocytic).
IntraerythrocyticinfectionofBabesiosisischaracterizedbythe
maltesecross.
TheMaltesecrossisacharacteristicarrangementofparasiteswithin
theerythrocytesParasiteswithinerythrocytesarearrangedsuch
thatpointedendsoffourparasitescomeincontacttherebygivinga
tetradconfigurationresemblingamaltesecross.
Tetradformsor'Maltesecross'appearanceisconsidered

pathognomicofBabesiosis.
BabesiosiscaneasilybeconfusedwithP.falciparummalaria.
FollowingtwofeaturesdistinguishBabesiosisfrommalaria
PresenceofmaltesecrossinBabesiosis(absentinmalaria)
AbsenceofpigmentHemozoininBabesiosis(presentinmalaria)
Note-
Maltesecrossisalsoseenincryptococcusandaspergillus.


239.Chordomaarisesfrom:
a)Pharyngealbursa
b)Notochord
c)Rathke'spouch
d)Luschka'sbursa
CorrectAnswer-B
Answer:-B.Notochord
Chordomaisarareslow-growingneoplasmthoughttoarisefrom
cellularremnantsofthenotochord.
Chordomascanarisefromboneintheskullbaseandanywhere
alongthespine.Thetwomostcommonlocationsarecraniallyatthe
clivusandinthesacrumatthebottomofthespine.
Therearethreehistologicalvariantsofchordoma:classical(or
"conventional"),chondroidanddedifferentiated.

240.WhatistheeffectofProgesteroneonly
pills?
a)Completelysuppressesovulation
b)Thinliningofuterus
c)Thickcervicalmucus
d)Alloftheabove
CorrectAnswer-D
Answer:D.Alloftheabove
Progestogen-onlypills/Progestin-onlyPills(POP)/Morning
afterpills-
Contraceptivepills
Containonlysyntheticprogestins&doesn'tcontainestrogen.
Mechanism:
Mucusthickeningintheneckofthewomb:
Penetrationofspermtoreacheggandwombbecomesdifficult.
Preventsovulation:
Theliningoftheuterusbecomesthin
Fertilizedeggimplantationisprevented
Advantages:
Doesn'tinterferewithbreastfeeding
Alsohelpsinpremenstrualsymptomsandpainfulperiods

241.Whichmetalresultsin"Saturninegout"
formation?
a)Cadmium
b)Lead
c)Beryllium
d)Mercury
CorrectAnswer-B
Answer:B.Lead
Onemanifestationofchronicleadtoxicityistherheumatologicentity
knownassaturninegout.
Illicitlydistilledbeveragesmayinadvertentlycontainharmfultoxins,
likemetalliclead.
Leadhasbeenknowntoplayaroleinpurinemetabolism&renal
insufficiency

242.Whichdrugdecreasesthebone
resorptioninosteoporosis?
a)Teriparatide
b)Risedronate
c)Cortisone
d)Cimetidine
CorrectAnswer-B
Answer:B-Risedronate
Risedronatebisphosphonatesinhibitsboneresorptionby
actionsonosteoclastprecursorsinosteoporosispatients

Risedronate:
Aminobisphosphonate
Indications:
Prevention&treatmentofosteoporosis
Mechanismofaction:
Inhibitsboneresorptionbyactiononosteoclasts
Reduceboneremodelling
Morepotentinblockingthebonedissolutionprocess.
Teriparatide,anPTHanalog,recombinanthumanPTHisalsoused,
yetinseverecasesofosteoporosis,improvingtheskeletal
microarchitecture

243.Phenylketonuriaisduetodeficiencyof:
a)Phenylalanine
b)Phenylalaninehydroxylase(PAH)
c)Phenylene
d)Allofthese
CorrectAnswer-B
AnswerB.Phenylalaninehydroxylase(PAH)
Abirthdefectthatcausesanaminoacidcalledphenylalanineto
buildupinthebody.
PKUisanautosomalrecessivemetabolicgeneticdisorder.
PKUischaracterizedbyhomozygousorcompound
heterozygousmutationsinthegeneforthehepatic
enzymephenylalaninehydroxylase(PAH),renderingit
nonfunctional.
Thisenzymeisnecessarytometabolizetheamino
acidphenylalanine(Phe)totheaminoacidtyrosine(Tyr).When
PAHactivityisreduced,phenylalanineaccumulatesandis
convertedintophenylpyruvate(alsoknownasphenylketone),which
canbedetectedintheurine.
ThePAHgeneislocatedonchromosome12inthebands12q22-
q24.1.Morethan400disease-causingmutationshavebeenfoundin
thePAHgene.

244.WHICHISNOTCORRECT:
a)MRIneededtoaccesshaemorrhage
b)GCSassessmenthelpsinprognosis
c)Haematomamustbeoperated
d)alloftheabove
CorrectAnswer-C
Answer-C.Haematomamustbeoperated
Treatmentofhematomadependsonthelocation,symptoms,and
theclinicalsituation.Somemayrequirenotreatmentatallwhile
othersmaybedeemedamedicalemergency.
Simpletherapiesathomemaybeutilizedintreatingsuperficial
(undertheskin)hematomas.Mostinjuriesandbruisescanbe
treatedwithresting,icing,compression,andelevatingthearea.This
isrememberedbytheacronymRICE.
Thesemeasuresusuallyhelptoreduceinflammationanddiminish
itssymptoms.
Rest
Ice(Applytheiceorcoldpackfor20minutesatatime,4to8times
aday.)
Compress(Compressioncanbeachievedbyusingelastic
bandages.)
Elevate(Elevationoftheinjuredareaabovetheleveloftheheartis
recommended.)
Medicaltreatmentforahematoma
Forcertainsmallandsymptom-freehematomasnomedical
treatmentmaybenecessary.Ontheotherhand,symptomatic
hematomasorthoselocatedincertainlocationssometimesrequire
medicalorsurgicaltreatment.

Eventhoughnospecificmediationisavailableforthetreatmentof
hematomas,managementofanyrelatedsymptomscanbeachieved
bymedications.Forexample,painfromahematomacanbetreated
withpainmedicationssuchasacetaminophen(Tylenol).
Surgicaldrainageisacommonmethodoftreatmentforcertain
hematomas.

245.Maastrichtclassificationofdonation
aftercardiacdeath.Whatcategoryis
stage3?

a)Awaitingcardiacarrest
b)Broughtindead
c)Unsuccessfulresuscitation
d)Cardiacarrestafterbrain-stemdeath
CorrectAnswer-A
Answer:A>Awaitingcardiacarrest
Thefirstlevelofdefinitionissimpleandbasedonwhetherthe
situationisuncontrolled(categoriesIandII)orcontrolled(categories
III,IV,andV).
CategoryI,thepatientisdeclared"deadonarrival"and,
CategoryII,thereisan"unsuccessfulresuscitation"whetherit
occurredoutorinthehospitalforbothsituations.
CategoryIIIisthemostusualsituationinwhichthetreating
physicianandfamilyare"awaitingcardiacarrest"todeclarethe
deathofthepatient.
CategoryIVisalwayscharacterizedby"cardiacarrestduringbrain
death."ThespecialsituationoftheBelgianlawallowingthe
euthanasiaiselaborated
CategoryV"euthanasia,"andincludespatientswhograntaccessto
medicallyassistedcirculatorydeath.Organdonationafter
euthanasiaisallowedunderthescopeofdonationaftercirculatory

246.Whatisthecauseofmyocardialshock
otherthanMI?
a)acutemitralregurgitation
b)ventricularseptalrupture
c)isolatedrightventricularshock
d)alloftheabove
CorrectAnswer-D
Answer:D>Alloftheabove
Leftventriculardysfunction(LVD)-
Mostfrequentcauseof
cardiogenicshock
Followedby,
Acutemitralvalveregurgitation
Ventricularseptaldefect
Isolatedrightventricularshock
Tamponade/cardiacrupture

247.Nitricoxideactsbyincreasing?
a)BRCA1
b)BRCA2
c)Interleukin
d)cGMP
CorrectAnswer-D
Answer:D>cGMP
Nitricoxidediffusestothesurroundingsmoothmuscle
cells,increasingcGMP.
Cyclicguanosinemonophosphate(cGMP)
Cyclicnucleotidederivedfromguanosinetriphosphate(GTP).
Function:
cGMPactsasasecondmessengermuchlikecyclicAMP.
Mechanismofaction:
Activationofintracellularproteinkinasesinresponsetothebinding
ofmembrane-impermeablepeptidehormones.

248.Positiveacidschiffmacrophagesseenin
?
a)Whipplesdisease
b)Crohnsdisease
c)AIDS
d)Noneoftheabove
CorrectAnswer-A
Answer:A>Whipplesdisease.
Thetraditionallaboratorydiagnosisisbasedonlightmicroscopy,
whichshowsdiastase-resistant,periodicacid-Schiff(PAS)-positive,
non-acid-fastgranulesin.....Thedistinctioncouldbemade
byacid-faststaining,whichispositiveforpatientsinfectedwithM.
aviumandnegativeforthosewithWhipple'sdisease.

249.Aboypresentedwithmultiplenon
suppurativeosteomyelitiswithsicklecell
anaemia.Whatwillbethecausative
organism?

a)Salmonella
b)S.aureus
c)H.influenzae
d)Enterobacterspecies
CorrectAnswer-A
Ans.A.Salmonella
Followingarethevariousmicro-organisminvolvedin
osteomyelitis

Agegroup
Mostcommonorganisms
Newborns(younger S.aureus,Enterobacterspecies,andgroupA
than4mo)
andBStreptococcusspecies
Children(aged4mo S.aureus,groupAStreptococcusspecies,
to4y)
Haemophilusinfluenzae,andEnterobacter
species
Children,
S.aureus(80%),groupAStreptococcus
adolescents(aged4 species,H.influenzae,andEnterobacter
ytoadult)
species
Adult
S.aureusandoccasionallyEnterobacteror
Streptococcusspecies
Sicklecellanemia
Salmonellaspeciesaremostcommonin
patients
patientswithsicklecelldisease

250.Termpathologyrefersto:
a)Work
b)Function
c)Details
d)Explains
CorrectAnswer-A
Ans.A.Work
Pathology
isthemedicaltermforthewayadiseaseworks.
Thescienceofthecausesandeffectsofdiseases,especiallythe
branchofmedicinethatdealswiththelaboratoryexaminationof
samplesofbodytissuefordiagnosticorforensicpurposes.

251.1yrchildweighing6kgissufferingfrom
AcuteGastroenteritisalongwithsignsof
sunkeneyes&skinpinchgoingbackto
normalveryrapidly.Whatwillbeyour
management?

a)RLinfusion120mlinthefirsthourfollowedby360mlinthe
next5hours
b)RLINFUSION180mlinthefirsthourfollowedby420mlinthe
next5hours
c)RLINFUSION180MLINthefirsthourfollowedby480mlinthe
next5hours
d)RLINFUSION240mlinthefirsthourfollowedby360mlinthe
next5hours
CorrectAnswer-B
Ans.B.RLINFUSION180mlinthefirsthourfollowedby420ml
inthenext5hours
Severedehydrationconstitutesamedicalemergencyrequiring
immediateresuscitationwithintravenousfluids.
Intravenousaccessshouldbeobtained,andpatientsshouldbe
administeredabolusof20-30mL/kglactatedRinger's(LR)or
normalsaline(NS).
Ifpulse,perfusion,and/ormentalstatusdonotimprove,asecond
bolusshouldbeadministered.
Afterthis,thepatientshouldbegivenaninfusionof70mL/kgLRor
NSover5hours(children<12months)or2.5hours(olderchildren).
Ifnoperipheralveinsareavailable,anintraosseouslineshouldbe
placed.Serumelectrolytes,bicarbonate,urea/creatinine,and

placed.Serumelectrolytes,bicarbonate,urea/creatinine,and
glucoselevelsshouldbesent.

252.Whatconstitutesmalpighianlayer?
a)Corneumlucidum
b)Corneumspinosum
c)Spinosumandbasale
d)Basalegranulosum
CorrectAnswer-C
Ans.C.Spinosumandbasale
TheMalpighianlayeroftheskinisgenerallydefinedasboththe
stratumbasaleandstratumspinosumasaunit,althoughitis
occasionallydefinedasthestratumbasalespecifically,orthestratum
spinosumspecifically.ItisnamedafterMarcelloMalpighi.

253.Mechanismofactioncolchicineinacute
gout
a)Uricacidnephrolithiasis
b)DeficiencyofenzymeXanthineoxidase
c)Increaseinserumurateconcentration
d)Renaldiseaseinvolvinginterstitialtissues
CorrectAnswer-B
Ans.B.DeficiencyofenzymeXanthineoxidase.
Goutisahereditarydisorderwithincreaseinserumuricaciddue
toincreasedproduction,ordecreasedexcretionofuricacidanduric
salt.
Thoughttobecausedbylackofanenzymeneededtocompletely
metabolisepurinesforrenalexcretion.
Table1.MetabolicRiskFactorsforGout
Obesity,eating13w-1nel-richfoods(highlevelsofmeatand
seafoodconsumption)
Excessivealcoholintake
Metabolicsyndrome
Obesity
Hypertension
Hyperlipidernia
Hyperglycemia
Type2diabetesmellitus
Hypertension
Hyperlipidernia
Serumurate--elevatingmedications
Historyofurolithiasis
Chronickidneydisease,glomerular,orInterstitialrenaldisease;

Chronickidneydisease,glomerular,orInterstitialrenaldisease;
polycystickidneydisease
Potentialgeneticoracquiredcauseofuricacidoverproduction,
includingmalignancy
Lead,orheavy-metalintoxication

254.Oxygentherapymaynotbeusefulin
a)Asthma
b)Pneumonia
c)Subglotticstenosis
d)Pulmonaryfibrosis
CorrectAnswer-D
Ans:D.Pulmonaryfibrosis
ManyEMSprotocolsindicatethatoxygenshouldnotbewithheld
fromanypatient,whileotherprotocolsaremorespecificor
circumspect.However,therearecertainsituationsinwhichoxygen
therapyisknowntohaveanegativeimpactonapatient'scondition
likeparaquatpoisoning,pulmonaryfibrosisandlungdamage
resultingfrombleomycintreatment.

255.Newbornbabywithheartratelessthan
60beatsperminutecanberesuscitated
byallexcept

a)chestcompression
b)oxygentherapy
c)tactilestimulation
d)slappingtheback
CorrectAnswer-D
Ans:D.slappingtheback
Thefirstthreesareindicatedwhileslappingthebackisnot
recommendedinanewbornwhohas
Heartratelessthan60beatsperminute.



256.Mobitztype2seconddegreeAVblockis
seeninallexcept:
a)Hypothyroidism
b)CoronaryArteryDisease
c)Sarcoidosis
d)Cushingsyndrome
CorrectAnswer-D
Ans:D.Cushingsyndrome
Delayorlackofconductionthroughtheatrioventricular(AV)node
andbelowhasmultiplecauses.Degenerativechanges(eg,fibrosis,
calcification,orinfiltration)arethemostcommoncauseofnon-
ischeamicAVblock.IdiopathicfibrosisorcalcificationoftheAV
conductionsystem,commonlyseenintheelderly,cancause
completeAVblock.
CausesofMobitztype2seconddegreeAVblockare-
Damageoftheconductionsystemfromcoronaryarterydisease,
valvesurgery,myocardialinfarction,myocarditis,infiltrative
cardiomyopathies(sarcoidosis,hemochromatosis),myxedema,
Lymedisease,neuromusculardisease,andAVjunctionablation[6]

257.Whencanonediagnoseacute
respiratorydistressinachild?
a)Within7daysofknownclinicalinsult
b)Respiratoryfailurenotfullyexplained
c)Leftventriculardysfunction
d)Alloftheabove
CorrectAnswer-D
Ans:D.Alloftheabove
PediatricCriticalCareMedicine2015
Excludepatientswith1:m11-natalrelatedlung
Age
disease
Iiming
Within7daysofknownclinicalInsult
originof
Respiratoryfailurenotfullyexplainedbycardiacfailure
Edema
orfluidoverloadChest
Chest
imagingfindingsofnewinfiltrate(siconsigentwithacute
Imaging
pulmonary
Noninvasivemechanical lovasivemechanical
ventilation
ventilation-
PARRS(Noseverity
Mild
Moderate Severe
gratification)
Oxygenation Fullface-maskbi-level
EK01<
O1t16
ventilationor
4501c8
16
CiSt
CPAPz5crnHAa
Ss09.c7.51 7.5OSI
1233
PFratios300
<12.31
SFratio52E4I
Specia-lPopulations
tandardCriteriaaboveforage,timing,originofedema
Cyanotic

Cyanotic
andchestimagingwithanacutedeteriorationIn
Heart
oxygenationnotexplainedbyunderlyingcardiac
Disease
disease.
standardCriteriaaboveforage,timing,andoriginof
Chronic
edemawithchestimagingconsistentwithnewinfiltrate
Lung
andacutedeteriorationinoxygenationfrom
Disease
baselinewhichmeetoxygenationcriteriaabove_
StandardCriteriaforage,timingandoriginofedema
Left
withchestimagingchangesconsistentwithnew
Ventricular infiltrateandacutedeteriorationinoxygenationwhich
dysfunction meetcriteriaabovenotexplainedbyleftventricular
dysfunction.

258.A6-year-oldboyexperiencedlife
threateningshock,hisCTscanshowed
largeamountofascites,bowelwall
thickeningandpoororabsent
enhancementofthestrangulatedbowel
segment,showinggangrenousbowelon
surgicalexploration.
Trueaboutanastomosisis-

a)Shouldbedonebycontinuouslayersasittakeslesstime
b)ShouldbeDonewithcatgut
c)ShouldbeDonebysinglelayerseromuscularlembertsutures
d)ShouldbeDonebySinglelayertakingsubmucosa
CorrectAnswer-C
AnsC.ShouldbeDonebysinglelayerseromuscularlembert
sutures
DiagnosisisofcongenitalIHwithstrangulatedsmallbowelwith
gangrenoussmallbowel
Transmesentericherniawasthemostcommontypeinolderchildren
aswellasinneonates.
IHresultsfromincompleteclosureofsurgicallycreatedmesenteric
defects,andusuallyacquiredresultingfrompreviousabdominal
surgeryespeciallyRoux-en-Yanastomosis
Anastomosisshouldbedonebysinglelayerseromuscularlembert
sutures
TheLambertsuturegenerallyisusedinabdominalsurgery.Itisan
invertingsuture,thatcanbeeithercontinuousorinterrupted,usedto

invertingsuture,thatcanbeeithercontinuousorinterrupted,usedto
jointwosegmentsofanintestinewithoutenteringthelumen(the
innerchannelthroughwhichstomachcontentsflow).

259.InACLSwhichdrugcanbegiven
followingventricularfibrillationafter
cardiacarrestotherthanepinephrine?

a)Amiodarone
b)Dopamine
c)Adenosine
d)Atropine
CorrectAnswer-A
Ans.A.Amiodarone
V-FiborVFisthemostcommonrhythmthatoccursimmediately
aftercardiacarrest.Inthisrhythm,theheartbeatswithrapid,erratic
electricalimpulses.
Treatment:
Shock/Defibrillation:every2minutesinasingleoneshock,
successive,shockableincrements
200joules-FollowedbyimmediateCPRfor2minutes/giveand
circulateadrug(s)
300joules-FollowedbyimmediateCPRfor2minutes/giveand
circulateadrug(s)
360joules-FollowedbyimmediateCPRfor2minutes/giveand
circulateadrug(s)
Drugs:
GiveEpinephrine1mgofa1:10,000solu,onevery3to5minutes
[NoLimit]
Giveeither:
Amiodarone[ifnotcontraindicated,canbegiven2x]:300mgfirst
dose/150mgseconddoseat3to5minutesincrements.

Lidocaine:Firstdose:1mg/kgor1.5mg/kg.Canrepeatitathalfthe
originaldoseuptoatotalof3mg/kg[Secondandremainingdoses
aregivenateither0.5mg/kgor0.75mg/kgdependingonyourstar,ng
dosage.]

260.Whichofthefollowingstatementisfalse
aboutMRvaccinationcampaign
launchedbyWHO?

a)Childrenfrom9monthstolessthan15vaccinated
b)Congenitalrubellasyndrome(CRS),responsibleforirreversible
birthdefects
c)Indiahasnotyetlaunchedthiscampaign
d)Willreplaceroutineimmunizationformeaslesvaccine
CorrectAnswer-C
Answer:C.Indiahasnotyetlaunchedthiscampaign
Oneoftheworld'slargestvaccinationcampaignagainstmeasles,a
majorchildhoodkillerdisease,andcongenitalrubellasyndrome
(CRS),responsibleforirreversiblebirthdefects.
India,alongwithtenotherWHOSouthEastAsiaRegionmember
countries,haveresolvedtoeliminatemeaslesandcontrol
rubella/congenitalrubellasyndrome(CRS)by2020
Allchildrenfrom9monthstolessthan15yearsofagewillbegiven
asingleshotofMeasles-Rubella(MR)vaccinationduringthe
campaign
Followingthecampaign,MRvaccinewillbecomeapartofroutine
immunizationandwillreplacemeaslesvaccine,currentlygivenat9-
12monthsand16-24monthsofageofchild.
Forthosechildrenwhohavealreadyreceivedsuchvaccination,the
campaigndosewouldprovideadditionalboostingtothem.

261.Whichofthefollowingtrueregarding
HemophiliaA
a)SerumlevelsoffactorVIIIaredecreased
b)DeficiencyoffactorIX
c)PTincreased
d)FITdecreased
CorrectAnswer-A
Answer:A.SerumlevelsoffactorVIIIaredecreased
HemophiliaisanXlinkeddisorderofcoagulationcausedbythe
deficiencyinacirculatingplasmaprotein.HemophiliaAiscausedby
thedeficiencyoffactorVIII,andhemophiliaBiscausedbythe
deficiencyoffactorIX.
ItisPTTwhichisaffected(increased)andnotPT(unaffected).
FactorVIIIisinvolvedintheintrinsicpathwaywhichismeasuredby
PTTandnotinextrinsicpathwaywhichismeasuredasPT.
Bleedingisthecommonmanifestationofhemophiliaandthe
commonbleedingmanifestationsarehemarthoses,hematomas,
mucocutaneousbleeding,intracranialbleeding,hematuriaand
pseudotumor.

262.Markedbleedingisseeninwhichof
followingconditions?
a)VMAdisease
b)HaemophiliaA
c)HaemophiliaB
d)ALL
CorrectAnswer-D
Answer:D.ALL
Bleedingdisorderscanbeinheritedoracquired.Inheriteddisorders
arepasseddownthroughgenetics.Acquireddisorderscandevelop
orspontaneouslyoccurlaterinlife.Somebleedingdisorderscan
resultinseverebleedingfollowinganaccidentorinjury.Inother
disorders,heavybleedingcanhappensuddenlyandfornoreason.
Therearenumerousdifferentbleedingdisorders,butthefollowing
arethemostcommonones:
HemophiliaAandBareconditionsthatoccurwhentherearelow
levelsofclottingfactorsinyourblood.Itcausesheavyorunusual
bleedingintothejoints.Thoughhemophiliaisrare,itcanhavelife-
threateningcomplications.
FactorII,V,VII,X,orXIIdeficienciesarebleedingdisorders
relatedtobloodclottingproblemsorabnormalbleedingproblems.
vonWillebrand'sdisease
isthemostcommoninheritedbleeding
disorder.ItdevelopswhenthebloodlacksvonWillebrandfactor,
whichhelpsthebloodtoclot.

263.Reedsternbergcellsarefoundin
a)Hodgkin'sdisease
b)Sicklecellanaemia
c)Thalassemia
d)CML
CorrectAnswer-A
ReedSternbergcellsareyouderivedfromBlymphocytes,
classicallyconsideredcrippledgerminalcenterBcells,Seenagainst
aseaofBcellswhichgivethetissueamoth-eatenappearance.
Theyarelargeandareeithermultinucleatedorhaveabibbed
nucleus(thusresemblingan"owl'seye"appearance)withprominent
eosinophilicinclusion-likenucleoli.
TheyareCD30andCD15positive,usuallynegativeforCD20and
CD45.
Thepresenceofthesecellsisnecessaryforthediagnosisof
Hodgkin'slymphoma-theabsenceofReed-Sternbergcellshasvery
highnegativepredictivevalue.
Theycanalsobefoundinreactivelymphadenopathy(suchas
infectiousmononucleosis,carbamazepineassociated
lymphadenopathy)andveryofteninothertypesofnon-Hodgkin
lymphomas.


264.BywhichmethodforeignDNAis
introducedintoacellbyavirusorviral
vector?

a)Transduction
b)Transcription
c)Lysogenicconversion
d)Transformation
CorrectAnswer-A
Ans.A.Transduction
Transduction
istheprocessbywhichforeignDNAisintroduced
intoacellbyavirusorviralvector.Anexampleistheviraltransfer
ofDNAfromonebacteriumtoanother.

265.Whichoneofthefollowingshows
allostericinhibition?
a)Malonicacid&succinate
b)2,3BPG
c)Aminoacidalanine&pyruvatekinase
d)Citrate
CorrectAnswer-B
Answer:B.2,3BPG
Negativeallostericmodulation(alsoknownasallosteric
inhibition
)occurswhenthebindingofoneliganddecreasesthe
affinityforsubstrateatotheractivesites.Forexample,when2,3-
BPGbindstoanallostericsiteonhemoglobin,theaffinityfor
oxygenofallsubunitsdecreases.

266.Whichofthefollowingisseenin
seropositiverheumatoidarthritis?
a)Multiplejointsaffected
b)Symmetricaljointsymptoms
c)Jointpainandswelling
d)All
CorrectAnswer-D
Answer:D.All
PositiveforRheumatoidfactorinbloodisseropositivity.
Patientswithpositiverheumatoidfactorusuallypresentwith
symptomslike
Jointdeformities&disability
Symmetricalinvolvementofjoints
Inflammation
Swellingandpainfulinmultiplejoints
,especiallyofhandsand
feet.
Morningstiffness(shortterm)
Developmentoffirmlumpsnearjoints-"Rheumatoidnodules"
Deteriorationofbone&cartilage(X-rayfindings)


267.Whichofthefollowingisnotseenin
Anteriormediastinum
a)Thyroidtumour
b)Thymoma
c)Lymphoma
d)Neurogenictumor
CorrectAnswer-D
Answer:D.Neurogenictumor
Theanteriormediastinumistheportionofthemediastinumanterior
tothepericardiumandbelowthethoracicplane.
Itformstheanteriorpartoftheinferiormediastinum
containsthethymus,lymphnodes,andmaycontaintheportionsof
aretrosternalthyroid.

MediastinalTumorsandOtherMasses
Superior
Anterior
Posterior
Middle
Mediastinum
Mediastinum
Mediastinum
Mediastinum
Neurogenic
Bronchogenic
Lymphoma
Thymoma
tumors
cyst
Thymoma
Teratoma
Lymphoma
Pericardialcyst
Gastroenteric
Thyroidlesions
Lymphoma
Lymphoma
hernia
Metastatic
Thyroidlesions
carcinoma
Parathyroid
Parathyroid
tumors
tumors


268.Struvitestoneiscausedbywhichmetal?
a)Magnesium
b)Calcium
c)sodium&potassium
d)both(a)&(b)
CorrectAnswer-A
Ans.A.Magnesium
Struvite,acrystallinesubstanceiscomposedofmagnesium
ammoniumphosphate(MgNH4PO4?6H2O).
Struviteurinarystoneshavealsobeenreferredtoas"infection
stones"and"triplephosphate"stones.
Struvitestones
canbecausedbyalkalineurine,steroidtherapy,
abnormalretentionofurine,aurinarytractinfection,oranother
disorderoftheurinarytract.
Therearefiveprimarytypesofcommonlyencounteredurinary
stones,i.e.,calciumoxalate,calciumphosphate,magnesium
ammoniumphosphate,uricacid,andcystine.

269.Whichofthefollowingstatementsabout
Gravesdiseaseisfalse?
a)Resultsinhyperthyroidism
b)Autoimmunedisorder
c)CommoninMale
d)ReferredasToxicdiffusegoitre
CorrectAnswer-C
Ans.C-Commoninmale
Graves'disease:
Autoimmune
systemdisorder
Bothmenandwomengetaffected;
Yet,10timesmorecommoninwomenthanmen
Affectsyoungerwomen<40years
Resultsinoverproductionofthyroidhormones(hyperthyroidism).
Signsandsymptoms:
Anxiety
Irritability
Heatsensitivity
Increasedperspiration/warmandmoistskin
Weightloss
Goiter(Glandularenlargement)
Menstrualcyclechanges
Erectiledysfunction/reducedlibido
GravesOphthalmopathy-Bulgingeyes-Exophthalmos
Gravesdermopathy
-Thick,redskinonshins/topoffeet.
Antibodyforgravesdisease-Thyrotropinreceptorantibody
(TRAb)actsonthe
regulatorypituitaryhormoneinterferingthe
normalsecretionofthyroxine.

TRAboverridesnormalregulationcausinganoverproductionof
thyroidhormones(hyperthyroidism).


270.Aldosteronesynthesisisstimulatedbywhichofthe
following?
a)ACTH
b)Hyperkalemia
c)Hypernatremia
d)Exogenoussteroids
CorrectAnswer-B
Ans.B.Hyperkalemia
Mineralocorticoidsecretionisstimulatedbyhyperkalemia,
angiotensin-II,ACTHandhyponatremia,inreducingorderof
efficacy.
1.Aldosteronesecretioninresponsetohyperkalemiaisthemost
importantandformsthebasisforrenalregulationofbodypotassium
balance.
2.StimulationofaldosteronebyangiotensinII(throughtherenin-
angiotensinsystem)isimportantforthecorrectionofhypovolemia
andhypotensioninconditionslikesaltdepletionorrenalischemia.
3.StimulationofaldosteronesecretionbyACTHresultsindiurnal
variationofaldosteronesecretion.However,ACTHisnotan
importantphysiologicalregulatorforaldosteronesecretion.
4.Hyponatremiaisaweakstimulatorofaldosteronesecretion

271.Whichofthefollowingisfalseabout
Alzheimer'sdisease?
a)Onein10peopleage65andolderhasAlzheimer'sdisease
b)Alzheimer'sdiseaseiscurable
c)Causedementia
d)Alloftheabove
CorrectAnswer-B
Ans.B.Alzheimer'sdiseaseiscurable
Alzheimer'sdisease
Alsocalled:seniledementia.Aprogressive
diseasethatdestroysmemoryandotherimportantmental
functions.
Memorylossandconfusionarethemainsymptoms.
Currently,thereisnocureforAlzheimer's.Butdrugandnon-drug
treatmentsmayhelpwithbothcognitiveandbehavioralsymptoms.
ThetreatmentsavailableforAlzheimer'sdonotsloworstop
theprogressionofthedisease,buttheymayhelpwiththe
symptomsforatime.
TherearethreecholinesteraseinhibitorstotreatAlzheimer's:
Donepezil(Aricept)
Rivastigmine(Exelon)
Galantamine(Reminyl)
Peoplemayexperience:
Cognitive
:mentaldecline,difficultythinkingandunderstanding,
confusionintheeveninghours,delusion,disorientation,
forgetfulness,makingthingsup,mentalconfusion,difficulty
concentrating,inabilitytocreatenewmemories,inabilitytodosimple
maths,orinabilitytorecognisecommonthings
Behavioural:aggression,agitation,difficultywithselfcare,

irritability,meaninglessrepetitionofownwords,personalitychanges,
restlessness,lackofrestraint,orwanderingandgettinglost
Mood:anger,apathy,generaldiscontent,loneliness,ormood
swings
Psychological:depression,hallucination,orparanoia
Alsocommon:behavioralsymptoms,inabilitytocombinemuscle
movements,jumbledspeech,orlossofappetite

272.Whichofthefollowingistrueabout
vitaminK?
a)Anticoagulant
b)Prolonguseofantimicrobialleadstodeficiency
c)Dietaryallowanceis15-20mg
d)Alloftheabove
CorrectAnswer-B
Ans.B.Prolonguseofantimicrobialleadstodeficiency
Certainpeopleareatincreasedriskifthey:
takecoumarinanticoagulantssuchaswarfarin,whichthinsthe
blood
aretakingantibiotics
haveaconditionthatcausesthebodytonotabsorbfatproperly(fat
malabsorption)
haveadietthatisextremelylackinginvitaminK
VitaminKisagroupofstructurallysimilar,fat-solublevitaminsthe
humanbodyrequiresforcompletesynthesisofcertainproteinsthat
areprerequisitesforbloodcoagulationandwhichthebodyalso
needsforcontrollingbindingofcalciuminbonesandothertissues
ThebodyneedsvitaminKtoproduceprothrombin,aproteinand
clottingfactorthatisimportantinbloodclottingandbone
metabolism.
WithoutvitaminK,bloodcoagulationisseriouslyimpaired,and
uncontrolledbleedingoccurs.Preliminaryclinicalresearchindicates
thatdeficiencyofvitaminKmayweakenbones,potentiallyleading
toosteoporosis,andmaypromotecalcificationofarteriesandother
softtissues
Dietaryallowanceforadultsperday-50-100mg.


273.Whichdrugsneedscontinuous
monitoringofprothrombintime?
a)Aspirin
b)Lepirudin
c)Digoxin
d)Coumadin
CorrectAnswer-D
Ans.D.Coumadin
Coumadin
(warfarin)isananticoagulant.
Warfarinisacoumarinanticoagulantusedfortheprophylaxisand
treatmentofthromboemboliccomplicationsassociatedwithcardiac
valvereplacementandatrialfibrillation,aswellastheprophylaxis
andtreatmentofvenousthrombosisandpulmonary
embolism.Increasedmetabolismofwarfarinresultsininsufficient
prolongationofprothrombintime.

274.Whichofthefollowingaretheriskfactor
forcutaneouslymphoma?
a)Age
b)Gender
c)Weakenedimmunesystem
d)All
CorrectAnswer-D
Answer:D.All
RiskFactorsforLymphomaoftheSkin
AgeAgeisanimportantriskfactorforthisdisease,withmostcases
occurringinpeopleintheir50sand60s.Butsometypesofskin
lymphomacanappearinyoungerpeople,eveninchildren.
GenderandraceMost(butnotall)typesofskinlymphomaaremore
commoninmenthaninwomen.Mostalsotendtobemorecommon
inAfrican-Americansthaninwhites.Thereasonsforthisarenot
known.
WeakenedimmunesystemSkinlymphomasmaybemorecommon
inpeoplewithacquiredimmunodeficiencysyndrome(AIDS),who
haveaweakenedimmunesystem.Theymayalsobemorecommon
inpeoplewhohavehadanorgantransplantsuchasaheart,kidney
orlivertransplant.Thesepeoplemusttakedrugsthatsuppresstheir
immunesystem,whichmayraisetheriskofskinlymphoma(or
lymphomasinotherpartsofthebody).
InfectionsInfectionwiththehumanimmunodeficiencyvirus(HIV),
thevirusthatcausesAIDS,mayincreaseaperson'sriskofskin
lymphoma.

275.WhichisnotincludedinAIDSrelated
complex?
a)Ectopicpregnancy
b)Recurrentgenitalcandidiasis
c)Generalisedlymphadenopathy
d)Chronicdiarrhea
CorrectAnswer-A
Answer-A.Ectopicpregnancy
HIVsymptoms:AIDSrelatedcomplex(ARC)
ItbelongstoclassBofHIVsymptoms.Thepatientsatthisstage
havevariousdiseasesthatoccurbecausetheHIvirushas
weakenedtheimmunesystem.
ThefollowingHIVsignsmayhavepatientswithARC:
Long-lastingdiarrhea(overfourweeks)
Unintendedheavyweightloss
Longlastingfever
Nightsweats
Bacterialinfectionscausedbybacteria
Bacterialbloodpoisoning(sepsis)
Phthisis
Herpeszoster
Oralhairyleukoplakia(whitishchangesonthelateraltongueborder)
Fungicausedbyfungi
HIVsymptoms?Women:vaginalinflammationcausedbyfungi,
malignantchangesinthecervix

276.Whichisthetreatmentofchoicefor
irradiationinChordoma?
a)Protons
b)Electrons
c)Gammaradiation
d)3D-CRT
CorrectAnswer-A
Answer:A-ProtonTherapy
Chordoma:
Slow-growingneoplasm
Arisingfromcellularremnantsofnotochord.
Arisefromboneinskullbaseandalongspinalcord.
Mostcommonlocations-
Craniallyatclivus
Insacrumatbottomofspine
Radiationtherapy:
Arerelativelyradioresistant
Highdosesofradiationrequiredtocontrol.
Hence,highfocusradiationlikeprotontherapyandcarbonion
therapyarepreferredthanconventionalradiationmethods.
Closeproximitytovitalstructureslikebrainstem,requireshigh
precisionandaccuracyforanyplannedsurgicalresection.
Radiationwithhighaccuracyandminimaldamagewithmaximal
safetyisdelivered.

277.Whatdifferentiatesdeliriumfrom
dementia?
a)Confusion
b)Difficultyincommunicating
c)Hallucination
d)Suddenchange
CorrectAnswer-D
Answer:D.Suddenchange
Delirium
Alsocalledtheacuteconfusionalstate,deliriumisamedical
conditionthatresultsinconfusionandotherdisruptionsinthinking
andbehavior,includingchangesinperception,attention,moodand
activitylevel.
Indementia,changesinmemoryandintellectareslowlyevident
overmonthsoryears.Deliriumisamoreabruptconfusion,emerging
overdaysorweeks,andrepresentsasuddenchangefromthe
person'spreviouscourseofdementia.Thinkingbecomesmore
disorganized,andmaintainingacoherentconversationmaynotbe
possible.
Thehallmarkseparatingdeliriumfromunderlyingdementiais
inattention.Theindividualsimplycannotfocusononeideaortask.

278.Genitalwartsarecausedbywhichvirus?
a)Herpessimplex
b)Humanpapilloma
c)Cytomegalovirus
d)Varicellazoster
CorrectAnswer-B
Ans.B.Humanpapilloma
Genitalwarts
aresoftgrowthsthatappearonthegenitals.Genital
warts
areasexuallytransmittedinfection(STI)causedbycertain
strainsofthehumanpapillomavirus(HPV).Theseskingrowthscan
causepain,discomfort,anditching.

279.Whichdrugregimenisgivenina
pregnantwomanwithHIVinfection?
a)Tenofovirdisoproxilfumaratewithemtricitabine
b)Tenofovirdisoproxilfumaratewithlamivudine
c)Abacavirwithlamivudine
d)All
CorrectAnswer-D
Ans.D.All
PreferredRegimensforHIVAntiretroviralTherapy(ART)in
Pregnancy
Two-NRTIbackbone
Regimensincludethefollowing:
Tenofovirdisoproxilfumaratewithemtricitabine(TDF/FTCco-
formulated)ortenofovirdisoproxilfumaratewithlamivudine(3TC)
oncedaily(usewithcautioninrenalinsufficiency)or
Abacavirwithlamivudine(ABC/3TC)oncedaily(onlyifHLA-B5701?
negative);avoidcombinationwithritonavir-boostedatazanavirifthe
pretreatmentHIVviralloadexceeds100,000copies/mL.
ForwomenwhohavenevertakenHIVmedicines,thepreferredHIV
regimenshouldincludetwonucleosidereversetranscriptase
inhibitors(NRTIs)
plusanintegrasestrandtransferinhibitor
(INSTI),anon-nucleosidereversetranscriptaseinhibitor
(NNRTI)
,oraproteaseinhibitor(PI)withlow-doseritonavir(brand
name:Norvir).
Theregimengenerallyshouldincludeatleastoneofthe
followingNRTIsthatpasseasilyacrosstheplacenta:
abacavir
(brandname:Ziagen)
emtricitabine(brandname:Emtriva)

lamivudine(brandname:Epivir)
tenofovirdisoproxilfumarate(brandname:Viread)
zidovudine(brandname:Retrovir)

280.Whichofthefollowingstructure
developsfromdorsalmesentery?
a)Greateromentum
b)Lesseromentum
c)Liver
d)Diaphragm
CorrectAnswer-A
Ans.A.Greateromentum
Theportionofthedorsalmesenterythatattachestothegreater
curvatureofthestomach,isknownasthedorsalmesogastrium.The
partofthedorsalmesenterythatsuspendsthecolonistermedthe
mesocolon.Thedorsalmesogastriumdevelopsintothegreater
omentum.

281.WhichofthefollowingisthebestStent
forFemoropoplitealBypass?
a)Dacron
b)Reversedsaphenous
c)PTFE
d)None
CorrectAnswer-C
Answer:C-PTFE-coveredself-expandingnitinolstents
PTFE-coveredstentsareengineeredwitha30?100-micronpore
sizetoallowfortheendothelialliningofthestent-graftandvessel
healing.
Thetwomaingrafttypesusedforlowerextremitybypassesarethe
greatsaphenousveinsandpolytetrafluoroethylene(PTFE)grafts.
Oneofthemostwidelyusedstent-graftsinthetreatmentofchronic
lowerextremityischemiaistheViabahnendoprosthesis(Gore
Medical,Flagstaff,Ariz).
Itisconstructedwithexpandedpolytetrafluoroethylene(ePTFE)liner
attachedtoanexternalnitinolstent.
Theinnersurfaceisbondedwithheparin.

282.Trileneisdegradedby:
a)EnzymaticDegradation
b)NonEnzymaticdegradation
c)ChemicalDegradation
d)None
CorrectAnswer-A
Answer:A.EnzymaticDegradation
Trileneortrichloroethyleneisagoodanalgesic,lessdepressant,
andnon-flammable.
Cardiacdysrhythmia,ortachypnoeamayoccurduring
administration.
Itshouldnotbeusedintheclosedcircuitasitreactswithsoda-lime
toproduceatoxicgas(phosgene).
Recoveryisslowandnauseaaswellasvomitingmaybepresent.
Itshouldnotbeusedwithadrenalineinfiltrationlestdysrhythmiabe
convertedtoventricularfibrillation.
Degradation:
Broughtaboutbyenzymaticdegradation
Theenzymethatstartsonebranchofthispathway,toluene1,2-
dioxygenase,hasmanyothercatalyticabilities,whichare
documentedinatableoftheReactionsofToluene1,2-Dioxygenase.
Thespontaneousdegradationoftrichloroethyleneepoxidecan
produceasmanyasfourproducts:dichloroacetate,carbon
monoxide,glyoxylate,andformate.Thenumber,type,and
proportionofproductsseendependsonthelocalenvironment.

283.Theearliestfeatureof3rdcranialnerve
involvementindiabetesmellituspatient
is-

a)Normallightreflex
b)Abnormallightreflex
c)Normallightandaccommodationreflex
d)Abnormallightandaccommodationreflex
CorrectAnswer-A
Answer:A.Normallightreflex
Theoculomotornerveisthethirdcranialnerve.Itenterstheorbit
viathesuperiororbitalfissureandinnervatesmusclesthatenable
mostmovementsoftheeyeandthatraisetheeyelid.Thenervealso
containsfibersthatinnervatethemusclesthatenablepupillary
constrictionandaccommodation(abilitytofocusonnearobjectsas
inreading).Theoculomotornerveisderivedfromthebasalplateof
theembryonicmidbrain.Inpeoplewithdiabetesandolderthan50
yearsofage,anoculomotornervepalsyoccurs.

284.Duringsquintsurgery,anesthesiologist
seesthemachineandseethebp
suddenlydropsto40.Whatwillbebest
immediatemanagement-

a)Giveatropine
b)Increaselevelofanesthesia
c)Askthesurgeontostopthesurgery
d)Giveadrenaline
CorrectAnswer-D
Answer:D.Giveadrenaline
Adrenalineshouldbegiventoraisethebloodpressure.
Epinephrine,alsoknownasadrenalinoradrenaline,is
ahormone,neurotransmitter,andmedication.Epinephrineis
normallyproducedbyboththeadrenalglandsand
certainneurons.
Itplaysanimportantroleinthefight-or-flightresponseby
increasingbloodflowtomuscles,outputoftheheart,pupil
dilation,andbloodsugar.Itdoesthisbybinding
toalphaandbetareceptors.
Physiologicresponsestoepinephrinebyorgan

Organ
Effects
Heart
Increasesheartrate;contractility;conductionacross
AVnode
Lungs
Increasesrespiratoryrate;bronchodilation
Systemic
Vasoconstrictionandvasodilation
Liver
Stimulatesglycogenolysis

Systemic
Triggerslipolysis
Systemic
Musclecontraction

285.Allarespecialvisceralefferentcolumn
except-
a)Glossopharyngealn
b)Nucleusambiguus
c)vagusnerve
d)trigeminalnerve
CorrectAnswer-B
Answer:B.Nucleusambiguus
Specialvisceralefferentfibers
(SVE)aretheefferentnervefibers
thatprovidemotorinnervationtothemusclesofthepharyngeal
archesinhumans,TheonlynervescontainingSVEfibersarecranial
nerves:thetrigeminalnerve(V),thefacialnerve(VII),the
glossopharyngealnerve(IX),thevagusnerve(X)andtheaccessory
nerve

286.WhichofthefollowingconditionisNOT
causedbyParvovirusB19?
a)Roseolainfantum
b)Aplasticanemiainsicklecelldisease
c)Fetalhydrops
d)Erythemainfectiosum
CorrectAnswer-A
Answer:A.Roseolainfantum
PrimaryinfectionbyparvovirusB19oftenproducesanacute,
severe,andsometimesfatalanemiamanifestedasarapidfallinred
bloodcellcountandhemoglobin.
Thesepatientsmaypresentinitiallywithnoclinicalsymptomsother
thanfever;thisiscommonlyreferredtoasaplasticcrisis.
Erythemainfectiosum
(alsoreferredtoasfifthdiseaseoracademy
rash)isamorecommondiseasethatisclearlyattributableto
parvovirusB19.
ActivetransplacentaltransmissionofparvovirusB19canoccur
duringprimaryinfectionsinthefirst20weeksofpregnancy,
sometimesresultinginstillbirthoffetusesthatareprofoundly
anemic.
Theprogresscanbesoseverethathypoxicdamagetotheheart,
liver,andothertissuesleadstoextensiveedema(hydropsfetalis).

287.Whichofthefollowingstatementsisnot
trueaboutiliolumbarligament?
a)Upperfibresattachedtoiliaccrest
b)Lowerfibresattachedtobaseofsacrum
c)Helpinmaintaininglumbosacraljointstability
d)UpperattachmenttotransverseprocessofT12
CorrectAnswer-D
Answer:D-UpperattachmenttotransverseprocessofT12
TheligamentattachestoL5
Iliolumbarligament:
Strongligamentpassingfromthetipoftransverseprocessoffifth
lumbarvertebra
toposteriorpartofinnerlipofiliaccrest
Upperbandsgetsattachedtotheiliaccrest.
Lowerbands
getsattachedtobaseofsacrum.
Majorfunctionistostrengthenthelumbosacraljoint.

288.Wherewillbetheplacementlocationfor
AuditoryBrainstemImplant?
a)Scalatympani
b)Recessof4thventricle
c)IAC
d)backofear
CorrectAnswer-B
Answer:B-Recessof4thventricle.
Theimplantisusuallyplacedinthelateralrecessofthefourth
ventricleatthetimeoftumorresectiontostimulatethe
cochlearnucleus

AuditoryBrainstemImplant(ABI):
TumorresectionsurgeryinNFpatientsresultincochlearnerve
damageorlossoffunctionofnerveresultingindeafness.
ABIareusefulinrestoringauditoryperceptiontodeafpatientswith
neurofibromatosistype2(NF2)
Alsousedintreatmentofcongenitallydeafchildrenwithcochlear
malformationsorcochlearnervedeficiencies.
Placementlocation:Lateralrecessof4thventricle

289.Whichconditionisassociatedwith
Congenitaladrenalhypoplasia?
a)Malepseudohermaphroditism
b)Femalepseudohermaphroditism
c)Truepseudohermaphroditism
d)Sequentialpseudohermaphroditism
CorrectAnswer-B
Answer-B-
CongenitalAdrenalHyperplasia:
Thisisthemostcommoncauseofandrogenicexcessinfetuseswith
femalepseudohermaphroditism.
Thehyperplasticglandssynthesizedefectiveenzymesthatcause
impairedcortisolsynthesis.
ThisleadstoexcessivepituitaryACTHthesecretionofthefetal
adrenalglandswithsecretionoflargeamountsofcortisol
precursors,includingandrogenicprehormones.
Theseprehormones,forexample,androstenedione,areconverted
totestosteroneinfetalextra-adrenaltissues.

290.Whichistrueregardingataxia
telangiectasia:
a)IncreaseinAFP
b)Increasestheriskofsquamouscellcarcinoma
c)Autosomaldominant
d)Noneofabove
CorrectAnswer-A
Answer:A-IncreaseinAFP
Increaseinalpha-fetoproteinisobservedinAtaxia
telangiectasia

Ataxia-telangiectasia/Ataxia-telangiectasiasyndrome/Louis-Bar
syndrome
Rare,neurodegenerative,autosomalrecessivedisordercausing
severedisability.
Ataxiareferstopoorcoordination;Telangiectasiareferstosmall
dilatedbloodvessels.
Partsaffected:
Cerebellum-movement&coordinationdifficulties
Immunesystem-Predisposingtoinfections.
Geneticrepairsystem-PreventingprocessforrepairingDNA-
Cancerrisk
Features:
Increasedincidenceoflymphoma&Leukemia
Increasedalpha-Fetoproteinlevels
Oculomotorapraxia
(difficultyincoordinationbetweenhead&eye
movements)
Dysarthria

291.Adiabeticpatient2daysafterpost
cataractsurgerydevelopsdevelops
hypopyon.Whatwillbethe
management?

a)Intravitrealantibiotics
b)Eyedrops
c)Surgery
d)Notreatmentrequired
CorrectAnswer-A
Ans.A.Intravitrealantibiotics

292.

WhatistheThinnestpartofneuro-retinalrim
accordingtoISNTrule?

a)Inferior
b)Superficial
c)Temporal
d)Medial
CorrectAnswer-C
Ans.C.Temporal
TheISNTruleisaneasywaytorememberhowtheopticnerveis
supposedtolookinanormaleye.Normallytheneuro-retinalrimis
thickestInferiorlyandthinnestTemporally.Withglaucoma,however,
youbegintoseeverticalthinning,withatrophyalongtheinferiorand
superiorrims.

293.Leidenthrombophiliaiscausedby
mutationaldeficiencyofwhichofthe
followingfactors?

a)FactorV
b)FactorVII
c)FactorIX
d)FactorX
CorrectAnswer-A
Ans.A.FactorV
FactorVLeiden
thrombophiliaisaninheriteddisorderofblood
clotting.FactorVLeidenisthenameofa
specificmutation(geneticalteration)thatresultsinthrombophilia,or
anincreasedtendencytoformabnormalbloodclotsinblood
vessels.FactorVLeidenisthemostcommoninheritedformof
thrombophilia.

294.Anteversionofuterusismaintainedby?
a)Cardinalligament
b)Uterosacralligament
c)Pubocervicalligament
d)Roundligament
CorrectAnswer-D
Ans.D.Roundligament
Inmostwomen,theuterusisantevertedandanteflexed.The
functionoftheroundligamentismaintenanceoftheanteversionof
theuterus(apositionwherethefundusoftheuterusisturned
forwardatthejunctionofcervixandvagina)duringpregnancy.
Normally,thecardinalligamentiswhatsupportstheuterineangle
(angleofanteversion).

295.Long-standingpelvicinflammationmay
leadtowhichofthefollowing
conditions?

a)Pyometra
b)Uterinepolyposis
c)Pseudopregnancy
d)Cysticendometrialhyperplasia
CorrectAnswer-A
Ans.A.Pyometra
Pyometraiscollectionofpusduetoobstructionofflowintheuterine
cavity.
ItmaybeduetoLong-standingPIDorsecondarytocervical
stenosis.

This post was last modified on 30 July 2021